MV First assist

Ace your homework & exams now with Quizwiz!

Once the integument is broken, what are the chances of bacterial colonization and contamination?

1

As far as antiseptics go, Povidine-iodine are no different than ___________?

Sterile water in decreasing bacterial wound contamination

What are the goals during induction for AAA repair?

Stress-less induction Minimal CV effect Graded stimulation techniques Tight hemodynamic control Euvolumia or mild hypovolemia

What arteries are in the superior area of spinal cord perfusion?

Subclavian and Vertebral

If ICP is of major concern following a craniotomy, the craniotomy plate may be left off to give the brain room to swell.

True

T/F We can have a pulmonary embolism and a DVT?

True

Postoperative infection in total hip arthroplasty is

Usually catastrophic

Virchows Triad is associated with DVT; what is in virchows triad?

Venous stasis, Vein endothelial damage, hypercoaguable state (often decreased AT III)

Excessive secretion of growth hormones in adults causes

acromegaly

Some patients may show the most severe and lifethreatening type of reaction to certain drugs known as

anaphylaxis

The assistant must equal the surgeon's _____ and ________ __________to provide countertraction

angle and direction EXACTLY

Dextran is used parenterally to

expand blood plasma volume

Which arteries supplies blood to the thyroid gland?

external carotid

Which nerve must be preserved to prevent muscle weakness during a parotidectomy?

facial

Traction

force placed on an object in a particular direction

Identify the main artery of the thigh.

femoral

Which nerves must be identified during an inguinal hernia repair?

iliohypogastic and ilioinguinal

When performing an inguinal hernia repair and incising the external oblique muscle, the sensory nerve that innervates the scrotum and must be preserved is the

ilioinguinal

What is the princial goal of suctioning blood and body fluids fromt the wound?

improve wound visibility

What is lactational mastitis?

infection of the breast during breastfeeding - MC caused by S. aureus. Tx with Abx and follow for abscess formation

The arterial blood supply to the breast is supplied from branches of the

internal mammary, axillary and lateral thoracic

Osteoarthritis

is degenerative joint disease

What should not be administered to a patient who is exhibiting asystole?

isoproterenol

Which position would be the most optimal for a pilonidal cycstectomy?

kraske

What is a fibrocartilage structure of the acetabular fossa that encloses the head of the femur?

labrum

A method t protect tissue from the CO2 laser is to cover the area with: A. Dry towels B. Steri drape C. Ioban drape D. Wet towels

D. Wet towels

Nephron sparing (partial nephrectomy) help prevent renal insufficiency's including:

Dialysis

A type C aneurysm is also known as what?

Dissecting

This is widely used as initial screening test for DVT

Doppler

Femoral hernias occur more frequently in which group of individuals

Females

Identify the main artery of the thigh.

Femoral

Direct hernia is a weakness in the:

Floor of the inguinal canal

What dye is used in opthalmic surgery to diagnose corneal abrasions?

Fluorescein sodium

Parenchyma is

Functional tissue of the kidney

A type A aneurysm is also known as what?

Fusiform

Using Wagner classifiction, grade the following ulcer: An ulcer with Subcutaneous involvement

Grade 2

How does tamoxifen work?

It binds estrogen receptors

This suture has the least relative tissue reaction of nonabsorbale sutures

Monofilament POLYPROPYLENE

Which cranial nerves innervate the inferior, medial, superior rectus, inferior oblique, and levator palpebrae muscle of the eye?

Oculomotor (III)

What is an intraoperative hazard of laparascopic surgery?

perforation

Which nerve innervates the the diaphram?

phrenic

A miotic drug is

pilocarpine

Ulnar nerve damage could result from

placement of an elbow on unpadded table

What blood componenets are essential in clot formation?

platelets

In the lateral chest position, a sand bag, or padding is placed under the chest at the axillary level to

prevent pressure on the lower arm

Know the following vessels

inferior mesenteric vein, superior mesenteric vein, superior hemorrhoidal vein, splenic vein, right gastric artery, portal vein

Intercostal nerves and vessels are located

inferior to the rib

During total hip arthroplasty through an anterolateral approach, which of the following maneuvers is needed to dislocate the femoral head?

adduction and external rotation

Which of the following angled plates is MOST frequently chosen for a compression hip screw fixation?

135 degrees

Jackson's membrane would be found near the: A. Cecum B. Ascending colon C. Descending colon D. Rectum

A. Cecum

A broad spectrum bactericidal antibiotic that interferes with synthesis of the cell wall and has a chemical structure similar to the penicillins is: A. Cefazolin B. Aminoglycoside C. Tetracycline D. Sulfonamide

A. Cefazolin

Prone: LE Positioning *Knees are slightly ?

*Legs padded and flexed slightly at knees and hips *Pillows can be placed under ankles *Legs padded and flexed slightly at knees and hips *Pillows can be placed under ankles

Sciatic Nerve: Mechanisms of Injury *this nerve is most commonly injured in ? position

*Lithotomy position *Especially at risk if the patient is thin, the table hard, and the operation long *Most commonly injured due to the lithotomy position (especially if the duration is longer than 4 hours) *Compression -As the nerve passes under the piriformis muscle *Stretch -External rotation of the leg or extension of the knee *IM injection

Lateral Decubitus: Head/Neck Positioning

*Make sure are aligned with the spine *Excessive neck rotation may cause brachial plexus injury *Pressure points -Eye/ear -Foam donut commonly used

Prone Surgeries: Examples

*Neurosurgeries of Posterior Fossa *Cervical/Thoracic/Lumbar Spinal Fusions *Renal Surgeries *Anal fistulomy *Achilles Tendon Repair

Brachial Plexus: Signs/Symptoms

*Non-cardiac surgery -Motor deficits in the upper and middle roots (median and radial nerves) *Cardiac surgery (associated with median sternotomy) -Sensory deficits in the lower nerve root (ulnar)

Sitting Surgeries: Examples

*Shoulder Arthroscopy or Replacement *Neurosurgery for Posterior Fossa

___Compression is a force that changes with the changing outside forces presented to the fracture site.

Dynamic

List the characteristics of deep tissue injury. Deep tissue injury is characterized by a purple or maroon localized area of discolored intact skin or a blood-filled blister.

List the characteristics of Stage I pressure ulcer. Stage I is characterized by intact skin with nonblanchable redness of a localized area, usually over a bony prominence.

Trendelenberg Surgeries: Examples

*Laparascopic Abdominal Hysterectomy *Cystoscopy *Robot Assisted Laparoscopic Prostatectomy

Which opthalmic medications are injected into the anteior chamber during a cataract procedure to aid in keeping the iris in position prior to inserting the lens prosthesis?

acetylcholine chloride

Drains are typically passed to the exterior through?

adjacent stab wound incision

A white count of 14,000 cu/mm would most likely indicate

an infection

The most common skin cancer is

basal cell carcinoma

Which organ is frequently trapped in the inguinal sac of female patients?

ovary

A 70-year-old patient who underwent an 8-hour colon resection is at the greatest risk for what?

postoperative infection

What is the purpose of a jejunojejunostomy?

to prevent the reflux of bile and pancreatic fluids into the esophagus.

General and regional anesthesia contribute to body heat loss by causing

vasodilation

What are two types of the thoracic aorta?

visceral branches and parietal branches

High doses of nitrous oxide block the metabolism of what vitamin?

vitamin B12

An injury to the ulnar nerve could cause

claw hand

The wound classification of a thoracotomy is

clean-contaminated wound

If you are not mirroring the surgeon during surgery you are not able to provide proper _________

countertraction

The single most important concept in the role of the surgical assistant is

countertraction

Radical Nephrectomy includes removal of:

kidney, adrenal gland, surrounding tissue

What do the lumbar arteries supply?

lumbar vertebrae and muscles

What does the lymphatic system do?

lymph is collection from ECF and carried to other parts of the body. Lympatic ducts drain into the venous system.

What are the sites of breast CA metastasis?

lymph nodes (MC) lung/pleura liver bones brain

The major vessels are located on which aspect of the thigh at the mid-shaft level?

medial

Increased WBC, bilirubin, alkaline phosphatase and positive blood culture would be associated with

cholangitis

What condition can be postop complication of compression plating?

localized osteoporosis

The legal doctrine that mandates every professional to carry out his or her duties according to national standards of care practiced throughout the country is the: A. Doctrine of reasonable man B. Doctrine of res ipsa loquitur C. Doctrine of superior D. Doctrine of informed conscent

A. Doctrine of reasonable man

Q. A high pitched, short, soft sound produced by percussion over dense tissue is known as which of the following?

A. Dullness Explanation: Dullness results from percussion over dense tissue such as the liver or heart. It is a high-pitched, short, soft sound

In which incision could retention sutures he used? A. Vertical midline B. McBurney C. Transferse D. Thoracoabdominal

x

Sutures placed in a wound to prevent wound evisceration care called: A. Stent B. Fixation C. Retention D. Traction

x

Physiologic Changes in Trendelenberg Position: Pulmonary

*Significant decrease in lung capacities

A chalazion is a chronic inflammation of the

meibomian gland

What types of stitch patterns provides an excellent cosmetic closure?

subcuticular stitch

Ability to hold an instrument in part of the hand while using the rest of the hand to accomplish another task is an example of

two hands/ten fingers

In what order should clamps be removed?

vein first, then artery

During hemostasis, prothrombin and thromboplastin, along with calcium ions in the blood, form: A. Fibrinogen B. Thrombin C. Fibrin D. ADP

x

Supine Position

*Head- neutral alignment, gel type donut pillow to prevent alopecia *Arms -Tucked- elbow next to body, parallel to legs and trunk padded and secured, palm toward body -Abducted- < 90 degrees, supinated, padded and secured, head in neutral position to prevent brachial plexus stretch *Legs -Don't cross, pillow under legs to alleviate lumbosacral strain

Reverse Trendelenberg Surgeries: Examples

*Laparoscopic Cholecystectomy *Laparoscopic Surgeries of the upper abdomen

Physiologic Changes in Sitting Position: Cardiovascular

*Decreased Cerebral Volume *Decreased CO and ABP despite incresed HR and SVR

What nerve runs LATERAL to or through the pectoral minor muscle, actually lateral to the lateral pectoral nerve? What does it innervate?

medial pectoral nerve • innervates the pectoral minor and pectoral major muscles

Which nerves are affected by carpal tunnel syndrome?

median

What is a narcotic antagonist?

Naloxone hydrochloride (Narcan)

When are Contact Precautions implemented? Contact Precautions are implemented for patients known or suspected to be infected or colonized with epidemiologically important organisms that can be transmitted by direct or indirect contact.

Which heat-resistant, spore-forming microorganism is used to test steam sterilizer effectiveness? Geobacillus stearothermophilus is used for steam sterilizers.

What is the most frequent site of AAA?

aortoiliac bifurcation (L4 level)

The usual location of a Meckel's diverticulum is

approx. 2 feet from the ileocecal valve

What is the preop staging workup in a pt with breast cancer?

bilateral mammogram CXR (check for lung mets) LFTs (check for liver mets) serum Ca level, alkaline phos (if they indicate bone mets, do bone scan) head CT if pt has focal neurologic deficits for brain mets

What is the MC breast tumor in patients younger than 30 years?

fibroadenoma

Which anastomoses is accomplished during a Whipple procedure (pancreaticoduodenectomy)?

hepaticojejunostomy

Impaired healing of a vertical abdominal incision can result in

herniation

Significantly decreased volume of circulating blood or plasma in the body may cause the patient to go into

hypovolemic shock

The kidney lift should be situated directly below what part of the patient when positioning a patient for a nephrectomy?

iliac crest

What are the major types of invasive breast carcinoma?

invasive ductal carcinoma (90%) invasive lobular carcinoma (10%) inflammatory carcinoma

What is the lymphatic drainage of the breast (2)?

lateral - axillary lymph nodes medial - parasternal nodes that run with internal mammary artery

The celiac artery trifurcates into the

left gastric, splenic, and hepatic arteries

What types of stitches incoporates the seromuscular layer and submucosa?

lembert

A drug used to reverse hypotension is

levartenernol (levophed)

What is the principal muscle of the pelvic floor?

levator ani

The two muscles that form the pelvic diaphragm are

levator ani and coccygeus

Which nerves travel along the anterior-lateral chest wall and innervates the serratus anterior muscle?

long thoracic

What 4 nerves must the surgeon be aware of during an axillary dissection?

long thoracic nerve thoracodorsal nerve medial pectoral nerve lateral pectoral nerve

When transferring a thoracotomy patient to recovery, the chest drainage system should be

maintained below the patient's chest level

What are the causes of breast abscess? What is MC bacteria?

mammary ductal ectasia (stenosis of breast duct) and mastitis nursing = staph aureus nonlactaing = mixed infection

Anatomical landmarks for a laparoscopic inguinal hernia include all of the following EXCEPT: *inguinal ring *inferior epigastric vessels *spermatic vessels *medial scrotal sac

medial scrotal sac

Which nerves are compressed in carpal tunnel syndrome?

median

What is an abnormal activiated clotting time?

more than 120 seconds

What are complications of internal K-wire fixation f a Colles' fracture?

nerve damage

What are the parietal branches of the abdominal aorta?

phrenicoabdominal arteries, lumbar arteries, deep circumflex iliac arteries, external iliac arteries, internal iliac arteries, median sacral artery

During lateral positioning, a

pillow is placed between the legs

Cell "drinking" is called

pinocystosis

During a posterolateral approach for an intracapsular hip fracture, which of the following muscles would be released along with the short external rotators and capsule from the posterior aspect of the femur as a single musculocapsular flap?

piriformis

When a loop of jejunum is anastomosed end to side with the esophagus in a total gastrectomy, a jejunojejunostomy may be done to

prevent regurgitation esophagitis

What is the desired outcome for the Bankart procedure?

preventing recurrence of dislocation, reducing arthritic changes, and restoring joint motion

The reversal agent for heparin-induced anticoagulant is

protamine sulfate

The principal goal of the surgical assistant during the surgical procedure is to

provide exposure

What is a continuous circular suture placed around the lumen of a structure causing it to close?

purse-string

Which nerve is at risk for injury in a humerus shaft fracture?

radial

Which nerves are at risk for injury when a humeral shaft fracture occurs?

radial nerve

What involves small, corneal incisions as treatment for myopia?

radical keratotomy

Which lymph nodes are located between the pectoralis major and minor muscles?

rotter

What type of biopsy is performed when a nonpalpable breast mass is seen on mammogram?

sterotactic (mammotome) biopsy or needle localization biopsy

The nerve of Grassi is associated with the

stomach

You are planning an elective surgery on a patient on LMWH, what should be the protocol

stop it 15 hours before surgery

The incision for the tracheostomy is approx. 1/4" above the

suprasternal notch

If a surgical assistant places the operative patient's well-being above all other factors, then the s.a. is aid to have a strong

surgical conscience

Which of the following is considered a non-impregnated, nonadherent dressing? a. adaptic b. vaseline gauze c. telfa d. xeroform

telfa

In addition to the zygoma, which other bone comprises the zygomatic arch?

temporal

The five branches of the facial nerve from

temporal, zygomatic, buccal, madibular, cervical

Which incision is indicated for an esophagogastrectomy?

thoracoabdominal

What nerves are located lateral to the long thoracic nerve and innervates the latissimus dorsi muscle

thoracodorsal

What is the epiploic foramen of winslow?

vertical slit about 2.5 cm at the right border of the lesser sac Anteriorly the is bounded by right free margin of lesser sac containing the portal vein, hepatic artery and bile duct, Posteriorly, the peritoneum overlying the IVC Superiorly, the caudate process of the liver Inferiorly, the first part of the duodenum

A method to protect tissue fromt the CO2 laser is to cover the area with

wet towels

An injury to the long thoracic nerve could cause

winged scapula

Using the Wagner classification of neurotrophic ulcers, what is observed in grades 0-5?

0-Partial dermal erosion; 1-full thickness dermal erosion; 2-SubQ involved; 3-Extends to tendon/capsule/bone; 4-Local gangrene; 5-Entire foot gangrene. .

Common Peroneal Nerve Injury: Signs/Symptoms * ? drop occurs with this nerve injury

*Foot drop *Loss of dorsal extension of the toes *Inability to evert the foot

A stapedotomy is removal of the stapes superstructure and creation of a fenestra (opening) in the fixed stapes footplate for treatment of abnormal bone growth around the footplate that results in its immobility. A prosthesis is placed to restore ossicular continuity and alleviate conductive hearing loss. The patient scheduled for a stapedotomy will probably have a surgical diagnosis of:

a. otosclerosis.

An organelle that is responsible for the packaging of proteins is the

golgi complex

Know the nerves in the axillary region

lateral pectoral long thoracic thoracodorsal medial pectoral

Which arteries could be injured if the screws for an anterior cervical discectomy are placed laterally rather than straight or at a medial angle?

vertebral

Which diagnostic imaging techniques are used to aid in diagnosing a rotator cuff tear?

mri

Which nerve can be injured if the legs are improperly flexed at the hip int he lithotomy position?

obturator

Ulnar Nerve Injury: Signs/Symptoms *inability to abduct the ? finger *sensation diminished in ? and ? fingers *Atrophy of intrinsic muscles causes ?

*Inability to abduct or oppose the 5th finger *Diminished sensation in 4th and 5th fingers *Atrophy of the intrinsic muscle of the forearm causing a "claw hand" *Delayed onset -Possible postop injury? -Documentation is key **Not always preventable

Name an indication for treating a mid-shaft fracture of the femur with compression plating.

A. Third degree open fracture with a compromised blood supply B. Multiple trauma C. No access to rodding system or the correct sized rod

Which of the following refers to a civil wrong committed against a person or property, not including a breach of contract? A. Tort B. Malpractice C. Assault and battery D. Negligence

A. Tort

Q. An emergency cart should be in the operating suite at all times. This cart should include all of the following except?

A. Traction equipment Explanation: An emergency cart would not include traction equipment. An emergency cart should include: Intravenous fluids Resuscitation medications Life-support equipment Intravenous access equipment

During a bunion surgery, to relieve tension, what type of incision should be done

An incision adding a slight curve to a linear incision across a joint helps to reduce tension

What are some advantages of the endovascular AAA approach?

Avoid Abdominal Incision Lack of Aorta X-Clamping Potential for Regional/MAC Anesthesia Shortened Hospital Stay (2 days average)

What are the rules for Neuraxial Anesthesia and Unfractionated Heparin?

Avoid if other coagulopathies present Heparin given >1 hr post insertion Remove catheter 1 hr prior to or 2-4 hrs post heparin dose Low concentrations of local post-op to facilitate evaluation

The normal pH of urine is: A. 1.2- 4.6 B. 4.6- 8.0 C. 8.0-1 1.4 D. 11.4- 14.0

B. 4.6- 8.0

At which vertebral level is the odontoid process located? A. C1 B. C2 C. C7 D. L5

B. C2

The thyroid gland consists of right and left lobes joined by the: A. Larynx B. Isthmus C. Parathyroid gland D. Cricoid cartilage

B. Isthmus

Adipose tissue lies in which of the following skin layers? A. Epidermal B. Subcutaneous C. Dermal D. Reticular

B. Subcutaneous

HOw is sharp debridement differentiated from surgical debridement?

By the location of the procedure

List examples of devices commonly used during prone positioning. The Wilson frame and the Jackson spinal surgery bed are often used during prone positioning.

Describe the reverse Trendelenburg position. In the reverse Trendelenburg position, the patient is supine with the upper torso elevated and the feet down.

When positioning for a subdural hematoma using a horseshoe headrest, be sure to

Extend the neck 10 degrees in order to lower the vertex

What are the six folds of peritoneum of the anterior abdominal wall?

Falciform Ligament Median Umbilical ligament Medial Umbilical Ligament Lateral Umbilical Ligament

True or False? Beta blockers are contraindicated for AAA repair.

False

True or False? Epidurals are contraindicated for AAA repair.

False

The part of the femur that articulates with the acetabulum is the

Femoral head

What is selective clamping?

Finding the artery that is connected to tumor and only clamping that.

Wounds in which there is significant bacterial contamination, foreign bodies, or extensive tissue trauma are generally closed by which intention? A. Delayed primary B. Primary union C. Lag D. Granulation

Granulation

A patient who is opposed to the acceptance of blood transfusion on the religious principle is undergoing a hysterectomy. During surgery, there was significant blood loss causing hematocrit to drop to 30% with a resulting drop in oxygen saturation. Under these conditions, what is the appropriate response?

Increase IV fluid drip to allow hemodilution

What should you assess for after the cross-clamp is applied?

Increased Afterload Left Ventricular Failure Myocardial Ischemia

What is a "sentinel" node biopsy?

Instead of removing all the axillary lymph nodes, the primary draining or sentinel lymph node is removed

What arteries are in the mid thoracic area of spinal cord perfusion?

Intercostals

This full thickness graft is more and contracts less, but more difficult to heal, overall best choice

Intermediate 0.012-0.016 in

What are the potential complications after a MRM?

Ipsilateral arm lymphedema, infection, and injury to nerves, skin flap necrosis, hematoma/seroma, phantom breast syndrome

If you make an incision perpendicular to the to RSTLs

It will cause gap under tension

Describe the left infracolic space

Larger than the right, quadrilateral in shape Upper border - attachment of transverse mesocolon left is the descending colon and the left paracolic gutter lower - attachment of sigmoid mesocolon

The piriformis, obturator internus, and gemelli act to

Laterally rotate the thigh

What law states that wall tension = transmural pressure X vessel radius? (T = P x r)

Law of Laplace

Which bony prominence of the proximal femur is located on the medial aspect?

Lesser trochanter

The primary drug utilized to treat MH is: A. Diazepam B. Dalalone C. Dantrolene D. Dienestrol

C. Dantrolene

In cataract surgery, a viscous jelly sometimes used to occupy space is: A. Alpha chymotrypsin B. Mannitol C. Healon D. Wydase (R)

C. Healon

A needle biopsy of the lung may be accomplished with a

Chiba

What are the spleno renal and gastrosplenic ligaments?

..... extension of the greater omentum to the spleen and from the spleen to the kidney but don't understand how this works the kidney is retroperitoneal!

When scar tissue is laid down in a wound, how strong is it, and what is it made of?

Collagen, its about 80% of original tensile strength.

Ryan has an indirect hernia that is characterized by a small neck thin walls and close attachment to the cord structures. Mark has a hernia with a short wide neck and a thick walled sack. Based on this description Ryan has a _____hernia and Mark has a ______hernia

Congenital; acquired

This needle is used in deep tissue and capsule

Cutting needle- 3-O material

A herniated disc most commonly occurs in which section of the vertebral column?

Lumbar

a direct inguinal hernia occurs

Medial to the inferior epigastric artery

The sphincter located at the distal end of the common bile duct that opens into the duodenum is the: A. Oddi B. Vater C. Wirsung D. Pyloric

A. Oddi

When assisting the surgeon with the draping, where should the surgical assistant stand in relation to the surgeon? A. On the opposite side of the O.R. table from the surgeon B. On the same side of the O.R. table as the surgeon C. Near the foot of the O.R. table D. At the head of the O.R. table

A. On the opposite side of the O.R. table from the surgeon

Q. LEEP stands for which of the following?

A. Loop electrode excision procedure Explanation: LEEP stands for loop electrode excision procedure The loop electrode excision procedure is a electrosurgical loop technique which can be used to help remove a core of tissue from the patient's cervical canal.

Q. The _____ must be consulted when the nurse is creating goals and plans?

A. Patient Explanation: The goals and expected outcomes are based on the needs and expectations of the patient. You need patient buy in for patients to achieve the outcomes. The family may assist, but the patient must be consulted.

Common pathogens infecting traumatic wounds often require the use of: A. Penicillinase-resistant penicillin B. Aminoglycoside C. Ampicillin D. Tetracycline

A. Penicillinase-resistant penicillin

Which nerve innervates the diaphragm? A. Phrenic B. Thoracic C. Median D. Thoracodorsal

A. Phrenic

During lateral positioning, a: A. Pillow is placed between the legs B. Sandbag is placed between the knees C. Rolled towel is placed under the bottom leg D. Sheet is folded flat between the legs

A. Pillow is placed between the legs

CHAPTER 7

NEEDLES, SUTURES, INSTRUMENTS

This suture has the greatest relative tissue reaction of nonabsorble sutures

Natural fiber materials

This suture material has the relative least strength of non absorbable sutures (TQ)

Natural fibers (silk, cotton, linen)

Which of the following lasers is used to vaporize a portion of the residual lens capsule after cataract surgery?

Nd: YAG

It is a surgical instrument used to hold and drive a surgical needle during the suturing, it is made of non corrosive, high strength, good quality steel allows jaws

Needle Holder

What is the primary indication for discectomy?

Neural compression

This type of ulcer is AKA Diabetic foot ulcer, or Mal perforans

Neurotrophic ulcer

Is fibrocystic disease a risk factor for breast CA?

No

The sphincter located at the distal end of the common bile duct that opens into the duodenum is the

Oddi

What risk factors get 3 points each in the risk evaluation of DVT?

Over 70, prior hx of PE, inherited thrombophilia, acquired thrombophilia

Patient Cart Positioning

Over right should at 15 degree angle

What is a measurement of the pressure of carbon dioxide in the arterial blood?

PaCO2

What comprises the lining of a herniated abdominal sac?

Peritoneum

An Exmarch bandage is MOST frequently used for which of the following types of surgery?

Podiatric

This is a symptomatic CVI after DVT

Post phlebetic syndrome

What step does Firefly occur in?

Step 3

The 3-teniae coli converge at what point?

b. The cecum at the base of the appendix

Substituting a cryo-preserved cadaveric vein or an in situ or reversed vein graft from the patient for an arterial bypass graft is an option for patients who have healthy, easily accessed saphenous veins. How does an in situ vein graft differ from a reversed vein graft?

b. The in situ method leaves the vein in its position and the valves are removed.

Select the diagnostic and interventional radiology procedure that is not classified as a vascular procedure.

a. Percutaneous nephrolithotomy

During a laminectomy what is the preferred form of hemostasis?

c. Cottonoid strips or patties

Seepage of skin preparation solution under a patient may result in a

chemical burn

The carotid body and carotid sinus are two sensory structures; the carotid body is a

chemoreceptor

Which cells secrete pepsinogen?

chief

What type of patients are at risk for developing latex sesitivity?

children with spina bifida

What dressing is derived from bovine, equine, or porcine sources and is effective in recalcitrant wounds?

collagen

When given as a preoperative medication, atropine produces

drying of secretions

Lidocaine with epinephrine is contraindicated in surgery of the

hand

What where are additional superficial veins located?

in the appendages as well as wherever needed

Which affect renal vascular resistance?

mannitol

Which nerve that innervates the muscles around the chin must be protected from injury when repairing a fracture of the madible?

marginal mandibular

What should be sent postop with the patient who has arch bars applied?

wire scissors

Which hernia involves the peritoneum and te abdominal viscera forms what part of the hernia sac?

sliding

A left subcostal incision is typically used for

splenectomy

Is occluding temporary or permanent?

temporary

What might contraindicate medullary rodding?

the proper positioning for rodding of the multiple traumatized patient may not be feasible

The avascular area located in the mesentery and to the left of the middle colic artery is

the space of riolan

What is the right subphrenic space? What is the left subphrenic space?

the spaces of the supracolic compartment in relation to the falciform ligament the right is to the right of falciform the left is to the left of falciform Closed superiorly by the coronary ligament - right triangular ligament - left

Infants are at risk of losing body heat quickly due to their

thin layer of subcutaneous brown fat for insulation

What nerves innervates the latissimus dorsi muscle?

thoracodorsal

A ventriculoperitoneal shunt is used to

treat hydrocephalus

During a laparoscopic hernia repair, stapling is avoided in the triangular area inferior to the internal inguinal ring called

triangle of doom

A cross section of the mid-shaft of the femur would appear

triangular

The great sensory nerve of the face and head is the

trigeminal

Removal of all liver tissue to the right of the falciform ligament is called?

trisegmentectomy

Damage to which nerve causes "winged scapula"?

long thoracic nerve "winged scapula" is the name of the deformity if you cut the long thoracic nerve in the axillary

In the patient with a fractured femur, the RNFA will

*examine the patient for possible associated injurieds to the knee and hip *Anticipate extensive soft tissue injury *examinethe patient for possible injuries to the sciatic nerve and superifiial femoral artery and vein

What is normal cardiac output for an adult?

4-8L/minute

This is a removal of necrotic and fibrotic tissue from wound base, the goal is to achieve 100% granular base

Debridement

What characteristics of draping materials is true?

flame-retardant

The most commonly identified ovarian cyst is the

follicular

The superior margin of the hip bone s called the

iliac crest

Cardiac arrhythmias of ventricular origin can be treated with

lidocaine

The flexure at the junction of the duodenum and jejunum is the

ligament of treitz

How long does the inflamm phase last?

0-3days; recovery from wound conversion

The isotonic solution that is closest to human physiology is

0.9 NS

Where are the drains placed with an MRM?

1. axilla 2. chest wall (breast bed)

What is the lifetime risk of breast cancer (the incidence)? What percentage of women with breast cancer have no known risk factor?

12% 75%

What is the ideal PSI for wound irrigation?

15 PSI

The correct range for the pneumatic tourniquet pressure on an average adult for an upper extremity is

250-300mmHg

Grade and stage the following ulcer using the UTHS San antonio classification. Ulcer is uninfected, ischemic and extends into tendon.

2C.

Normal adult range for potassium is

3.5 - 5.5 mEq/L

how long is the proliferative phase?

3d till wound is healed; this is the wound modulation state

Water constitutes what average normal percentage of total body weight?

50-70%

What is the normal value of the BUN for adults?

7-8 mg/dl

Gerota's fascia is another name for:

Adipose capsule

CHAPTER 5

ANESTHESIA

Cell "drinking" is called: A. Phagocytosis B. Pinocytosis C. Osmosis D. Diffusion

B. Pinocytosis

A hemodynamic complication in the patient could be caused by: A. Administration of 100% oxygen B: Myocardial ischemia C. Premature ventricular contractions D. Neurological thoracic outlet syndrome

B: Myocardial ischemia

What are the major breast cancer susceptibility genes (2)?

BRCA 1 BRCA 2

This suture is an absorbable synthetic, the first absorbable synthetic suture, provides superb knot strength

Dexon

What is a laminectomy?

Excision of the laminae to expose the spinal canal

Foman, Cottle, and Aufricht retractors are all considered which of the following kinds of instrument?

ENT

CHAPTER 4

MAINTAINING ASEPTIC PRACTICE

Postop complications of an appendectomy include stenosis of the ileocecal valve T/F?

Postop - False

Which muscle of the hip joint does not attach to the greater trochanter?

Quadratus

CHAPTER 8

SURGICAL MODALITIES

The coronary arteries originate at the base of the ascending aorta, aka

Sinuses of Valsalva

Describe the fascia lata.

The deep fascia of the thigh investing all of the thigh muscles like an elastic stocking

What is the medial umbilical folds and what do they contain?

The medial umbilical ligament passes from ..... to the umbilicus it contains the obliterated remains of the umbilical artery

What might contraindicate medullary rodding?

The proper positioning for rodding of the multiple traumatized patient may not be feasible

During ventriculoperitoneal shunt placement

The valve assembly is soaked in NS & antibiotic solution, never lubricated.

A disadvantage of an anterolateral incisional approach to the femoral shaft is

The vastus lateralis muscle is incised requiring longer rehab; increased pain and bleeding

Costal facets are located on which vertebrae?

Thoracic

Putting a Z

The arms of the Z should be equal in length and the angle should be 60 degrees. An angle of 60% result in 75% increase in length, central arm is drawn along the line of contracture

Steven Morganstein, a 51-year-old male with hepatobiliary disease, is scheduled for surgery. The perioperative nurse, during the preoperative assessment, questions and examines the patient for signs and symptoms of jaundice, petechiae, and lethargy and:

a. reviews his chart for bleeding and coagulation times and the platelet count

Being surgically ____________ is very important

ambidextrous

Which of the following types of laser is used in retinal surgery?

argon

The correct arm placement when positioning a patient supine with arms extended on armboards is

armboards extended <90* with palms facing up

An imperforation or closure of a normal opening is called

atresia

A laceration or avulsion of the extensor tendon at the proximal phalanx or proximal interphalangeal (PIP) join is called a

boutonniere deformity

What is similar in patient teaching about postoperative care for both endovascular aortic aneurysm repair and abdominal aortic aneurysm resection?

c. no lifting of more than 5 lb for 6 weeks

The type of fracture that is caused by impaction of one one upon another is

compression

What is a Monteggia fracture?

fracture of the proximal third of the ulna with radial dislocation

The drug added to a local anesthetic to increase diffusion is

hyaluronidase

The earliest sign of MH is

increase in end-tidal CO2

What is the most common liver cancer?

metastatic disease

Failure of the intestines to encapsulate within the peritoneal cavity of a newborn is

omphalocele

What is the major absolute CI to lumpectomy and radiation?

pregnancy Others: prior radiation to chest, positive margins, collagen vascular dz, extensive DCIS

Newborn vomiting, free of bile, and projectile in nature, is indicative of

pyloric stenosis

What is a total (simple) mastectomy?

removal of breast and nipple without removal of axillary nodes

During cataract surgery, the irrigation/aspiration handpiece is used for

removed residual cortex

What arteries may be detached and reanastomosed to the graft during a AAA resection?

renal

Which vein proximally travels across the neck of the abdominal aorta?

renal

A stagnorn stone is one that lodges and continues to grow int he

renal calyx

Sutures placed in a wound to prevent wound evisceration are called

retention

Which is an alternate procedure to stapedectomy?

stapedotomy

Why is Heparin given in a AAA repair?

to prevent thrombosis distal to clamping

What is the MC site of breast cancer?

upper outer quadrant

During a carotid endarterectomy, the patient's heart rate drops to 30 bpm. This is likely due to the stimulation of the

vagus nerve

Which fractures is most commonly associated with a fat embolism?

femur

Clostridium perfringens is associated with with clinical condition?

gas gangrene

During a Nissen fundoplication, care should be taken when placing medial traction on the stomach to prevent avulsion of the

gastric arteries

A transplanted kidney is placed into the

iliac fossa

Which ligaments that contain fibrous tissue and non-striated muscle fibers passes in the inferior direction from the cervix on each side of the rectum to the front of the sacrum?

uterosacral

What muscle is located between the vastus lateralis and the femoral shaft?

vastus intermedius

What muscle is located directly underneath the iliotibial band?

vastus lateralis

When using the posterolateral incisional approach, which muscles are retracted anteriorly to expose the fracture site?

vastus lateralis and vastus intermedius

Name the two arteries supplying blood to the brain?

vertebral and internal carotids

Which solutions are used during a diagnostic cystoscopy?

water

At what point does the external iliac artery become the femoral artery?

at the location of the inguinal ligament

Which procedures involve abdominal, perineal, and groin dissection?

radical vulvectomy

Phagocystosis by the white blood cells is an example of the body's

second line of defense

secondary intention (granulation)

secondary intention (granulation)Allowing a wound to close on it's own.

Identify the triad of anatomic sites of abdominal wall weakness with a potential for hernias

Inguinal canal, femoral rings, umbilicus

What is the name of the breast tissue that tapers into the axilla?

"tail of Spence"

Remyelination is done by ?

*Achieved by Schwann cells wrapping around an axon, producing a segment of myelin *Much faster and more effective than regeneration

Regeneration is ? ..rate of re-growth is ?-?mm/day

*After injury, several neurofibrils emerge to form a growth cone, which may grow down the endoneurial tube to the end organ *Rate of re-growth is 1 to 4 mm/day *Not always successful, can lead to a poor prognosis

Brachial Plexus: Injury Prevention

*Arm abduction limited to 90 degrees or less *Keep head and neck in a neutral position *Use of non-sliding mattresses in place of shoulder braces

Which of the following ligaments encloses the round ligament and extends from the lateral surface of the uterus to the pelvic wall? A. Broad B. Suspensory C. Ovarian D. Infundibulopelvic

A. Broad

Deoxygenated blood is carried from the right ventricle to the lungs via which structure? A. Pulmonary arteries B. Pulmonary veins C. Coronary artery D. Vena cava

A. Pulmonary arteries

Clean contaminated wound

Operative wound in which the respiratory, alimentary, or genitourinary tract is entered under controlled conditions.

Q. In formulating a goal for a patient, which of the following is least appropriate?

A. Describe something that the nurse will do for the patient. Explanation: This is not a good characteristic of a goal for a patient. The goal should describe something that the patient will do, not something that the nurse will do.

CHAPTER 10

POSTOPERATIVE CARE, PAIN MANAGEMENT

Describe a pantaloon hernia.

A combination direct and indirect hernia

Tetralogy of Fallot

A congenital heart defect which is classically understood to involve four anatomical abnormalities of the heart that include ventricular septal defect, pulmonary valve stenosis, right ventricular hypertrophy, and transposition of the aorta (although only three of them are always present).

What is peritoneum?

A continous sheet of a single layer of flattened mesothelial cells with phagocytic properties

What is adjuvant for DCIS?

Tamoxifen postlumpectomy XRT

The specific gravity of a normal urine sample is: A. 1.015-1.025 B. 1.025-1.035 C. 1.035-1.045 D. 1.045- 1.055

A. 1.015-1.025

Traffic patterns in the surgery department are designed to: A. Aid in preventing surgical site infections B. Provide access for the public C. Promote negative air pressure in the operating rooms D. Maximize air currents in the department

A. Aid in preventing surgical site infections

Q. Of the following, which would be considered one of the five basic types of MIS (minimally invasive surgery)?

A. All of the above Explanation: The five basic types of MIS consists of: single incision laparoscopic surgery, robotic surgery, video assisted thoracoscopic surgery, multiple-incision MIS and natural orifice transluminal endoscopic surgery.

Which of the following is a narcotic antagonist? A. Naloxone hydrochloride (Narcan(R)) B. Butorphanol tartrate (Stadol(R)) C. Pentazocine hydrochloride (Talwin(R)) D. Propoxyphene hydrochloride (Darvon(R))

A. Naloxone hydrochloride (Narcan(R))

Q. What is a Gruenwald forceps be used for?

A. Gruenwald forceps would typically be used for neurological and nasal procedures. The Gruenwald bayonet forceps is considered a dressing forcep. They have a spring handle, long shanks, serrated jaws and are about 8 inches long.

Q. A smoke plume contains which of the following?

A. Harmful toxins Explanation: The laser equipment should be used according to the manufacturer's instructions. The smoke plume contains harmful toxins. The filters are considered toxic waste and should be handled according to the facility and government policy.

Q. The scrub technician receives a medication from the circulator. It is no longer in its original container, which of the following interventions should the scrub technician do next?

A. Label the medication immediately Explanation: Typically, most medications are clear in color and it would be very easy to mix medications up if they are not labeled.

The celiac artery trifurcates into the: A. Left gastric, splenic and hepatic arteries B. Internal and external iliac and femoral arteries C. Lumber, gastric, and splenic arteries D. Renal, testicular, and suprarenal arteries

A. Left gastric, splenic and hepatic arteries

The two muscles that form the pelvic diaphragm are: A. Levator ani and coccygeus B. Pubococcygeus and iliococcygeus C. Psoas and piriformis D. Puborectalis and obturator internus

A. Levator ani and coccygeus

The structure that binds the small intestine to the posterior abdominal wall is the: A. Mesentery B. Greater omentum C. Falciform ligament D. Lesser omentum

A. Mesentery

Q. Which of the following is a right of patients?

A. Right to self determination, right to fair treatment, and right to privacy and dignity. Explanation: The American Nurses Association created five basic rights for patients. These rights protect patients. They include: right to self determination, right to privacy and dignity, righty to anonymity and confidentiality, right to fair treatment, and right to protection from discomfort and harm.

Infants are at the risk of losing body heat quickly due to their: A. Thin layer of subcutaneous brown fat for insulation B. Higher body mass in comparison to and adult C. Lower blood volume D. High demand or need fr oxygen

A. Thin layer of subcutaneous brown fat for insulation

What structures are located in the anterior mediastinum? A. Thymus, ascending aorta, lymph nodes B. Upper trachea, esophagus, C. Phrenic nerves, lower trachea and bifurcation D. Descending aorta, thoracic duct, vagus nerve, azygous vein

A. Thymus, ascending aorta, lymph nodes

Which of the following is at the greatest risk for fluid volume deficit? A. Trauma victim with extensive burns of the body B. Congestive heart failure patient C. Healthy elderly patient with a fractured hip D. Athlete with compound fracture

A. Trauma victim with extensive burns of the body

The node of Lund is located in the: A. Triangle of Calot B. Inguinal canal C. Patella Bursa D. Carotid triangle of the neck

A. Triangle of Calot

Q. How many clocks are typically displayed in an operating room?

A. Two Explanation: Typically there are two clocks in an operating room. One clock is a standard clock, either analog or digital. The second clock is typically a time-elapsed clock.

Seepage of skin preparation solution under a patient may result in: A. Bruising B. Nerve damage C. Chemical Burns D. Skin abrasion

C. Chemical Burns

Which cells secrete pepsinogen? A. Islands of Langerhans B. Beta C. Chief D. Parietal

C. Chief

The main consideration in surgical needle selection is

to minimize trauma

What cavities within the brain is filled with CSF?

ventricles

In which incision could retention sutures be used?

vertical midline

Which of the following cranial nerves innervates the inferior, medial, superior rectus, inferior oblique, and levator palpebrae muscles of the eye? A. Olfactory (I) B. Optic (II) C. Oculomotor (III) D. Trochlear (IV)

C. Oculomotor (III)

Lecture VI

DVT Prphylaxis

What are the requirements of a neurotrophic ulcer/Mal Perforans

Deformity, Neuropathy, and repetitive microtrauma

FireFly Uses

Identify vessels

Define the term intrinsic factors. Intrinsic factors are the health and the body structure of the patient.

What nerve complex easily damaged by extending the arms more than 90 degrees during positioning? The brachial plexus can be injured by positioning the arms more than 90 degrees.

The gram stain differentiates

bacteria

Spinal fluid is produced by the

choroid plexus

Which of the following types of stitches incorporates the seromuscular layer and submucosa. A. Lembert B. Connell C. Halsted D. Cushing

x

The area where the common bile duct joins the pancreatic duct and enters the duodenum is called the

ampulla of vater

An enzyme active in the digestion of starches is

amylase

Meperidine hydrochlride (demerol) is a/an

analgesic

In what situation is it a good idea to use Epinephrine?

anaphylaxis

the anaerobic bacterium that causes gas gangrene is

clostridium perfringens

The drug added to a local anesthetic to increase diffusion is: A. Alpha-chymotrypsin B. Hyaluronidase C. Epinephrine D. Varidase

B. Hyaluronidase

A common complication during emergence from general anesthesia is: A. Hypotension B. Hypoxia C. Hyperthermia D. Hyperemia

B. Hypoxia

Q. Operating room attire can consist of which of the following?

A. Shoe covers and head covers. Explanation: This attire can consist of: Shoe covers Head covers Mask Eyewear Two piece pant-suite For the sterile personnel, surgical gowns and gloves would be added

In examining the male patient you place your finger and his superficial ring when all ring and ask him to cough. You then feel a sudden impulse medial to your finger. You conclude that your patient most likely has

A direct inguinal hernia

During the administration of general anesthesia, there is a loss of the eyelid reflex in stage: A. I B. II C. III D. IV

B. II

The nerve of Grassi is associated with the: A. Stomach B. Liver C. Heart D. Bladder

A. Stomach

Heat loss from the body occurs by all the following methods except: A. Radiation B. Evaporation C. Condensation D. Conduction

C. Condensation

The vein of Sappey drain the: A. Spleen B. Bladder C. Diaphragm D. Lumbar verterbrae

C. Diaphragm

When given as a preoperative medication, atropine produces: A. Analgesia B. Drowsiness C. Drying of secretions D. Lowering of anxiety

C. Drying of secretions

According to NPUAP, describe whats observed in Stages III, and IV of pressure ulcer stages.

III-Full thickness skin loss involving damage/necrosis of subQ tissue that may extent into, but not thru underlying fascia; IV-Full thickness skin loss with extensive destruction, tissue necrosis, or damage to bone, muscle, or supporting structures.

_____ is an accurate way to monitor patient taking anticoagulant

INR

FireFly Uses

Identify healthy tissue

Degeneration is of the ? body part?

If nerve is severely damaged, whether by compression or stretch, there is degeneration of the axon and, with it, the myelin

Bupivicaine and ropivacaine are look-alike sound like drugs. adverse drug reactions can occur if the clinician confuses the two. both drugs promote nerve conduction blockage analgesia when given locally but bupivacaine is cardiotoxic. When selecting a local anesthetic to fill an elastomeric pain pump with what is one contraindication that should be considered?

If the patient is hypersensitive to amide local anesthetics

In this test for DVT, sphyngomomameters are placed on calf and thigh, inflated to occlude venous return, and then defflated to measure pressure changes. When obstructed, emptying of the calf is delayed.

Impedence Plesthysmography

What direction should the scissors be angled when cutting out the tumor?

In

In this phase of healing, you will see cell infiltration, vasodialation, edema, hyperemia, phagocytosis.

Inflamm phase 0-3d

How is the sentinel lymph node found?

Inject blue dye or techetium labeled sufur colloid

When can D-Dimer exclude DVT

Low levels

What part of the femur articulates with the tibia and patella?

Medial & lateral condyles

What are the major txs of breast cancer?

Modified radical mastectomy Lumpectomy and radiation (both treatments either with or without postop chem/tamoxifen)

This suture has the least relative cofficient of friction of non-absorbable sutures

Monofilament

What is the cause of mesenteric traction syndrome?

Mesenteric traction syndrome occurs during abdominal surgery and is described as sudden tachycardia, hypotension and flush. Mesenteric traction of the small intestine may cause histamine release from mesenteric mast cells. Metabolite of prostacyclin (6-keto-prostaglandin F1 ) and thromboxane B2.

What pts are candidates for enzymatic debridement?

Non surgical pts, pts needing anticoagulation meds, if sharp debride unavail, pts recieving care at home, and those with necrotic tissue next to a tendon/bone

It is Olybuster suture, with polyglycol and polyethylene, newer synthetic monofilament, non-absorbable, minimal reaction, it is not a subject to breakdown and degradation as are nylon and silk, good stretching, good elasticity (50% of its length)

Novafil

This suture is good when you expect wound edema, wounds that swell, and this suture will return and maintains tension

Novafil

What could be described as the columns of the vertebral arch?

Pedicles

What follows a positivie sentinel node bx?

Removal of the rest of the axillary lymph nodes

Which of the following associations is incorrect?

Rete testis-opening of the ductus deferens

This has a triangular shape throughout its length and cutting edge along the outside needle curvature to prevent tissue cutout

Reverse cutting needle

This needle is used in SubQ tissue

Taper needle -------- 4-O

Non-union can be caused when bone graft isn't used to fill a bony deficit at the fracture site

True

Non-union can be caused when bone graft isn't used to fill a bony deficit at the fracture site. *True or False

True

The sciatic nerve turns into the

Tibial and common fibular nerves

What are the 3 ways to obtain a wound culture?

Tissue biopsy (punch/scalpel-most accurate), Needle aspiration, and swab cultures-Z-technique, Levines technique

Which muscle is located on the lateral aspect of the thigh?

Vastus lateralis

When using the posterolateral incisional approach, which muscles are retracted anteriorly to expose the fracture site?

Vastus lateralis and vastus intermedius

The vertebral foramen is formed by the

Vertebral body, pedicles, and laminae

What is the recommended steam quality that is necessary to achieve sterilization? A steam quality of 97% or greater is necessary for sterilization.

What are the time and temperature parameters for gravity displacement steam sterilizers? Recommended time and temperature settings for gravity displacement sterilizers are: 121-123° C (250-254° F) for 15-30 minutes, or 132-135° C (270-275° F) for 10-25 minutes

Define the term asepsis. Asepsis is the term used to describe the absence of infectious organisms.

What are the time and temperature parameters for steam flush/pressure pulse sterilizers? Recommended time and temperature settings for flash sterilizers are: 121-123° C (250-254° F) for 20 minutes, or 132-135° C (270-275° F) for 3-4 minutes.

What is assessed during the primary assessment on admission to the postanesthesia care unit (PACU)? The primary assessment focuses on the ABCs (airway, breathing, and circulation).

What are the two main risks of using opioids to manage pain? Unintended advanced sedation and respiratory depression.

List examples of regional anesthesia techniques. Regional anesthesia includes: spinal anesthesia (subarachnoid block [SAB]), epidurals, caudals, and major peripheral nerve blocks.

What depolarizing muscle relaxant is used in the OR? Succinylcholine (Anectine) is the only depolarizing agent used in the OR.

To prevent the plates, screws and rods from breaking when performing a spinal fusion, bone graft is need to fuse to

bone

Aminoglycosides are a category of antibiotics that

can cause severe ototoxicity and nephrotoxicity

Extrauterine disease of the female reproductive system may utilize the following lasers via a colposcope or laparoscope EXCEPT the: *co2 *nd:yag *candela *argon

candela

Which term refers to the transmission of fluid along a strand of suture?

capillarity

What is a result of potassium imbalance?

cardiac arrhythmias

When an open appendectomy is performed, it is necessary to identify the

cecum and ileocecal valve

A broad spectrum bactericidal antibiotic that interferes with synthesis of the cell wall and has a chemical structure similar to the penicillins is

cefazolin

What are the visceral branches of the abdominal aorta?

celiac artery, cranial mesenteric artery, renal arteries, ovarian/testicular arteries, caudal mesenteric artery

What is the nourishing coat of the eyeball that consists mainly of blood vessels?

choroid

What is the body's normal reponse to cell and tissue injury?

inflammation

The initial reaction to invasion of a foreign body or a local injury is called

inflammation.

Which phase of healing begins within minutes of injury?

inflammatory

What is the most common site of aortic cross-clamp?

infrarenal

What types of incisions are used for laparscopy?

infraumbilical

A lower oblique incision is called a/an

inguinal

Hesselbach's triangle is bounded by the rectus abdominis muscle, inferior epigastric vessels and

inguinal ligament

Assist, do not ________

inhibit

The 1/3 vs 2/3 rule allows the surgical assistant to effectively assist the surgeon without ___________________

interfering with his hands or instruments

During digital reanastomosis, which of the following items is commonly used?

operative miscroscope

Res ipsa loquitur

or the thing speaks for itself

During hemostasis, prothrombin and thromboplastin, along with calcium ions in the blood, form

thrombin

Wich hemostatic agents are most frequently used during a spinal fusion?

thrombin

What is Mondor's dz?

thrombophlebitis of superficial breast veins

What structures are located in the anterior mediastinum?

thymus, ascending aorta, lymph nodes

The patellar ligament is attached proximally to the patella and distally to which structure?

tibial tuberosity

Under which structure is the nerve trapped in carpal tunnel syndrome?

transverse carpal ligament

Which type of needle would be selected for use on skin or tendon?

cutting

What is the biggest risk factor for DVT?

Prior DVT.

Supine Positioning Complications

*Backache *Peripheral nerve injury -Ulnar -Brachial Plexus *Alopecia

Lithotomy Position: Complications

*Backache, hip pain *Peroneal, sciatic nerve injury *Hip dislocation *Hypoxemia due to V/Q mismatch

Prone: Nerve damage *? is most vulnerable

*Brachial Plexus is vunerable in this position *Inadequate support of shoulders which allows them to sag anteriorly may cause traction on the plexus *Extending arms over the head compress the plexus against the clavicle and first rib *Abduction of arms at right angles to the body can place traction on the plexus

Which of the following are reasons for keeping the OR at a relative humidity level between 50-50%? 1. reduced risk of infection 2. lower levels of patient discomfort 3. minimization of static electricity 4. depressed waste anesthesia gas

1 and 3

Which of the following techniques demonstrate a proven benefit in the prevention of postoperative wound infections? 1. administering appropriate antibiotics within 1 hour of incision time 2. maintaining suitable antibiotic coverage for 48-72 hours postoperatively 3. irrigating the wound with normal saline 4. irrigating the wound with sterile water

1 and 3 only

Q. Which of the following would not be considered a comorbid condition which would be considered important to the risk assessment in anesthesia for the patient with an endocrine disorder?

A. Cirrhosis Explanation: Cirrhosis would be a condition of hepatic disease. The endocrine disorder would include these conditions: Diabetes mellitus Thyroid disease Parathyroid disease Hypothalamic pituitary adrenal disorder

The anaerobic bacterium that causes gas gangrene is: A. Clostridium perrfringens B. Staphylococcus aureus C. Escherichia coli D. Treponema pallidum

A. Clostridium perrfringens

Q. To monitor the body tempature, which two indicators should be assessed?

A. Core and skin temperature Explanation: Routine monitoring of body temperature should include measuring the core and skin temperature. The nurse will also consider the patients age and body weight when identifying an outcome for this parameter. For instance, patients who are overweight may be less sensitive to the cold.

The tibial and common peroneal nerves are terminal branches of which of the following nerves? A. Femoral B. Sciatic C. Splenic D. Peroneal

B. Sciatic

What is observed in the UTHS san antonio stages A-D?

A-Clean; B-Non-ischemic infected; C-Ischemic; D-Ischemic infected

All of the following trade names would be correct for silk sutures except?

A. Vascufil Explanation: Vascufil would be a trade name for polybutester sutrures. The silk suture trade names would be: Mersilk, Sofsilk, Perma-hand.

The five branches of the facial nerve from superior to inferior after it enters the parotid gland is: A. Buccal, cervical, mandibular, zygomatic, temporal B. Temporal, zygomatic, buccal, mandibular, cervical C. Cervical, temporal, mandibular, zygomatic, buccal D. Mandibular, temporal, zygomatic, buccal, cervical

B. Temporal, zygomatic, buccal, mandibular, cervical

What are the goals during emergency AAA repair?

Blood loss control Attain BP Preserve myocardial Function Preserve major organ function

How is a count recorded in the record when an item is missing but not located on an X-ray? Unresolved counts are documented as follows: if the X-ray is negative, the count is recorded as incorrect, and the X-ray results are noted on the patient's intraoperative record. An incident/occurrence/event report should be initiated according to institution policy.

Define the term diffusion. Diffusion is the movement of molecules from an area of high concentration to one of low concentration

CAD is _____ associated with AAA.

HIGHLY

Use of this drug is supposed to have fewer hemorrhagic side effects, and greater efficacy than heparin. Not necessary to monitor PT, PTT on it, only the hematocrit/platelet count.

LMWH (Lovenox/enoxaparin)

The first and foremost rule of a safe & successful surgical procedure is ___________________

Principle of Traction/Countertraction

Example of "no wasted moves"

Reloading a needle so that it is always ready for re-use

What statement describes a nonrecurrent laryngeal nerve?

Variant laryngeal nerve that branches from the vagus and travels directly into the larynx

What muscle is located directly underneath the iliotibial band?

Vastus lateralis

High doses of nitrous oxide block the metabolism of which of the following? A. Vitamin B6 B. Vitamin B12 C. Vitamin C D. Vitamin D

Vitamin B12

Name the boundaries of the axilla for dissection. a. superior boundary b. posterior boundary c. lateral boundary d. medial boundary

a. axillary vein b. long thoracic nerve c. latissiumus dorsi muscle d. lateral to, deep to, or medial to pectoral minor muscle, depending on level of nodes taken

Fibroadenoma... a. What is it? b. What is clinical presentation? c. How is it diagnosed? d. Tx? e. what is its claim to fame?

a. benign tumor of breast consisting of stromal overgrowth, collagen arranged in swirls b. solid, mobile, well-circumscribed** round breast mass, usually < 40 years c. negative needle aspiration looking for fluid; u/s; core Bx d. surgical resection for large or growing lesions; small can be observed closely e. MC breast tumor in women <30yrs old

The ovarian artery is a branch of the

abdominal aorta

During the preoperative nursing assessment in the ambulatory surgery receiving unit, the nurse notices bruises and abrasions in various stages of healing on her patient's abdomen, arms, and breasts. In a quiet and respectful manner the nurse said, "The staff at our facility are concerned about your safety. We know that many things can happen in our lives that affect our mental and physical health." This technique is an appropriate method of establishing a sense of __________ with the patient while conveying caring and concern.

b. normalizing

What organism is a procaryotic?

bacteria

Adenocarcinoma of th head of the pancreas can result in jaundice, because of compression of the

bile duct

which of the following associations is incorrect

c. left border of the heart-mainly left atrium

Which is not a carppal bone?

cuboid bone

What is usually caused by an injury to the middle meningeal artery?

epidural hematoma

The inner lining of the mucosa of the g.i. tract is composed of

epithelium

Which of the following microorganisms is responsible for causing postoperative wound edge erythema and rebound tenderness? A. S. aureus B. Clostridium C. Group B streptococci D. S. epidermis

group B streptococci

Deep vein thrombosis occurs most commonly in the

left iliac vein

After a mitral valve replacement, a patient coming off bypass shows signs of cardiac ischemia. Which of the following MOST likely happened during the procedure to cause this? A. ligation of the inferior vena cava B. occlusion of the circumflex artery C. occlusion of the innominate vein D. ligation of the subclavian artery

occlusion of the circumflex artery

A barbituate that may be used for sleep in the night before surgery is

secobarbital

The mammary gland is fixed to the overlying skin and underlying pectoral fascia by fibrous bands known as

suspensory ligaments of cooper

What is the most common change in vital signs that are associated with early hypovolemic shock?

tachycardia

When a drug loses its effectiveness in a patient, the patient is said to have developed

tachyphylaxis

When closing the peritoneum or intestine, which type of needle is used?

taper

Peripheral Nerve Injury

*18% of cases in ASA CLosed case claims

Median Nerve

*Unlikely to be injured from positioning *More at risk from IV placement or injection of drugs in infiltrated IV -Adjacent to medial cubital and basilic veins in the antecubital fossa

What is observed in the UTHS-San Antonio grades 0-3?

0-Pre or post ulcer site; 1-ulcer into subQ; 2-ulcer into tendon/capsule; 3-Ulcer into bone/joint

When assisting on a pancreaticoduodenectomy, the surgical first assistant should be prepared for reconstruction of which of the following structures? 1. duct of Wirsung 2. duct of Santorini 3. common bile duct 4. common hepatic duct

1, 2, and 3 only

Taper point needles are generally used in which of the following tissue types? 1. gastrointestinal 2. tendons 3. adipose 4. vascular

1, 3, and 4

Normal adult range for sodium is

136 - 145 mEq/L

Following inflammation, what is the correct sequence of the stages of bone healing? 1. callus formation 2. cellular proliferation 3. ossification 4. remodeling

2, 1, 3, 4

Surgeon is allowed _____ of surgical field

2/3

Close the collecting system with which size suture?

3-0, RB-1

What is the ICG dose?

3.75mg

What degree of endoscope is MOST commonly used during a laparoscopic cholecystectomy?

30

How long until you can see and how long will it last?

30 seconds

How much of the total stress (weight) is delivered across the femoral fracture when using a compression plate?

30%

Normal adult male range for RBC is

4.2 - 6.1 mill/ul

What is the normal range of red blood cells in a complete blood count for an adult male?

4.6 - 6.2 million/nl

The normal pH of urine is

4.6 - 8.0

Patient Positioning

60-90 degree lateral decibutus

The normal pH of blodd is

7.3 - 7.4

If a diabetic has a lab test show a reading above __________ Hb A1C, the ulcer is unhealable.

7.5-8

_______% of patients diagnosed with pulmonary embolism also have DVT

80

What percentage of AAA's are due to atherosclerosis?

90%

After a mastectomy, at what drainage level are drains removed?

< 30 cc/day drainage or post op day 14 whichever comes first

Stage 1

<7cm, limited to kidney, partial nephrectomy

Stage 2

>7cm, limited to kidney, nephrectomy

What type of wound healing occurs when approximation of wound edges is intentionally delayed by 3 or more days after injury or surgery? Delayed primary closure or tertiary intention occurs when approximation of wound edges is intentionally delayed by 3 or more days after injury or surgery.

A peri-rectal abscess is an example of what class of wounds? A peri-rectal abscess is an example of a class IV Dirty or Infected Wound.

The vertebral canal houses the spinal cord and is formed by

A succession of vertebral foramina

Describe a facet joint.

A synovial joint formed by the articulation of two articular processes of the vertebrae

Describe cauda equina.

A term meaning horse tail & A bundle of nerve rootlets in the subarachnoid space caudal to the termination of the spinal cord

What is normal cardiac output for an adult? A. 4-8 L/minute B. 8-12 L/minute C. 12-16 L/minute D. 16-20 L/minute

A. 4-8 L/minute

A white count of 14,000 cu/mm could most likely indicate: A. An infection B. Normal range C. An anemic condition D. The count is low

A. An infection

Meperidine hydrochloride (Demerol(R)) is a/an: A. Analgesic B. Bronchial dilator C. Vasoconstrictor D. Sedative

A. Analgesic

Cephalothin sodium (Keflin(R)) is a/an: A. Antibiotic B. Steroid C. Diuretic D. Mydriatic

A. Antibiotic

Q. Postprocedural pneumonia is treated with which of the following?

A. Antibiotics Explanation: Antibiotics are used to treat postcedural pneumonia. In more severe cases, bronchoscopy may be required.

The third ventricle of the brain empties into the fourth ventricle through the: A. Aqueduct of Sylvius B. Choroid plexuses C. Foramen venosum D. Foramen spinosum

A. Aqueduct of Sylvius

Q. The scrub nurse or specialist performing the scrub role would do what?

A. Arrange the instruments on the sterile table Explanation: The scrub nurse would arrange instruments on the sterile table prior to surgery. A circulating nurse would send specimens to the lab, test a tourniquet, and provide the family a report on the patient's status.

Q. A surgical procedure that replace part of a joint with a prosthesis is commonly referred to as which of the following?

A. Artheroplasty Explanation: Artheroplasty is the replacement of part of a joint with a prosthesis. Examples of such would include a hip artheroplasty or a knee artheroplasty.

A Prolongation of the breast tissue located in the upper, outer part of the breast that passes through an opening in the axillary fascia is called the: A. Axillary tail of Spence B. Cooper's ligament C. Axillary fascia D. Sebaceous gland

A. Axillary tail of Spence

Q. Patients with congenital or valvular heart disease or prosthetic valves are susceptible to what?

A. Bacterial endocarditis Explanation: The patient who has a congenital heart defect, valvular disease or a prosthetic valve is at risk for developing bacterial endocarditis. Acute bacterial endocarditis affects the lining of the heart valves, while bacterial meningitis affects the lining of the spinal column and brain. In both cases, antibiotics are used to treat the condition

The branches of the aortic arch are: A. Brachiocephalic, left common carotid, left subclavian B. Right and left common carotid C. Brechiocephalic, right common carotid, right subclavian D. Celiac trunk, superior and inferior mesenteric

A. Brachiocephalic, left common carotid, left subclavian

The carotid body and the carotid sinus are two sensory structures; the carotid body is a: A. Chemoreceptor B. Neuroreceptor C. Peptide receptor D. Beta-adrenergic receptor

A. Chemoreceptor

Which of the following is the nourishing coat of the eyeball that consists mainly of blood vessels? A. Choroid B. Iris C. Ciliary body D. Central artery

A. Choroid

Which of the following veins drains the intracranial venous sinuses to veins on the outside of the skull? A. Emissary B. Middle cerebral C. Cerebellar Veins D. Radial Veins

A. Emissary

Q. The ability to recognize one's own emotions and the role they play in the care relationship is known as which of the following?

A. Emotional intelligence Explanation: Emotional intelligence is a key component of quality care. This is the ability to recognize one's own emotions and the role they play in the care relationship. This includes emotional reactions to patients whose beliefs, practices, and values may be very different from your own.

Dextran is used parenterally to: A. Expand blood plasma volume B. Nourish vital tissue C. Carry oxygen through the system D. Increase blood viscosity

A. Expand blood plasma volume

By definition, carcinoma in situ is a malignant neoplasm that has not: A. Extended beyond the basement membrane B. Spread throughout the parenchyma of the organ C. Invaded by direct extension into adjacent organs D. Metastasized thought the body from regional lymph nodes

A. Extended beyond the basement membrane

Which of the following arteries supply blood to the thyroid gland? A. External carotid B. Internal carotid C. Axillary D. Vertebral

A. External carotid

Q. The Swan-Ganz catheter is threaded towards the heart through all the following veins, EXCEPT?

A. External jugular Explanation: The external and internal jugular are a pair of veins that collect blood from the head, brain, face, and neck. The Swan-Ganz catheter is threaded through the internal jugular into the subclavian vein towards the heart.

Q. Hemostatic agents could include which of the following types?

A. Fibrin sealants and Active hemostats Explanation: The hemostatic agents include these types: Fibrin sealants Active hemostats Flowables

Q. Allied health professionals deliver

A. High quality services for patient care Explanation: Allied health professionals bring high quality services in the patient care field. Allied health care professionals make the health care system function properly. They typically work in teams to accomplish this.

Which of the following is a potential physiological response when lowering a patient's legs from the lithotomy position? A. Hypotension B. Hypertension C. Bradycardia D. Tachypnea

A. Hypotension

Q. There are two respiratory movements in breathing, one that draws air into the lungs and one that forces air out of the lungs. These movements would be referred to as which of the following?

A. Inspiration/expiration Explanation: Inspiration is the drawing of air into the lungs while expiration is forcing the air out of the lungs. These are the two movements involved in breathing. The control centers for each of these movements is located in the medulla.

Which of the following arteries divides into the anterior and middle cerebral arteries? A. Internal carotid B. External carotid C. Vertebral D. Basilar

A. Internal carotid

The uterine artery is a branch of the anterior trunk of the: A. Internal iliac artery B. External iliac artery C. Aorta D. Femoral Artery

A. Internal iliac artery

The uterine artery is a branch of the anterior trunk of the: A. Internal iliac artery B. External iliac artey C. Aorta D. Femoral artery

A. Internal iliac artery

The arterial blood supply to the breast is supplied from branches of the: A. Internal mammary, axillary and lateral thoracic B. Internal thoracic and subclavian C. Brachiocephalic and pulmonary D. Intercostal and phrenic

A. Internal mammary, axillary and lateral thoracic

Which of the following should not be administered to a patient who is exhibiting asystole? A. Isoproterenol B. Atropine C. Procyclidine hydrochloride D. Lidocaine

A. Isoproterenol

The ovary is suspended from the posterior lead of the broad ligament by the: A. Mesovarium B. Parietal peritoneum C. Ovarian Ligament D. Uterus

A. Mesovarium

Q. The cortex and medulla of the kidneys are made up of functional units of the kidneys known as which of the following?

A. Nephrons Explanation: The functioning units of the kidneys are the nephrons. Each kidney contains approximately 1.3 million nephrons.

Q. What agent should be used for prepping burned skin?

A. Normal saline Explanation: Normal saline solution should be used to prep burned skin. It can also be used for denuded skin

Q. Severe and sustained hypertension or hypotension can occur when the local anesthetic epinephrine is given with all of the following except for?

A. Povidone-iodine Explanation: Local anesthetics may have severe adverse reactions with other drugs. Epinephrine is one drug that may react adversely with other drugs. For instance, severe and sustained hypertension or hypotension can occur when the local anesthetic epinephrine is combined with tricyclic anti-depressants, monoamine oxidase inhibitors, and phenothiazines. Povidone-iodine is commonly used for prepping the surgical site and is not a common risk with epinephrine.

Q. What is septicemia

A. Presence of pathogens contaminating the blood Explanation: Septicemia means the blood has become septic (contaminated)--a systemic infection. This condition is due to the presence of bacteria or pathogens in the blood. It can start from infections elsewhere such as a respiratory infection or simple infected cut and become worse very quickly. Septicemia is a serious, life-threatening complication following surgery requiring prompt medical care.

Q. To promote independent lifelong behavioral changes, a perioperative nurse refers an obese patient to classes given onsite and offsite by fitness and nutrition experts in the community. This strategy fosters which behavior in patients?

A. Self-learning Explanation: The nurse's action of referring the patient connects the patient to helpful community resources. This nurse is fostering self-learning in and helping the patient to appreciate the support provided by other professionals or experts.

The coronary arteries originate at the base of the ascending aorta, also known as the: A. Sinuses of Valsalva B. Pulmonary sinuses C. Coronary sinuses D. Aortic sinus

A. Sinuses of Valsalva

The large artery that arises from the left side of the aortic arch and descends into the arm is the: A. Subclavian B. Carotid C. Brachiocephalic D. Radial

A. Subclavian

Which of the following statements describes a nonrecurrent laryngeal nerve? A. Variant laryngeal nerve that branches from the vagus and travels directly into the larynx B. Variant laryngeal nerve that branches off the superior laryngeal nerve and innervates the thyrohyoid membrane C. Variant laryngeal nerve that branches off the right recurrent laryngeal nerve and innervates the cricothyroidmuscle D. Variant laryngeal nerve that branches off the spinal branch of the accessory nerve and innervates the larynx

A. Variant laryngeal nerve that branches from the vagus and travels directly into the larynx ( Nonrecurrent laryngeal nerve is a variant nerve that branches off the vagus and travels directly into the larynx at the level of the inferior horn of the thyroid cartilage)

Name a disadvantage of ORIF with plating technique.

Additional soft tissue trauma due to open surgical intervention

What statement concerning the Bankart procedure is correct?

Anterior capsule is reattached to rim of the glenoid

This structure runs along the length of the vertebral column between the vertebral column and the abdominal aorta. It strongly supports the joints of the vertebral column and helps prevent the vertebral column from hyperextending.

Anterior longitudinal ligament

Normal adult range for potassium is: A. 1.5-3.5 mEq/L B. 3.5-5.5 mEq/L C. 5.5-7.0 mEq/L D. 7.0-8.5 mEq/L

B. 3.5-5.5 mEq/L

Which of the following are eucaryotic? A. Viruses B. Fungi C. Cyanobacteria D. Bacteria

B. Fungi

The earliest sign of MH is: A. Pyrexia B. Increase in end tidal CO2 C. Sudden cardiac arrest D. Cyanotic skin and finger tips

B. Increase in end tidal CO2

Which organ is completely retroperitoneal? A. Liver B. Kidney C. Colon D. Duodenum

B. Kidney

Which of the following is a principal muscle of the pelvic floor? A. Satorius B. Levator ani C. Internal oblique D. Rectus abdominis

B. Levator ani

An organelle that contains digestive juices is the: A. Nucleus B. Lysosome C. Ribosome D. Golgi complex

B. Lysosome

Which of these local anesthetics has a longer duration? A. Xylocaine B. Marcaine (R) C. Novocaine D. Cocaine

B. Marcaine (R)

The series of arcades along the mesenteric border throughout the length of the colon is called the: A. Arc of Riolan B. Marginal artery of Drummond C. Pancreaticoduodenal arch D. Paracolic arcade

B. Marginal artery of Drummond

The arterial blood supply to the nose is from branches of the internal and external: A. Carotids B. Maxillary C. Ethmoid D. Facial

B. Maxillary

The gelatinous substance within and intervertebral disc is called the: A. Spinous process B. Nucleus pulposus C. Annulus fibrosus D. Intervertebral foramen

B. Nucleus pulposus

Which of the following the the name for the small pancreatic duct? A. Wirsung B. Santorini C. Accessory D. Cystic

B. Santorini

Lymph channels run parallel to which structures? A. Nerves B. Veins C. Arteries D. Ligaments

B. Veins

Warm Ischemia

Bulldog clamps on renal artery and renal vein (30 minute limit)

The three protective tissue layers called the meninges lie in the following order, starting with the most superficial: A. Arachnoid, dura mater, pia mater B. Pia mater, dura mater, arachnoid C. Dura mater, arachnoid, pia mater D. Arachnoid, pia mater, dura mater

C. Dura mater, arachnoid, pia mater

The small ducts that drain bile directly into the gallbladder are called: A. Hepatopancreatic B. Luschka C. Hepatic D. Portal

C. Hepatic

Chvostek's signs can indicate: A. Hyponatremia B. Hypomagnesemia C. Hypocalcemia D. Hypoglycemia

C. Hypocalcemia

What is the principal foal of suctioning blood and body fluids from the wound? A. Dissection of tissue B. Retract tissue C. Improve wound visibility D. Measure blood loss

C. Improve wound visibility

Hesselbach's triangle is bounded by the rectus abdominis muscle, inferior epigasric vessels and: A. External oblique muscle B. Falciform Ligament C. Inguinal ligament D. Linea Alba

C. Inguinal ligament

Which vein is shared by the pancreas and liver? A. Superior vena cava B. Subclavian C. Portal D. Renal

C. Portal

The reversal agent for heparin induced anti-coagulation is: A. Sodium warfarin B. Propofol C. Protamine sulfate D. Narcan (R)

C. Protamine sulfate

The principal goal of the surgical assistant during the surgical procedure is to: A. Dissect tissue B. Apply dressings C. Provide exposure D. Position the patient

C. Provide exposure

The frozen section examination provides the surgeon with a: A. Guideline for antibiotic therapy B. Diagnosis for tuberculosis C. Quick preliminary diagnosis D. Microscopic examination of smear and cultures

C. Quick preliminary diagnosis

General and regional anesthesia contribute to body heat loss by causing: A. Vasoconstriction B. Vasospasm C. Vasodilation D. Vascularization

C. Vasodilation

Body temperature is regulated by the: A. Pons B. Cerebellum C. Midbrain D. Hypothalamus

D. Hypothalamus

Describe the lateral position. In the lateral position, the patient lies on his or her nonoperative side.

Define the term extrinsic factors. Extrinsic factors are the physical forces and conditions to which the patient is subjected during surgery.

Define the term continuous suture. A continuous suture consists of a series of stitches, of which only the first and last are tied.

Define the term interrupted suture. An interrupted suture is inserted into tissues or vessels in such a way that each stitch is placed and tied individually.

These are the largest cause of non traumatic amputations in the US

Diabetic foot ulcers; 86k performed last yr. 100% preventable

Bleeding vessel can be stopped by

Electrocautery Pressure Tied off Glue applied

Patient Positioning

Flex table at midpoint to increase working space, 15-20 degree

Which of the following is incorrect pertaining to the inguinal canal

Its roof is formed by the inguinal ligament

List the possible consequences of untreated postoperative pain. Untreated pain may be associated with decreased thoracic movement, increased splinting, reduced lung compliance and volume, decreased mobility, increased risk of thromboembolism, exaggerated catecholamine response, increased risk for myocardial ischemia, impaired immune system, and delayed return of bowel and gastric function

List the risk factors for postoperative nausea and vomiting (PONV). Risk factors for PONV include: Female gender, nonsmoker, history of PONV or motion sickness, use of volatile anesthetics, use of nitrous oxide, postoperative use of opioids, duration of surgery, and type of surgery.

Mirroring the surgeon refers to

Maintaining a position exactly opposite to the surgeon in the surgical field

What are the assiciated risk factors of DVT?

Malignancy, Trauma, CHF, Central venous catheter, pacemaker, extremity paresis, superficial vein thrombosis, oral conraceptives

A flap of tissue transferred from one site to another with its vascular supply intact

Skin Flaps

Dermabond

Skin glue

What stage of breast cancer? Tumor < or equal to 2 cm in diameter without metastases, no nodes

Stage I

What breast carcinomas are candidates for lumpectomy and radiation?

Stage I and II, tumors <5 cm

What are common options for breast reconstruction? (3)

TRAM flap (transverse rectus abdominis myocutaneous flap) implant latissimus dorsi flap

How can the long thoracic and thoracodorsal nerves be identified during an axillary dissection?

The nerves can be stimulated with forceps which results in contraction of the latissimus dorsi (thoracodorsal) and anterior serratus (LTN)

Which surgical approach for an open AAA repair gives the best control of all blood vessels?

Transperitoneal (midline)

What are the two common surgical approaches for an open AAA repair?

Transperitoneal (midline) Retroperitoneal (flank)

Which type of endoleak is when blood flows into the aneurysm sac due to incomplete seal or ineffective seal at the end of the graft?

Type I endoleak

What type of endoleak is when blood flows into the aneurysm sac due to inadequate or ineffective sealing of overlapping graft joints or rupture of the graft fabric?

Type III endoleak

What is important about Morison's pouch?

When lying flat it is the lowest part of the peritoneal cavity and so this is where fluid will accumulate.

This tissue contains fat cells.

Yellow Marrow

a. What is the incidience of BRCA in men? b. What is the average age at diagnosis? c. What are the s/sx? d. Most common presentation? e. How is it diagnosed?

a. <1% b. 65yrs c. Breast mass (most painless), breast skin changes, nipple discharge d. painless breast mass e. biopsy and mammogram

The cranial meninges are composed of

a. Dura mater b. Pia mater c. Arachnoid mater

Herbal, orthomolecular, individual biologic therapies and special dietary treatments are encompassed in biologically based therapies. Which of the biologically based therapies listed below are classified as therapeutic nutrition?

a. The diets of Drs. Atkins, Pritikin, and Weil

tui nyugen 32 year old hysteroscopy patient. What patient safety info relevant to hysteroscopy should be given to the receiving relief nurse before break?

a. a review of the amount of hyskon infused and fluid received back b. a reminder to monitor for discrepancies over 500ml

An intramuscular injection into the buttocks is most safely done:

a. into the superolateral quadrant.

The right coronary artery typically

a. supplies both the AV and SA nodes.

Cephalothin sodium (Keflin) is a/an

antibiotic

Cardioplegia induces cardiac

arrest

To accommodate near and distant focus, the lens changes shape and focus by relaxation and tightening of the zonular fibers. What physiologic change of the normal aging process is typically corrected with bifocals?

b. Presbyopia

the most likely reason for avascular necrosis in an 80 year old woman following a hip fracture is that the

b. fracture split the neck of the femur longitudinally

The superior mesenteric artery:_____

b. is accompanied by the superior mesenteric vein on its right.

study comparing tep and tapp techniques.

b. mesh infection rate was low with both techniques d. there was no apparent difference in teh rate of hernia recurrence

The science of plastic and reconstructive surgery, which means to mold or create form and shape, improves appearance and body image through an understanding of:

b. the anatomy and biology of tissue

The pterion

c. is part of the temporal bone.

The hepatorenal recess (Morison pouch):

c. receives infected fluids draining from the omental bursa in the supine position

this tissue is concerned with the formation of red blood cells

c. red marrow

A rounded protuberance found at a point of articulation with another bone is called a

condyle

Which of the following is incorrect pertaining to the gall bladder?

d. During cholecystectomy surgeons typically ligate the left hepatic artery

What is a common complication when transporting a post-anesthesia patient?

decreased cardiac output

An elderly patient who is one week postabdominal surgery complains of a "popping" sensation associated with coughing or straining. the most likely cause is

dehiscence

Meningiomas are benign tumors that originate in the

dura

A tumor that involves the junction of the right and left hepatic ducts?

klatskin

Insufficient blood flow to the heart muscle may lead to

myocardial infarction

When is best time for SBEs?

one week after menstrual period

Measures to treat atelectasis include

pain control and early ambulation

In severe aortic stenosis all of the following are indications for valve replacement EXCEPT: *palpitations *congestive heart failure *angina *syncope

palpitations

During dissection of the internal mammary artery (IMA), what solution should be used to dilate the artery and reduce muscle spasm?

papaverine

What is applied topically to arteries for prevention of vasoconstriction?

papaverine

Postop hernias are common after which incisions?

paramedian

The most reliable tendon graft for primary reconstruction of the anteior cruciate ligament is

patellar

Which of the following tendon autografts is MOST typically utilized when performing ACL reconstruction?

patellar

The terminal arterial branches of the aorta are the

right and left common iliacs

What does the cranial mesenteric artery supply?

supplies major part of the small and large intestine

What study is used to determine Zenker's diverticulum?

barium swallow

Identify the soft osseous material filling the cavities of the cancellous bone.

Marrow

A vesicourethral abdominal suspension is aka

Marshall-Marchetti

Supine: UE Abducted *degrees of abduction should be ? *turning head side to side could damage the ?

*90° or less (minimizes brachial plexus injury) *Padded arm boards *Turning head to side may cause stretching/compression of contralateral brachial plexus below the clavicle

Neurapraxia is ? Reversible in ?

*A mild degree of insult that results in conduction failure *Reversible (a few days to a few weeks)

Physiologic Changes in Prone Position: Cardiovascular

*Abdominal pressure impedes venous return if not properly positioned

Musculoskeletal assessment of the patient with a mid-shaft fracture of the femur includes

*Absent or decreased pulses *capillary refill *generalized edema *ecchymosis

Lithotomy: Positioning *hips should align with ?

*Arms either tucked at sides or abducted on arm boards *Hips should be at the break in the table which will allow the foot of the table to be dropped *Legs in stirrups -Allen -Suspension *Watch hands with raising the foot of the table

Prone: UE Positioning *Arms are in the ? position? *Elbows are ? *Shoulders flexed less than ?

*Arms extended on arm boards next to patient's head ("swimmer's position") and hands are pronated *Elbows flexed and padded (to prevent ulnar nerve compression) *Shoulders less than 90 degrees *Arms may be tucked at patient's sides

Median Nerve: Injury Prevention

*Avoid extreme wrist dorsiflexion *Avoid IV infusions at antecubital fossa

Lateral Decubitus: Torso Positioning *axillary roll should never be in the ?

*Axillary roll -Place slightly caudad to the axilla; never place in the axilla *Stabilized to prevent patient from rolling onto back or front *Deflatable beanbag -Must not compress axilla

Lateral Position Complications

*Compression and stretching of nerves Ulnar, peroneal, Brachial Plexus *Damage to dependent eye *Hypotension

Brachial Plexus: Mechanisms of Injury *Most commonly due to ? *Worse when arms are abducted >?

*Stretching *Arm abducted > 90° (caudad pressure in the axilla from head of humerus) -Head turned opposite direction -Patient shifts caudally and arm is secured -Arm is externally rotated -Median sternotomy and retraction -Stretching around the 1st rib and damage from 1st rib fractures -May be worse with asymmetric retraction of sternum

Brachial Plexus: *The 2 Mechanisms of Injury are ? ?

*Stretching *Compression

What are three anesthetic goals during AAA repair?

--Minimize myocardial, renal , pulmonary, CNS, other end-organ dysfunction. --Emphasis is on the heart. --Rapid identification of ischemia ( various modes)

Which of the following should be considered when positioning a patient? 1. Patency of the airway is the prime concern when positioning a patient 2. Pressue injuries are most common after surgical procedures lasting one hour or longer. 3. An anesthetized patient with muscle paralysis is at increased risk for muscle strain 4. The most common sites of peripheral nerve injury are the divisions of the brachial plexus and the ulnar, radial, peroneal, and facial nerves

1, 2, and 4 only

Which of the following statements is true concerning hypophysectomy? 1. Replacement of the adrenal steroid therapy is usually required. 2. Diabetes may result 3. Growth will be increased. 4. Cessation of menses and infertility may result.

1, 2, and 4 only

Tetralogy of Fallot includes which of the following defects? 1. ventricular septal defect 2. aortic stenosis 3. pulmonary valve stenosis 4. right ventricular hypertrophy

1, 3, and 4

One kilogram is equal to

1,000 grams

What is the relative risk of developing breast cancer from hormone replacement therapy?

1-1.5

Workup for breast mass?

1. clinical breast exam 2. mammogram or breast U/S 3. FNA, core Bx, or open Bx

When do you proceed to open biopsy for a breast cyst?

1. in the case of a second cyst recurrence 2. bloody fluid in cyst 3. palpable mass after aspiration

What is the treatment of BRCA in men?

1. mastectomy 2. sentinel LN dissection of clinically negative axilla 3. axillary dissection if clinically positive axillary LN

The specific gravity of a normal urine sample is

1.015 - 1.025

Assistant is allowed ______ of surgical field

1/3

Sutures fromt he ear may be removed after

10 - 14 days

When drawing a blood sample, a safe time lapse between blood drawing and analysis is

10 minutes

When establishing the pneumoperitoneum for laparoscopic procedures, the CO2 gas pressure is kept between

14 - 16 mmHg

What makes a breast tumor high risk?

> 2 cm lymphatic / vascular invasion nuclear grade (high) S phase (high) ER negative

Define the term superficial surgical site infection (SSI). A superficial SSI is one involving only the skin and subcutaneous tissue of the incision and at least one of the following: purulent drainage from the superficial incision, organisms isolated from a culture from the superficial incision, or signs and symptoms of infection.

A gunshot wound is an example of what class of wounds? A gunshot wound is an example of a class III Contaminated Wound.

Which of the following is an example of an isotonic IV solution? A. 0.9% sodium chloride B. 5% dextrose in water C. Lactated ringer's solution D. Hartmann's solution

A. 0.9% sodium chloride

Q. Piaget's formal operations stage begins when a person reaches the age of?

A. 11 Explanation: Piaget's formal operations stage begins at age 11 years and continues through adulthood. The child can think logically about abstract positions and test hypotheses; can further develop logical thinking and reasoning; and achieves cognitive maturity.

Which ligament connects the thyroid to the trachea? A. Berry B. Recurrent C. Lateral D. Posterior

A. Berry

Fissure of Sylvius separates the: A. Frontal and temporal lobe B. Parietal and occipital lobe C. Occipital and temporal lobe D. Temporal lobe and olfactory area

A. Frontal and temporal lobe

Where is Hartmann's pouch located? A. Gallbladder infundibulum B. Apex of the gallbladder C. Stomach D. Liver

A. Gallbladder infundibulum

Clostridium perfringens is associated with which clinical condition? A. Gas gangrene B. Toxic shock syndrome C. Botulism D. Anthrax

A. Gas gangrene

The purpose of hemostasis is to: A. Prevent blood loss B. Allow for oxygen diffusion C. Increase blood flow D. Promote active bleeding

A. Prevent blood loss

The arterial supply to the intestinal tract, with the exception of the stomach, duodenum, and distal rectum, is derived from the: A. Superior and inferior mesenteric arteries B. Cystic artery C. Gastroduodenal artery D. Hepatic artery

A. Superior and inferior mesenteric arteries

Q. A patient outcome must be measurable and observable. All of the following are appropriate parts of a measurable outcome EXCEPT:

A. evaluative language Explanation: Evaluative language is not a part of writing a patient outcome that is measurable and observable. The parts of a measurable and observable outcome include: subject, verb, conditions, performance criteria, and target time.

Q. In which of the following cases would a nurse be allowed to disclose confidential information?

A. if the patient is a danger to himself or others. Explanation: Confidential information can be disclosed under certain circumstances. For example, the nurse must disclose such information when authorized by a patient (but only to the extent authorized), when a patient is a danger to himself or others, or when required by law for a reportable communicable disease or suspected abuse.

Q. Which of the following is NOT a clinical sign of malnutrition?

A. increased skin turgor Explanation: Clinical signs of malnutrition appear in areas of rapid turnover of epithelial and mucosal cells. The signs include dry, flaky, or discolored skin; brittle hair and nails; bleeding gums and swollen tongue; muscle wasting; decreased skin turgor; ascites; and peripheral edema.

Q. A patient teaching plan would include all of the following EXCEPT:

A. patient's willingness to learn Explanation: This would not be part of the actual teaching plan, although the patient's willingness to learn is essential to the success of the education. All of the other choices are part of the actual teaching plan.

Q. Perioperative nursing includes three phases of the surgical experience. In which phase would the nurse assess the cardiovascular, pulmonary, GI, and neurologic systems for baseline functioning?

A. preoperative phase Explanation: Perioperative nursing includes three phases of the surgical experience: preoperative, intraoperative, and postoperative. Physical assessment such as assessing the cardiovascular, pulmonary, GI, and neurologic systems for baseline functioning is part of the preoperative phase.

What part of the hip are the hemispherical reamers used on?

Acetabulum

This an absolute contraindicated in the treatment with Heparin

Active bleeding, severe bleeding diasthesis or platelet count less than 20,000, neurosurgery, ocular surgery, intracranial bleeding in the 10 days

In risk evaluation of DVT, what risk factors warrant 2 points each?

Age 61-70, hx of idiopathic/unprovoked DVT, major surgery, malignancy, multiple trauma, spinal cord injury w/ paralysis.

The mesh-plug open hernia repair technique is indicated for which type of hernia?

All--Femoral, indirect inguinal, direct inguinal hernias

What are the boundaries of the lesser sac?

Anterior wall - posterior layer of lesser omentum, peritoneum over the posterior wall of stomach, posterior of the anterior two layers of the greater omentum Posterior wall - The anterior of the two posterior layers of the Greater omentum, which is adherant (but separable) to the transverse mesocolon. Above attachment of mesocolon to upper border of pancreas, peritoneum that covers upper neck and body of pancreas, upper part of left kidney, left suprarenal gland, abdominal aorta, ceoliac artery and diaphragm Extends to below the transverse colon L244

What is the path from aorta to anterior spinal artery?

Aorta --> posterior intercostal artery --> Artery of Adamkiewicz --> anterior spinal artery

With what type of ulcer is sharp debridement contraindicated?

Arterail insufficiency ulcer, remember, no blood going there, no healing!

This is also known as an ischemic ulcer

Arterial insufficiency ulcer

What arteries are in the thoraco-lumbar area of spinal cord perfusion?

Artery of Adamkiewicz 75% of time: present in T8-T12 10-15% of time: present in L1-L2

How do you treat mesenteric traction syndrome?

Ask surgeon to "please stop tracting the mesentery"

How do you proceed if the mass appears to be a cyst?

Aspirate it with a needle

In the therapy for confirmed DVT/PE, how is heparin given?

Asses PT and PTT; IV infusion of 5-10k units heparin, after initial bolus, decrease infusion rate to 800-1500 units/hr; maintaining PT at 2-2.5x baseline

What is the treatment in a confirmed DVT or PE?

Assessment of PT and aPTT/IV infusion 5000-10000 units of heparin/After initial bolus, heparin is infused at rate of of 800 to 1500 units per hours. Maintaining 2 to 2.5 baseline level

What is the dividing line between the supracolic and infracolic compartments?

Attachment of transverse mesocolon to posterior abdominal wall.

At what point does the external illiac artery become the femoral artery? A. Within the inguinal canal B. At the location of the inguinal canal C. At the location of the ilioinguinal nerve D. Within the triangle of doom

B. At the location of the inguinal canal

An imperforation or closure of a normal opening is called: A. Hypertrophy B. Atresia C. Stenosis D. Atrophy

B. Atresia

The gram stain differenciates: A. Viruses B. Bacteria C. Helminths D. Prions

B. Bacteria

Intraoperative fluid and electrolyte imbalance are affected by: A. Stress and anxiety, especially in children B. Blood loss, fluid shifts and replacement C. Adequate oxygenation and saturation D. Heart muscle contactility and function

B. Blood loss, fluid shifts and replacement

A vaginal response would be: A. Tachycardia B. Bradycardia C. Hypothermia D. Pyrexia

B. Bradycardia

The cartilage distal to the thyroid cartilage and the proximal to the thyroid gland is the: A. Articular B. Cricoid C. Tracheal D. Hyoid

B. Cricoid

Which of the following nerves travels along the anterior-lateral chest all and innervates the serratus anterior muscle? A. Axillary B. Long thoracic C. Thoracodorsal D. Musculotaneous

B. Long thoracic nerve

According to the Mallampati classification of preoperatively assessing airway adequacy for intubation purposes, what is Class III? A. Soft palate, anterior and posterior tonsillar pillars and uvula are visualized B. Only the base of the uvula can be visualized C. Tonsillar pillars cannot be visualized D. Uvula cannot be visualized at all

B. Only the base of the uvula can be visualized

One kilogram is equal to: A. 10 grams B. 100 grams C. 1,000 grams D, 10,000 grams

C. 1,000 grams

The recommended minimum number of air exchanges in the O.R. is: A. 5 times and hour with three exchanges of fresh air B. 10 times and hour with four exchanges of fresh air C. 15 times and hour with three exchanges of fresh air D. 10 times and hour with four exchanges of fresh air

C. 15 times and hour with three exchanges of fresh air

Identify the stage of bone healing that is bypassed when compression plate fixation is employed.

Callous

This the gold standard for invasive testing

Contrast venography

What are the indications for skin flaps

Coverage of an area with poor vascularity

What drug paralyzes the sphincter muscle of the iris?

Cycloplegics

Low levels of this test will rule out DVT.

D-dimer test; if have moderate to high risk, this WILL NOT rule DVT out alone.

This test that serves to exclude DVT and non specific, cheaper than Doppler and should never be used alone to diagnose DVT

D-dimer,

Normal adult range for sodium is: A. 102-113 mEq/L B. 113-124 mEq/L C. 124-135 mEq/L D. 136-145 mEq/L

D. 136-145 mEq/L

The ovarian artery is a branch of the: A. Pelvic artery B. Cystic artery C. Rectal artery D. Abdominal aorta

D. Abdominal aorta

Epinephrine should be used in which of the following situations: A. Glaucoma B. Tachycardia C. Hypertension D. Anaphylaxis

D. Anaphylaxis

Hemolytic reactions caused by ABO incompatibility are most often caused by: A. Blood transfusions lasting more than four hours B. Human clerical error and improper identification C. Blood contaminated with pyrogenic bacteria D. Antibodies to the donor foreign proteins

D. Antibodies to the donor foreign proteins

The correct arm placement when positioning a patient supine with arms extended on armboards is: A. Armboards extended >90 deg. with palms facing up B. Armboards extended <90 deg. with palms facing down C. Armboards extended >90 deg. with palms facing down D. Armboards extended <90 deg. with palms facing up

D. Armboards extended <90 deg. with palms facing up

Cricoid pressure is applied to prevent: A. Hemorrhage B. Infection C. Bronchospasm D. Aspiration

D. Aspiration

A solution used for eye irrigation is: A. Phenylephrine HCL B. Normal saline C. Alpha chymotrypsin D. Balanced salt solution

D. Balanced salt solution

Which of the following are considered biological hazards in the operating room? A. Anesthetic gases and toxic fumes B. Irradiation and cleaning agents C. Cytotoxic drugs and gases D. Body fluids and laser plume

D. Body fluids and laser plume

A rounded protuberance found at a point of articulation with another bone is called a: A. Crest B. Trochlea C. Tubercle D. Condyle

D. Condyle

A drug used to reverse hypotension is: A. Isoproterenol hydrochloride (Isuprel(R)) B. Propranolol hydrochloride (Inderal(R)) C. Procainamide hydrochloride (Pronestyl(R)) D. Levarterenol (Levophed(R))

D. Levarterenol (Levophed(R))

In the event that a child needs an emergency procedure and the parents cannot be located to sign the permission: A. No permission is necessary B. Permission is signed by a court of law C. Permission is signed by the surgeon D. Permission is signed by two consulting physicians

D. Permission is signed by two consulting physicians

Ulnar nerve damage could result from: A. Improper placement of legs in stirrups B. External rotation of the shoulder C. Hyperextension of the neck D. Placement of elbow on unpadded table

D. Placement of elbow on unpadded table

Leg tenderness, swelling of the whole leg > 3cm difference in circumference between calves, pitting edema, collateral superficical veins, these are more predictive of _______?

DVT

Which disorders are characteriezed by compression of the abductor pollicis longus and extensor pollicis brevis tendon near the radial styloid process?

De Quervain's

This is the removal of necrotic and fibrotic tissue from a wound base.

Debridement

What are some interventions the anesthetist may perform immediately prior to or during the cross-clamp of the aorta?

Deepen anesthetic level Maintain euvolemia to hypovolemia Use vasodilators (Nipride, Fenoldopam) PEEP of 15 cm (per Barash)

Define the term moderate sedation/analgesia (conscious sedation/analgesia). Moderate sedation/analgesia (conscious sedation/analgesia) is a drug-induced depression of consciousness achieved by the administration of sedatives or the combination of sedatives and analgesic medications, most often administered intravenously, and titrated to achieve a desired effect.

Define the ASA P1 status. An ASA P1 status is defined as a normal healthy patient.

What does necrotic tissue delay and provide a medium for?

Delays healing, and provides a medium for bacterial growth

Name the nerve that is responsible for postoperative shoulder pain if irritated during laparoscopic procedures. The phrenic nerve may be injured or irritated during laparoscopy.

Describe the advantages of minimally invasive surgery. Advantages of minimally invasive surgery include: ambulatory or short hospital stay, short postoperative recuperation, reduced postoperative pain, and earlier return to normal lifestyle.

Describe the mechanism of action responsible for healing via hyperbaric oxygen (HBO) therapy. HBO increases the capacity of blood to carry oxygen to the tissues. The increased oxygenation assists in cellular restoration, is directly bactericidal to anaerobes, and improves leukocyte migration and phagocytosis to the wound bed.

Describe the difference between dehiscence and evisceration. Dehiscence is separation of layers of a surgical wound. Evisceration is extrusion of internal organs or viscera through a gaping wound.

Define the ASA P5 status. An ASA P5 status is defined as a moribund patient who is not expected to survive 24 hours with or without an operation.

Describe the emergence phase of anesthesia. Emergence begins as the patient begins to ''emerge'' from anesthesia and usually ends when the patient is ready to leave the operating room (OR).

Infection equation

Dose X Virulence/Host resistance

What are the signs and symptoms of mesenteric traction syndrome?

Facial flushing, reduced SVR, reduced MAP, tachycardia, increased cardiac index

This one is less likely to cause thrombocytopenia

Fondaparinux

The small ducts that drain bile directly into the gallbladder are called

Hepatic

Name several effects seen after the aorta cross-clamp is removed.

Hypotension Metabolite Washout/Vasoactive agents Increased pulmonary vascular resistance Microaggregates/neutrophil/thromboxane A2 Hypovolemia

Using the Knighton classification of neurotrophic ulcers, what is observed in stages I-VI

I-Into dermis; II-Into SubQ tissue; III-Into Ligament, capsule, joint, bone; IV-Infected grade III; V-Necrotic tissue in wound; VI-Gangrene foot. .

According to NPUAP, describe what is observed in Stages I and II of pressure ulcers

I-Nonblanchable erythema of intact skin; II-Partial thickness skin loss involving the epidermis and/or dermis

When performing an open AAA, which artery or arteries are cross-clamped prior to cross-clamping the aorta?

Iliac

This ligamentous attachment from the anterior inferior iliac spine and acetabular rim proximally and the intertrochanteric line distally is said to be the strongest ligament in the body:

Iliofemoral ligament

Select the triad of the boundaries of the Hesselbach triangle.

Inguinal ligament, rectus abdominis muscle, deep epigastric vessels.

What statement is correct concerning the handling of renal calculi?

Kept dry to avoid interference with chemical analysis

Cardiac arrhythmias of ventricular origin can be treated with: A. Sodium bicarbonate B. Atropine C. Lidocaine D. Furosemide (Lasix (R))

Lidocaine

Describe the right infracolic space

Lies to the right of the root of the mestery apex is below - at IC junction right side - ascending colon and right paracolic gutter base - attachment of the mesocolon

Identify the longitudinal ligament that connects the tips of all spinous processes together?

Ligamentum Flavum

What is the largest of the moveable vertebrae?

Lumbar

Name the various folds of peritoneum that connect viscera to the abdominal wall

Mesentery - small bowel - transverse colon, sigmoid mesocolon, mesoappendix Lesser Omentum - connects stomach to liver Greater Omentum - hangs down from greater curvature of the stomach Various ligaments attached to liver and spleen Broad ligament - stretches out on either side of uterus

So is 4 bigger or smaller than OOOO (TQ)

OOOO is smaller than 4

This type of ulcer is the result of a transient period of prolonged pressure found over prominent anatomical areas, and complicated in pts w/ PVD.

Pressure/decubitus ulcer

first intention

Primary closure of a wound.

What instrument is used for the Ultrasound?

ProGrasp

This suture has an extreme smoothness compromises knot security, hence extra throws are required, when swelling occurs, this suture will stretch to accomodate the wound (also known as plasticity), and when swelling receeds, the suture will remain loose, it is widely used in podiatry

Prolene

This suture is recommended where minimal reaction is desired (contaminated and infected wounds)

Prolene

In this phase of wound healing you will see Fibroblast migration, angiogenesis, collagen laydown, GF release, Granulation, contraction, and epithelialization.

Proliferation phase

The primary factor when selecting the surgical needle is: A. The metal's ability to resist corrosion B. Preventing microbial growth C. Reducing as much as possible the trauma to the tissue. D. Ease of loading the needle on the needle holder.

Reducing as much as possible the trauma to the tissue

What is the tx for DCIS <1 cm? Tumor > 1 cm?

Remove with 1 cm margins +/- XRT Perform lumpectomy with 1 cm margins and radiation or total mastectomy (no axillary dissection)

What is Morison's pouch?

Right subhepatic space hepatorenal pouch is located between the liver and kidney Anteriorly it is bounded by the liver, Laterally by the diaphragm Posteriorly by the kidney Inferiorly, is continuous with the right paracolic gutter (which then continues down into the pelvis) Medially, is continous with the lesser sac through the epiploic foramen

Which of the following organisms normally found on the skin can cause wound infections?

Staphylococcus aureus

Excisional or total, incisional or partial, and cross hatching are the forms of what type of debridement?

Sharp debridement

What are the 5 mechanisms of debridement>

Sharp, mechanical, autolytic, enzymatic, and biotherapy

What is sliding clip renorrhaphy?

Sliding clip used with Hem-o-lock clip on 2-0 CT vicryl. Reduce suture from pulling through parenchyma.

Which fractures are caused by falling on the back of the hand with the wrist flexed, aka reverse Colles' fracture?

Smith's

This is continuous or interrupted sutures placed in the dermis beneath the epithelial layer, it absorbable or non-absorbable sutures used

Subcuticular sutures- Best for intradermal, produces great looking scar

The only depolarizing muscle relaxant in clinical use is

Succinylcholine (Anectine)

These are the clinical findings leading to a suspicion of DVT?

Sudden swelling in one extremity, Deep aching pain of tenderness in calf/thigh, variable and non-specific

What are the clinical physical findings of DVT?

Sudden swelling in one extremity, Deep aching pain of tenderness in calf/thigh, variable and non-specific

Which two vertebrae mark the span of the abdominal aorta?

T12 (diaphragmatic hiatus) L4 (common iliac bifurcation)

What staging system is used for breast CA? What stage is distant metastases? What stages are candidates for lumpectomy + radiation?

TMN (tumor, metastases, nodes) IV I, II (tumors < 5 cm)

What is the lateral umbilical fold and what do they contain?

The Lateral Umbilical fold passes from the medial border of the deep ring to the arcuate line. They do not reach the umbilicus. They contain the inferior epigastric vessels.

Which two similar hernia repair approaches reestablish the integrity of the transversalis fascia and simultaneously reestablish and strengthen the posterior inguinal floor by sewing the transversalis fascia to the Poupart ligament?

The McVay and the Cooper ligament repair

What defines the size of the suture material (TQ)

The O Size 5-O or OOOOO is smaller than size 4-O or OOOO The smaller the size the less tensile strength

The transabdominal preperitoneal patch hernia repair and the totally extraperitoneal patch repair differ in the manner in which access is gained to the preperitoneal space. Which of the two provides access to the preperitoneal space without entering the peritoneum?

The totally extraperitoneal patch (TEP) repair technique

A disadvantage of an anterolateral incisional approach to the femoral shaft is

The vastus lateralis is incised requiring longer rehab; increased pain and bleeding

How long is the maturation/remodeling phase?

Up to 1.5y

Which organism is most frequently implicated in surgical site infections? Staphylococcus aureus is most frequently implicated in surgical site infections.

What are the methods of air removal used by sterilizer units? Gravity displacement and dynamic air removal are methods of air evacuation from a sterilization chamber.

Define the term bioburden. Bioburden is the amount of microbial load and organic debris on an item before sterilization

What is the preferred sterilization for moisture- and heat-sensitive items? Low-temperature hydrogen peroxide gas plasma sterilization is used for moisture- and/or heat-sensitive items

Define the term virulence. Virulence describes the potency of a pathogen measured in the numbers required to kill the host

What non-spore-forming, nonmotile, aerobic bacillus is responsible for tuberculosis? Mycobacterium tuberculosis is responsible for tuberculosis

What is ethylene oxide (EO)? EO is an alkylating agent used to sterilize heat-labile and moisture-sensitive items.

What pathologic organism often seen in critical care and burn units? Pseudomonas aeruginosa is often found in critical care and burn units.

List the characteristics of Stage II pressure ulcer. Stage II is characterized by partial-thickness loss of dermis presenting as a shallow open ulcer with a red-pink wound bed without slough.

What risks are commonly associated with Fowler position? Venous thromboembolism (VTE) and air embolism are risks associated with the Fowler position.

List the five major components of a laser. The laser components are the: (1) laser head, (2) excitation source, (3) ancillary components, (4) control panel, and (5) delivery system.

What term is used to describe knot tying inside of the body during laparoscopic procedures? Knot tying inside of the body is known as intracorporeal suturing.

Define the term keloid. Keloids are dense, unsightly connective tissue or excessive scar formation that is often removed surgically.

When is antibiotic prophylaxis administered? Antibiotic prophylaxis is administered 1 hour before the skin incision. An additional dose is given if the surgical procedure is longer than the half-life of the antibiotic.

Depending on their location hernias are classified as direct inguinal, indirect inguinal, femoral, umbilical, incisional, or epigastric. Hernias in any of these groups are either reducible or not reducible. The characteristic reducible hernia can best be described as a hernia:

With visceral contents that can be returned to the abdomen.

What is an indication for total hip replacement?

a. Arthritis

Trina Gasterson, a 7-year-old girl recovering from a successful kidney transplant, is visited by her perioperative nurse on her day of discharge. While they conversed, Trina's mother opened a bottle of herbal supplement tablets from a local nutrition and dietary supplement store, whose label the nurse recognized

a. Astragalus, licorice, and echinacea

A broad range of implant materials are used in plastic and reconstructive surgery. Select the implant that represents a biologic composition of a surgical implan

a. Autologous human tissue

When microlaryngoscopy is performed to remove polyps or nodules from the vocal cords, what important postoperative instruction must the nurse give the patient during discharge teaching?

a. Complete voice rest is important

What major artery supplies the descending colon?

a. Inferior mesenteric

A direct inguinal hernia is located _____.

a. Inside Hesselbach's triangle

Which of the following is incorrect pertaining to the CSF?

a. It returns to the venous system by passing into the vertebral venous plexuses.

LCIS... a. What is it? b. SSx? c. Mammographic findings? d. How is Dx made?

a. Lobular Carcinoma In Situ (CA in lobules of breast) b. none c. none d. found incidentally on biopsy

replantation is an attempt to reattach a completely amputated digit or other body part. good candidates for replantation are those with the following amputations

a. almost any body part of a child

Which of the following associations pertaining to the dural venous sinuses is incorrect?

a. arachnoid granulations—calcifications of arachnoid mater within the sinuses associated with arteriosclerosis

blunt force to the larnyz can result in fracture and impose immediate airway obstruction

a. assist the anesthesia provider with securing the airway while applying cricoid pressure

Camilla Armstrong, a 59-year-old woman with multiple prior cardiac procedures for congenital cardiac anomalies at birth, is scheduled for an open approach coronary artery bypass graft. During the preoperative interview and assessment, the perioperative nurse reviewed Camilla's hematologic tests, including a detailed coagulation profile to uncover possible hemorrhagic disorders. During the assessment the nurse learned of Camilla's many varied co-morbid conditions, including prior vascular procedures, fibromyalgia, diabetes, and eczema. The nurse noted one of Camilla's many nursing diagnoses as: Risk for Bleeding related to surgical incision(s), tissue dissection, altered coagulation function, and inadvertent hypothermia. As a result of this diagnosis, the nurse implemented measures to have blood available, measure and report blood loss, assist with the use of the autotransfusion system per protocol, and have topical hemostatic agents available. The patient will be free from excessive bleeding as evidenced by (select all that apply):

a. chest tube drainage less than 100 mL/hr.

Fibrocystic dz a. What is it? b. SSx? c. Dx? d. Tx for symptomatic dz?

a. common benign condition; fibrous (rubbery) and cystic changes in the breast b. breast pain or tenderness that varies w/ menstrual cycle; cysts; fibrous "nodular" fullness c. breast exam, Hx, aspirated cysts (usually straw-colored or green fluid) d. STOP CAFFEINE! Pain meds (NSAIDs), Vit E, evening primrose oil

What methods should the surgical assistant consider when cutting sutures?

according to surgeon's preference

Both ears provide stereophonic hearing that gives us very specific hearing capabilities. Binaural hearing makes it possible for:

a. determination of the source location of sounds

Marla, the perioperative nurse circulating for a skin contracture release of the ear on a 5-year-old boy, transfers 0.25% bupivacaine via syringe to the scrub person's sterile back table. The scrub person, Jake, questions why she is not giving him bupivacaine with epinephrine, since there may be bleeding expected at the incision site. Marla's reply to Jake is based on the fact that:

a. epinephrine is contraindicated in areas with limited vascularity, such as the ears

The adrenal glands lie retroperitoneally beneath the diaphragm, capping the medial aspects of the superior pole of each kidney. The adrenal medulla secretes ______________ while the adrenal cortex secretes ____________ and ____________.

a. epinephrine; steroids and hormones

The anterolateral abdominal wall consists of an arrangement of muscles, fascial layers, and muscular aponeuroses lined interiorly by peritoneum and exteriorly by skin. The key landmark of the _____________ designates the roof of the inguinal canal and the key landmark of the __________ designates the floor of the inguinal canal.

a. external oblique aponeurosis; transversalis aponeurosis and fascia

Lymph channels run parallel to which structures?

veins

In the following illustration, the arrow traverses the:

a. transverse pericardial sinus

What is the #6 cranial nerve and what does it innervate?

abducens, eyes

Which movements of the upper extremity would be most painful if tendinitis of the supraspinatus muscle is present?

abduction of the shoulder

Which is the best indicator for possible failure of a below-th-knee amputation to heal?

absence of a popliteal artery pulse when palpated

Polyglactin 910 suture is

absorbable, synthetic

Which nerves innervate the trapezius and sternocleidomastoid muscle, and should be identified and preserved when performing a radical neck dissection?

accessory

Which of the following tumors is most commonly found in the cerebellum of children and in the cerebrum of adults and is the most common primary brain tumor?

astrocytoma

A prolongation of the breast tissue located in the upper, outer part of the breast that passes through an opening in the axillary fascia is called the

axillary tail of Spence

What is the name of the large vein that marks the upper limit of the axilla?

axillary vein

Which vein drains the right side of the vertebral column?

azygos

The thoracic cavity is an airtight compartment. The lateral walls are formed by 12 pairs of ribs. Each hemithorax contains 11 external intercostal muscles and 11 internal intercostal muscles. What accompanies each intercostal muscle?

c. an artery, vein, and nerve

Denise Michaelson had a scar revision of her knee performed along with her bilateral mastectomy with saline implants and ear piercing procedures. The knee scar, before revision, was an 8-cm linear, thin scar that extended obliquely across the anterior portion of her knee. The tension of the scar tissue met resistance when she would bend her knee. The surgeon opted to remove the scar with a procedure to break up the linear scar and rearrange the tissue direction to form a natural line. The proper name of this tissue transfer scar revision is the:

d. Z-plasty.

Which of the following is incorrect pertaining to the pattern of referred pain typically associated with angina pectoris?

d. may be felt in left fingers

an exapmple of an indicaiton for liver transplantation would be

d. primary hepatic cancer

Denise Michaelson, a 46-year-old woman with an early-stage second primary cancer in her left breast, has elected to have a bilateral mastectomy with saline implants. She has requested that the surgeon also revise a small unsightly scar on her right knee and pierce her ears. While Denise's breast tumor could be treated with an excisional biopsy, she has decided to have a mastectomy of both breasts, which is considered an appropriate preemptive (preventative) surgery for a woman of her age with two primary cancers of the breast. The saline implant insertion surgery is considered a _______________ procedure, the scar revision is considered a ___________ procedure, and the ear piercing is considered a _______________ procedure.

d. reconstructive, cosmetic and cosmetic

Which types of fistulas refers to leakage of urine into the vagina?

vesicovaginal

Medication given to combat malignant hypothermia is

dantrolene (2-3mg/kg)

The tibial and common peroneal nerves are terminal branches of the what nerves?

sciatic

Which of the following statements concerning the principles of suture placement is correct? A. Sutures should be placed as close to the wound edge as possible to prevent tissue necrosis. B. Sutures should be as far as possible from the wound edge to prevent knot slippage and displacement. C. Sutures should be placed in deep tissue to prevent hematoma. D. Suture placement should place as much tension on the tissue as possible to promote healing.

x

Physiologic Changes in Sitting Position: Pulmonary

*Increase in FRC *Increase in all Lung Volumes *Decreased Peak Inspiratory Pressure

Which of the following is an example of an isotonic IV solution?

0.9% sodium chloride

Which nerve can be injured if the legs are improperly flexed at the hip in the lithotomy position? A. Obturator B. Saphenous C. Femoral D. Peroneal

A. Obturator

Which of the following is a dye used in ophthalmic surgery to diagnose corneal abrasions? A. Fluorescein sodium B. Methylene blue C. Indigo carmine D. Gentain violet

A. Fluorescein sodium

The lateral ventricles within the cerebral hemisphere communicate with the third ventricle through the opening called: A. Foramen of Monro B. Foramen Ovale C. Foramen of Vesalius D. Foramen Magnum

A. Foramen of Monro

An organelle that is responsible for the packaging of proteins is the: A. Golgi complex B. Mitochondria C. Cilia D. Nucleus

A. Golgi complex

Q. Which of the following types of scissors would be considered tissue or dissecting scissors?

A. Sharp and Blunt. Explanation:The type of scissors to be used will be determined by the surgeon. This will depend on the type and location of the tissue that is to be cut. For tissue or dissecting, the sharp or blunt scissors are most often used

Q. The principle of nursing which holds that nurses should always tell the truth is known as which of the following?

A. Veracity Explanation: Veracity is a principle of nursing which holds that nurses should always tell the truth. Lying to or withholding information from a patient risks harming the trust between nurse and patient which is key for a functioning therapeutic relationship.

Q. The tissue in a person's brain and spinal cord which carries electrical impulses from one part of the nervous system to another would be considered which of the following?

A. White Matter Explanation: The white matter in the brain is made up of nerve fibers which have the responsibility of carrying the electric impulses through the nervous system. Unlike the gray matter, the white matter actually contains very few cell bodies.

A patient scheduled for surgery has a hematocrit reading of 45% of total blood volume, which is: A. Within normal range B. Below normal range C. Above normal range D. Inconclusive

A. Within normal range

Q. The set of steps used to make a nursing decision is known as which of the following?

A. algorithm Explanation: The algorithm is the set of steps used to make a nursing decision. The procedure refers to the set of "how-to" action steps; the standard of care refers to a description of the acceptable level of patient care. Clinical practice guidelines are statements outlining appropriate practice for clinical condition or procedure.

Mechanical, thermal, and chemical are appropriate mechanisms to achieve: A. Hemodialysis B. Hemostasis C. Hemolysis D. Hemoptysis

B. Hemostasis

Which cranial nerve emerges from the medulla, passes through the skull, and descends through the neck region into the thorax and abdominal region? A. IX B. X C. XI D. XII

B. X

Which of the following areas located in frontal gyrus of the brain controls the function of speech? A. Brodmann's B. Broca's C. Temporal D. Cerebral cortex

B. Broca's

Aminoglycosides are a category of antibiotics that: A. Inhibit bacterial replication B. Can cause severe ototoxicity and nephrotoxicity C. Are broad spectrum D. Can cause photosensitivity

B. Can cause severe ototoxicity and nephrotoxicity

Which of the following are branches of the left coronary artery? A. Anterior interventricular and right marginal B. Circumflex and anterior descending arteries C. Posterior interventricular and diagonal D. Sinoatrial nodal and atrioventricular nodal

B. Circumflex and anterior descending arteries

Which of the following drugs paralyzes the sphincter muscle of the iris? A. Miotics B. Cycloplegics C. Enzymes D. Viscoelastics

B. Cycloplegics

The three structures that form the anatomical triangle of Calot: A. Cystic artery, right hepatic duct, common bile duct B. Cystic duct, common hepatic duct, lower edge of the liver C. Cystic duct, common hepatic duct, abdominal aorta D. Cystic artery, common bile duct, superior edge of duodenum

B. Cystic duct, common hepatic duct, lower edge of the liver

A microscopic blood count that estimates the percentages of each type of leukocyte within a sample is called a: A. Platelet B. Differential C. Hematocrit D. Hemoglobin

B. Differential

The doctrine of respondent superior refers to: A. The legal terms for assault and battery B. Employer liability for employee's negligent conduct C. Professional misconduct D. "The thing speaks for itself."

B. Employer liability for employee's negligent conduct

Which of the following characteristics of drape materials is the correct? A. Able to absorb fluids B. Flame retardant C. Reflect operating lights D. Contain line

B. Flame retardant

Valve's of the gallbladder are called: A. Calot's B. Heister C. Hepatic D. Interlobular

B. Heister

Liability is a legal rule that: A. Holds the patient responsible for dictating his/her care B. Holds each individual responsible for his or her own actions C. Holds the hospital responsible for its personnel D. Holds the surgeon solely responsible for the care of the patient

B. Holds each individual responsible for his or her own actions

The principle of personal liability: A. Applies only to someone employed outside of a hospital B. Holds everyone legally responsible for their own negligent actions C. Holds the hospital responsible for negligence D. Prevents malpractice suits against hospital employees

B. Holds everyone legally responsible for their own negligent actions

Which of the following is a measurement of the pressure of carbon dioxide in the arterial blood? A. Partial pressure B. PaCO2 C. PaO2 D. QO2

B. PaCO2

Measures to treat atelectasis include: A. Bed rest and limited ambulation B. Pain control and early ambulation C. Repeated vigorous coughing D. Antimycotic agents

B. Pain control and early ambulation

The blood component administered to patients with a clotting problem is: A. Packed red blood cells B. Platelets C. Whole blood D. Serum albumin

B. Platelets

In the lateral chest position, a sandbag or padding is placed under the chest at the axillary level to: A. Aid in position stability B. Prevent pressure on the lower arm C. Promote proper body alignment D. Add warmth for the patient

B. Prevent pressure on the lower arm

Which ligament is attached to the posterior lateral condyle of the femur and to the notch of the midline of the tibia between the tibial condyles? A. Posterior cruciate B. Patellar C. Anterior cruciate D. Lateral collateral

C. Anterior cruciate

Which of the following are procaryotic? A. Molds B. Protozoa C. Bacteria D. Plants

C. Bacteria

The most common skin cancer is: A. Primary melanoma B. Actinic keratosis C. Basal cell carcinoma D. Squamous cell carcinoma

C. Basal cell carcinoma

Which of the following is a result of potassium imbalance? A. Bone loss B. Decreased osmotic pressure C. Cardiac arrhythmias D. Water intoxication

C. Cardiac arrhythmias

Which of the following antibiotics should not be given as a preoperative prophylaxis to a pediatric patient? A. Aztreonam B. Gentamicin C. Ciprofloxacin D. Cefazolin

C. Ciprofloxacin

The liver is divided into two principal lobes by which of the following ligaments? A. Cruciate B. Round C. Falciform D. Coronary

C. Falciform

Collagen is produced by which of the following connective tissue cells? A. Leukocytes B. Osteoclasts C. Fibroblasts D. Erythrocytes

C. Fibroblasts

When an immediate confirmation of malignancy is needed, the specimen will be sent to pathology for: A. Permanent section B. Culture and sensitivity C. Frozen section D. Identification

C. Frozen section

Which of the following cranial nerves innervates the heart?

CN X

What is the Tx of inflammatory CA of the breast?

Chemotherapy first! Then often followed by radiation, mastectomy, or both

The junction of the superior mesenteric and splenic veins forms the: A. Inferior vena cava B. Femoral vein C. Middle colic D. Portal vein

D. Portal vein

Which two electrolytes are essential for normal cardiac contractions? A. Phosphate and chloride B. Magnesium and sodium C. Bicarbonate and sulfate D. Potassium and calcium

D. Potassium and calcium

The anatomical division between the inguinal and femoral regions is the : A. Femoral artery B. Cooper's ligament C. Internal spermatic vessels D. Poupart's ligament

D. Poupart's ligament

Which vein proximally travels across the neck of the abdominal aorta? A. Inferior mesenteric B. Left common iliac C. Splenic D. Renal

D. Renal

What are the signs and symptoms of a transfusion reaction? Signs and symptoms of a transfusion reaction include: increased intraoperative bleeding, hyperthermia, weak pulse, hypotension, visible hemoglobinuria, vasomotor instability, and greatly decreased or no urinary output

Describe the steps used when weighing sponges to estimate blood loss. The procedure for weighing sponges is as follows: (1) Weigh a dry sponge in a plastic bag, and adjust the scale to register zero. (2) Place used, bagged sponges on the scale. (3) Record the scale reading: 1 g equals 1 mL of blood loss. (4) Note the blood loss on the record. (5) Add subsequent weight to the preceding weight each time sponges are weighed so that a running total blood loss, calculated from sponges, is available. (6) Measure blood in the suction canister(s) at regular intervals, subtracting the amount of any irrigating solution used. (7) Add the amount of blood loss calculated from suction canisters to the total recorded from sponges to obtain accurate blood loss estimates

Describe how sutures are measured. Suture size is measured in millimeters, and expressed in USP sizes with zeroes. The smaller the cross-sectional diameter, the more zeroes; sizes range from #7, the largest, to 11-0, the smallest

Describe when surgical counts (e.g., sponge, sharp, instrument) are performed. Counts are performed initially, before the patient enters the operating room (OR), before closure of a body cavity or deep, large incision; after closure of a body cavity; and at skin closure

While direct and indirect hernias both protrude into the inguinal canal and represent tears in the transversalis fascia which one of the two occurs within the Hesselbach triangle

Direct inguinal hernia

What structure provides leverage to the muscles attached to it to rotate the femur on its axis?

Greater trochanter

The series of arcades along the mesenteric border thorughout the length of the colon is called the

Margical artery of drummond

Describe key features used to evaluate suture material. Key features to evaluate when choosing suture material include: (1) physical characteristics, (2) handling characteristics, and (3) tissue-reaction characteristics.

List examples of cutting instruments. Cutting instruments include scalpels, scissors, drills, saws, osteotomes, rongeurs, adenotomes, dermatomes, and curettes

Describe Stage III anesthesia. Stage III spans from the onset of a regular breathing pattern until cessation of respiration.

List examples of direct care providers for anesthesia services. Direct care providers include: anesthesiologist, certified registered nurse anesthetist (CRNA) working alone or in collaboration with or under the direction of an anesthesiologist or a physician, and anesthesiologist's assistant (AA) working under the direct supervision of an anesthesiologist.

Describe Stage II anesthesia. Stage II spans from loss of consciousness to the onset of regular breathing and loss of the eyelid reflex.

List examples of intermediate-acting nondepolarizing muscle relaxants. Mivacurium, atracurium, cisatracurium, rocuronium, and vecuronium are intermediate-acting nondepolarizing medications.

What factors must be present to achieve steam sterilization? Time, temperature, and saturated steam are necessary for steam sterilization.

List the sequence for surgical incision draping. Draping begins at the area of the intended incision and proceeds outward to the periphery. Always drape from a sterile area to an unsterile area by draping the near side first.

What is the primary mode of transmission implicated in methicillin-resistant Staphylococcus aureus (MRSA) infections? The primary mode of transmission for MRSA is via direct contact from the hands of healthcare personnel.

List the sequence of steps to remove soiled surgical gloves. Do the following to remove surgical gloves: (1) Using the gloved fingers of one hand to secure the everted cuff, remove the glove, turning it inside out. Discard in biohazard trash. (2) Using the ungloved hand, grasp the fold of the everted cuff of the other glove and remove the glove, inverting the glove as it is removed. Discard in biohazard trash.

Define the term adhesions. Adhesions are described as the adherence of serous membranes to one another, causing fibrous tissue to form.

List the surgical techniques that maximize wound healing. Techniques to maximize healing include: adequate hemostasis, precise cutting and suturing techniques, efficient use of time to minimize wound exposure to air, elimination of dead spaces, and minimal pressure from retractors and other instruments.

What is monitored anesthesia care? Monitored anesthesia care is local anesthesia provided by the surgeon supplemented with monitoring and intravenous (IV) drugs administered by an anesthesia provider to provide sedation and systemic analgesia.

List the symptoms of local anesthetic toxicity. Local anesthetic toxicity may be manifested by the following symptoms: perioral numbness, metallic taste in the mouth, ringing in the ears, visual disturbances, and/or dizziness.

Describe Stage I anesthesia. Stage I spans from the initial administration of anesthetic agents to loss of consciousness.

List the symptoms of malignant hyperthermia. Hypercarbia, tachypnea, tachycardia, hypoxia, metabolic and respiratory acidosis, cardiac dysrhythmias, and elevation of body temperature at a rate of 1 to 2° C every 5 minutes are symptoms associated with malignant hyperthermia.

What are the radiographic tests for breast cancer?

Mammography and breast US MRI

Low tension on the wound, select proper materials and needle, skin PENETRATION AT 90 degrees, proper tension, knot tightening, needle handling

Skin suturing principles

Which of the following types of stitch patterns provides an excellent cosmetic closure? A. Subcuticular stitch B. Retention stitch C. Purse string stitch D. Simple stitch

Subcuticular stitch

With what pts should you avoid LMWH?

Those with renal insufficiency (CrCl <30ml/min), Pts over 120kg (obese)

This type of ulcer is also known as a venous stasis ulcer.

Venous insufficiency ulcer

This part of virchows triad is seen as blood return from legs impaired and blood pooling has more clotting factors, initiating an clotting process

Venous stasis

DVT often begins in ________

Venous valves cusp

The T-wave indicates: A. Ventricular repolarization B. Atrial repolarization C. The strength of the electrical signal D. The time interval between atrial and ventricular contractions

Ventricular repolarization

Define the term chronic wound. A chronic wound is one caused by an underlying pathophysiologic condition over time.

What type of wound healing occurs when the wound is closed with sutures, staples, or tape applied as soon after the time of injury as possible? Primary intention occurs when the wound is closed with sutures, staples, or tape applied as soon after the time of injury as possible.

What are the levels of the axillary lymph nodes? a. Level I (low) b. Level II (middle) c. Level III (high)

a. lateral to pectoral minor b. deep to pectoral minor c. medial to pectoral minor In breast cancer, a higher level of involvement has a worse prognosis, but the level of involvement is less important than the # of positive nodes

In the following illustration, the arrow points to the:

a. left vagus nerve.

what incision is used for lumbar laminectomy

a. midline direcly over the appropriate lumbar spine

When should the patient's chest tubes be attached to te drainage system during a thoracotomy?

after the thoracic cavity is closed

Delayed wound healing may be related to

age

During ulnar nerve transposition, the nerve is relocated to the

anterior aspect of the medial epicondyle

Which ligament is attached to the posterior lateral condyle of the femur and to the notch of the midline of the tibia between the tibial condyle?

anterior cruciate

Following injury to the lower limb, your patient's knee is evaluated. His leg can easily be displaced anteriorly on a fixed thigh. This would indicate a torn

anterior cruicate ligament

When performing a cannulated screw fixation for nondisplaced femoral neck fractures, compression of the

anterior screws is competed first

Hemolytic reactions caused by ABO

antibodies to the donor foreign proteins

The third ventricle empties into the fourth ventricle through which small duct, which is the only exit for the third and lateral ventricles?

aqueduct of Sylvius

Prior to plating a mandibular fracture, what should be applied to ensure proper occlusion of the teeth?

arch bars

The point of seperation between the posterior rectus sheath superiorly and absence of the sheath inferiorly is called the

arcuate line

What is the durg of choice when lidocaine fails to achieve response for treating ventricular tachycardia/ventricular fibrillation during sudden cardiac arrest?

bretylium

Which ligaments enclose the round ligament and extends from the lateral surface of the uterus to the pelvic wall?

broad

Which of the following areas located in the frontal gyrus of the brain controls of the function of speech?

brocas

By definition, carcinoma in situ is a malignant neoplasm that has not

extended beyound the basement membrane

Countertraction

force placed on the same object in the opposite direction

What is a good way to prevent hypothermia in the OR

forced air warming blanket

Which lymph nodes must be removed and histologically examined when performing a segmental pulmonary resection for lung cancer?

lobar

During a vascular procedure, monitoring the activated clotting time intraoperatively provides useful data for judging the need for reversal or addition of

heparin

During common bile duct exploration, which structures are more likely to be accidently injured?

hepatic artery

Which structures must be anastomosed during a liver transplant?

hepatic artery

The setting of the pneumatic tourniquet pressure on an average adult for a lower extremity should be

higher than the systolic pressure by one-half of the value

The principle of personal liability

hold everyone legally responsible for their hown negligent acts

Liability is a legal rule that

holds each individual responsible for his or her own actions

What is a potential physiological response when lowering patient's legs from the lithotomy position?

hypotension

Acid based imbalances reflect

hypoxia, hypothermia, and hypovolemia

Which hernia is typically the result of postoperative wound dehiscence?

incisional

What does Laplace's law say about the size of the aneurysm and risk of rupture?

increased size/radius --> increased wall tension --> increased risk of rupture

An emergency durg used for treating ventricular fibrilation or tachycardia is

inderal

The arterial supply to the bladder is derives mainly from the

internal iliac

The utereine artery is a branch of the anterior trunk of the

internal iliac artery

The uterine artery is a branch of th anterior trunk of the

internal iliac artery

If the suture breaks and the remainder of the suture line stays intact, which suture technique was utitlized?

interrupted

A 60 year old obese patient with a history of diabetes and renal insufficiency has undergone drainage of an abdominal abscess through a midline incision, what type of stitch should be considered for wound closure?

interrupted retention suture

the invagination of the proximal intestine into the lumen of the distal intestine is called

intussusception

Postoperatively, what is closely monitored immediately after a loop colostomy is performed?

ischemia

A patient who has undergone a right-side herniorrhaphy presents 2-5 days postoperatively with swollen and tender testis. What is a likely complication?

ischemic orchitis

The thyroid gland consists of right and left lobes joined by the

isthmus

What nerve runs MEDIAL to the medial pectoral nerve? What does it innervate?

lateral pectoral nerve • innervates the pectoral major • names describe orientation from the brachial plexus

The rectus sheath that meet to form a tough fibrous white band that extends from the xiphoid process to the pubic symphysis is the

linea alba

Winged scapula is a condition brought on by an injury to what nerve?

long thoracic

After receiving a preoperative epidural catheter, a patient is given a bolus of local anesthetic intraoperatively. The patient then demonstrates signs of hypotension and complete respiratory failure. Which of the following is the MOST likely cause

migration of the epidural catheter through the dura

What is the "triad of error" for misdiagnosed breast cancer?

more than 75% of cases of misdiagnosed breast cancer have these characteristics: 1. age < 45 years 2. self-diagnosed mass 3. negative mammogram

What type of invasive breast CA do patients with LCIS develop?

most commonly, infiltrating ductal carcinoma with equal distrbution in the contralateral and ipsilateral breasts

A threaded cortical screw cannot provide compression of bone fragments unless the

near cortex is overdrilled

What is a needle loc biopsy (NLB)? What is a mammotome Bx?

needle localization by radiologist, followd by Bx. Checked by mammogram to be sure all of lesion has been excised mammogram-guided computerized sterotactic core biopsies

The most common intraabdominal childhood tumor is

nephroblastoma

What is not considered a permanent urinary diversion?

nephrostomy

What do the external iliac arteries supply?

pelvic limb (femoral artery), mamma/scrotum, ventral abdominal wall

What do the internal iliac arteries supply?

pelvic muscles, pelvic viscera, buttock region (perineum

Common pathogens infecting traumatic wounds often require the use of

penicillinase-resistant penicillin

Which of the following sterilants is utilized by the Steris System I sterilizer?

peracetic acid

Which two tests are the most cost-efficient in preoperative determination of calculi in the bile tract?

percutaneous ultrasound, liver function tests

The branches of the deep femoral artery that travel posterior to the femur and pierce muscles and intermuscular septums are called the

perforators

Is ligating temporary or permant?

permanent

When positioning a patient for a gnecological procedure involving the candy cane stirrups, the surgical assistant is deliverate in avoiding pressure on the laterl aspect of the patient's knee to prevent injury to which nerve?

peroneal

The curved transverse incision used for pelvic surgery is called a/an

pfannensteil

The blood component administered to patients with a clotting problem is

platelets

What types of blood components is the most common source of bacterial infection in a transfusion?

platelets

What sutures should not be used int he placement of heart valves?

polydioxanone

Which vein is shared by the pancreas and liver?

portal

The junction of the superior mesenteric and splenic veins forms the

portal vein

In which circumstance could the patient sustain an injury to the pudenal nerves?

positioned on the fracture table

After completion of an ORIF of the distal radius utilizing a volar plate and screws, the dressing application should include what splint?

posterior wrist

Which two electrolytes are essential for normal cardiac contractions?

potassium and calcium

The anatomical division between the inguinal and femoral regions is the

poupart's ligament

When removing anal warts with a laser, the rectum is packed with a wet sponge to

prevent escape of methane gas

This tissue is concerned with the formation f red blood cells.

red marrow

What structures of the kidney does urine drain into the major calyces?

renal pelvis

During a routine shoulder arthroscopy on a patient wiht instability, a surgeon notices that the patient has a type II SLAP lesion. What is the appropriate treatment for this finding?

repair the glenoid labrum

What is the sequence of events leading to a neurotrophic ulcer?

repetitive microtrauma, since neuropathic, no pain so inflamm continues causing congestion which occludes local blood supply, ischemic area dies and form thick callus, and soon an ulceration.

If a Kelly clamp is left in a patient who underwent a cholecystectomy, which legal charges can be filed

res ipsa loquitur

Which of the following complications is MOST critical when positioning a patient for surgery?

respiratory compromise

The trauma patient with a cervical spine injury at or near C3 or C5 is at great risk for

respiratory difficulties

What is the loose areolar tissue that seperates the breast fromt he pectoralis major muscle?

retromammary space

The position used typically in thyroid and gallbladder surgery is

reverse trendelenburg

What is the most common cause of mitral stenosis?

rheumatic fever

The anastomsis, or linking, of arcades between the superior and inferior mesenteric vessels is known as the long anastomosis of

riolan

What ligaments travel in the upper part of the broad ligaments below and in front of the fallopian tubes, passing over the vesicle vessels and nerve?

round

What does the median sacral artery supply?

sacral vertebrae, caudal vertebrae and tail

DCIS cancer occurs in ____ breast. LCIS cancer occurs in ______.

same either (equal risk in both breasts)

Which vessels typically utilized for a coronary artery bypass?

saphenous vein and internal mammary artery

Whats an example of an addition superficial vein?

saphenous veins and cephalic veins

What are veins that accompany the major arteries called?

satellites of the arteries

What is Paget's dz of the breast?

scaling rash / dermatitis of the nipple caused by invasion of skin by cells from a ductal carcinoma

A Stamey endoscopic procedure is performed to

suspend the vesicle neck

Where is the transverse mesocolon attached?

transverse mesocolon is a double fold of peritoneum travelling from the transverse colon to the front of the 2nd part of duodenum, and to the anterior aspect of the head and the superior border of body and tail of pancreas. adherant to the posterior surface of the greater omentum

What is at the greatest risk for fluid volume deficit?

trauma victim with extensive burns of the body

The acronym NAVEL refers to the anatomical placement of the nerve, artery, vein, empty space, and lymph node in the femoral triangle. *true or false

true

What procedure steps are done under warm ischimia?

tumor excision, close collecting system and small vessels, reapproximate renal defect (6-8)

Abdominal aortic aneurysm is usually associated with _____ symptoms.

usually associated with NO symptoms

What are the physical/anatomic risk factors for breast cancer?

"CHAFED LIPS" Cancer in the breast Hyperplasia Atypical hyperplasia Female Elderly DCIS LCIS Inherited genes (BRCA) Papilloma Sclerosing adenosis

In the following diagram, identify the following arterial branches of the celiac trunk: Right hepatic artery; Left hepatic artery; Proper hepatic artery; Celiac trunk; Splenic trunk; Left gastric artery; Gastroduodenal artery

1. Celiac trunk..............................2. Splenic trunk 3. Left gastric artery.....................4. Common hepatic artery 5. Gastroduodenal artery.............6. Proper hepatic artery 7. Left hepatic artery....................8. Right hepatic artery

What are the 3 areas of spinal cord perfusion?

1. Superior (cervical, upper thoracic) 2. Mid Thoracic (T2-T8) 3. Thoraco-Lumbar (T8-L1/L2) Flow is bidirectional and depends on collaterals

In the diagram, identify the following nerves in the axillary region:

1. Thoracodorsal.......................2. Long thoracic 3. Medial pectoral......................4. Lateral pectoral

The recommended minimum number of air exchanges in the or is

15x/hr with 3 exchanges of fresh air

What is the amount of time that heparin is systemically distributed in the adult: A. 3 minutes B. 6 minutes C. 9 minutes D. 12 minutes

A. 3 minutes

Spinal fluid is produced by the: A. Choroid plexus B. Lateral ventricle C. Dura mater D. Cerebrum

A. Choroid plexus

The head of the pancreas is surrounded by the: A. Duodenum B. Jejunum C. Ileum D. Pyloris

A. Duodenum

Q. The _____ saw is an unpowered type saw that is used during amputations?

A. Gigli Explanation: Most saws and bone-cutting instruments are powered. The one that is not powered that is commonly used during an amputation would be the Gigli saw.

Burr holes are placed to

A. Remove localized fluid collection secondary to head trauma B. Gain access to place ventricular shunts or catheters C. Gain access for the craniotome to turn a bone flap D. Gain access for the craniotome to remove a bone plate

Q. Your patient has been diagnosed with multiple sclerosis and is having a hard time dealing with the diagnosis. She tells you that she feels helpless and worthless to her family now. She is having a problem with which of the following stages of Maslow's hierarchy of needs?

A. self-esteem needs Explanation: This patient is not meeting her self-esteem needs. At the level of self-esteem needs, when a person doesn't meet the need to feel self-confident and valuable as a person, she feels helpless, and worthless.

Which knee ligament prevents the femur from sliding posteriorly on the tibia and is the usual site for knee ligament reconstruction?

ACL

It is recommended that the square knot is: A. Performed as a two-handed tie B. Only used on small vessels C. Performed by placing vertical tension on the final throw D. Only used on superficial, not deep wound, vessels

Accomplished as a two-handed tie

In the risk evaluation of DVT, what risk factors warrant one point each?

Age 41-60, Prior history of post-op DVT, Family hx of DVT/PE, Leg swelling/ulcer/stasis/varicose veins, MI/CHF, stroke w/ paralysis, IBD, Central line, bed immobilization >12h. Anesthesia >2h.

Hydrogen peroxide, acetic acid, dakins solution, cadexomer iodine (iodosorb/iodoflex) are all what?

Antiseptics

What is the indication for total hip relpacement?

Arthritis

This is the process by which a wound bed clears itself of debris by the wounds on proteolytic exudate, maintians a moist wound bed, slow, pain free, and HYDROGEL is usually used

Autolytic debridement; d/c if increased signs of inflamm

Which of the following nerves innervates the latissimus dorsi muscle? A. Subscapular B. Thoracodorsal C. Lateral pectoral D. Intercostobrachial

B. Thoracodorsal

Langer lines generally lie perpendicular to the direction of: A. Skin folds B. Underlying muscle tension C. A midline incision D. Movement of the bone joints

B. Underlying muscle tension

Which of the following arteries supplies blood to the brain? A. Anterior and middle cerebral arteries B. Vertebral and internal carotids C. External and internal carotids D. Occipital

B. Vertebral and internal carotids

Why shouldn't the pt be paralyzed during an axillary dissection?

Bc the nerves (long thoracic/thoracodorsal) are stimulated with resultant muscle contraction to help ID them They can be stimulated w/ a forceps, which results in contraction of the latissimus dorsi (thoracodorsal nerve) or anterior serratus (long thoracic nerve)

While removing a cancerous testis the surgeon inadvertently allowed fluid from the testis to flow over the incised skin of the scrotum The surgeon should:

Be concerned about potential spread of cancer to superficial inguinal lymph nodes

This is the type of debridement which uses live organisms (maggots/leeches), to assist in medical debridement

Biotherapy debridement

How do you close a surgical wound

By approximate the skin edges and not overlap, DO NOT OVERLAP, THIS WILL BE UNACCEPTABLE

Which of the following lymph nodes is located between the pectoralis major and minor muscles? A. Axillary B. Inguinal C. Rotter D. Thoracic

C. Rotter

Which of the following ligaments travels in the upper part of the broad ligaments below and infront of the fallopian tubes, passing over the vesicle vessels and nerve? A. Cardinal B. Uterosacral C. Round D. Pubofemoral

C. Round

What are the most common lasers used in gynecological surgery?

CO2 and Nd: YAG

An emergency drug used in cardiac surgery that increases myocardial contractility is: A. Calcium chloride B. Levarterenol (Levophed (R)) C. Isoproterenol hydrochloride (Isuprel (R)) D. Furosemide (Lasix (R))

Calcium chloride

Vicryl is coated with this lubricant, that gives it superb handling and smooth tie down properties

Calcium stereate and Polyglactin 370

The most frequent location for a cerebral aneurysm is the

Cerebral arterial circle

The three phases of wound healing include all of the following EXCEPT: A. Fibroplastic phase B. Chronic phase C. Remodeling phase D. Inflammatory phase

Chronic phase

What type of vicryl is recommended in SubQ

Clear

What is the gold standard invasive test for DVT?

Contrast venography

Debridement, control of bacterial overburden, irrigation, and systemic support are the things we do to do what to a wound?

Convert a chronic wound to an acute wound

What are the suspensory breast ligaments called?

Cooper's ligaments

Structure covers the spermatic cord and contracts to make the testicles ascend in the scrotum?

Cremaster muscle

Describe the clinical signs of hypotension. Hypotension is characterized by rapid, thready pulse; disorientation; restlessness; oliguria; and cold, pale skin.

Define and describe bronchospasm. Bronchospasm is a lower airway obstruction caused by spasms of the bronchial tubes. The patient presents with wheezing, dyspnea, use of accessory muscles, and tachypnea.

Describe examples of rigid endoscopes. Rigid endoscopes include: cystoscopes, laparoscopes, sinuscopes, arthroscopes, bronchoscopes, laryngoscopes, and hysteroscopes.

Define capacitative coupling. Capacitative coupling occurs when energy is transferred through intact insulation on the laparoscopic instrument shaft to nearby conductive materials.

What actions should you take in the event of an incorrect sponge, sharp, or instrument count? In the event of an incorrect count, you should notify the surgeon of the unresolved count. Search for the missing item, including the surgical wound, field, floor, linen, and trash. Ask all personnel to direct their immediate attention to locating the missing item. If the item is not found, an X-ray film may be taken and read by the radiologist or surgeon as specified in institutional policy

Define the term intracellular fluids. Intracellular fluids (ICFs) are liquids within cell membranes that contain dissolved substances essential to fluid and electrolyte balance and metabolism. These substances comprise 70% of the body's fluid.

What characteristics are thought to increase the likelihood of acquired position-related neuropathies? Diabetes, peripheral vascular disease, electrolyte imbalance, vitamin B12 deficiency, alcoholism, smoking, malnutrition, advanced age, previous nerve injury, arthritis, obesity, and thin body habitus.

Define the term shear. Shear is the folding of underlying tissue when the skeletal structure moves while the skin remains stationary.

List the types of steam sterilizers used in the OR. Gravity displacement, pre-vacuum, and immediate use (flash) sterilizers are all examples of steam sterilizers used in the OR.

Define the term sterilization. Sterilization is the complete elimination or destruction of all forms of microbial life.

2 semi- elliptical skin wedges of the 5th toe run from dorsal-lateral- proximal to dorsal- medial distal over the PIPJ

Derotational arthroplasty

Describe the Trendelenburg position. In the Trendelenburg position, the patient is supine with the upper torso lowered and the feet raised.

Describe the lithotomy position. In the lithotomy position, the patient is supine with the legs raised and abducted, usually in stirrups.

What 3 factors are involved in the conversion form bacterial colonization to a bacterial wound infection?

Dose/amt of bacteria, Virulence of bacteria, and host resistance

_______________ compression is a force that changes as needed depending on circumstances or outside influences.

Dynamic

This type of debridement involves application of a substance which chemically digests and removes necrotic tissue.

Enzymatic debridement

It is well suited for skin retention and skin closure becausde of their elasticity, clear or dyed or black, degrades in vivo at 15% to 20% per year by hydrolysis

Ethilon Nylon suture - with memory Memory is the tendency to return to the original state

What is the anesthetist's goal for fluid management during AAA repair? (A) hypovolemia (B) euvolemia (C) hypervolemia (D) match urine output mL/mL

Euvolemia is important in assuring normal CV/ Renal performance

True or false: it is okay for the assistants first time in surgery to look like their first time in surgery because of poor technique

FALSE-pre practice all techniques

What are the methods for obtaining tissue for pathologic examination?

FNA Core biopsy (larger needle core sample) mammotome stereotactic biopsy open biopsy - can be incisional (cutting a piece of the mass) or excisional (cutting out the entire mass)

The lateral ventricles within the cerebral hemisphere communicate with the third ventricle through the opening called

Foramen of Monro

This is placed in the IVC if there is clinical or venographic evidence of clot mvmt within the deep venous system

Greenfield filter

What microorganisms is responsbile for causing postoperative wound edge erythema and rebound tenderness?

Group B steptococci

What is a measurement of the amount of bicarbonate in arterial blood?

HCO3

If a pt with a confirmed case of DVT has a platelet count of under 100k, or 50% baseline or 10% drop in the 1st 24h of LMWH therapy, what are they at risk of?

Heparin induced thrombocytopenia

What drugs are used in prophylaxis of DVT?

Heparin, LMWH, IPC's, TED hose

An example of a common postoperative complication related to an inguinal hernia surgery is

Hernia recurrence

Mersilene in uncoated to provide ___________

High coefficient of friction when passed through tissue

Define the term osmosis. Osmosis is the movement of a fluid through a semipermeable membrane from a solution that has a lower solute concentration to one that has a higher solute concentration.

How are first, second, and third spacing defined? First spacing is the normal distribution of fluid in extracellular and intracellular compartments. Second spacing refers to excess accumulation of interstitial fluid (edema), while third spacing occurs when fluid accumulates in areas that normally have no fluid or only a minimal amount of fluid

During the adminstration of general anesthesia, there is a loss of the eyelid reflex in stage

II

What causes localized temperature elevation during the inflammatory response?

Increased blood flow

An emergency drug used for treating ventricular fibrillation or tachycardia is: A. Aramine (R) B. Atropine (R) C. Inderal (R) D. Calcium chloride

Inderal (R)

(______) = (dose-x-virulence)/(host resistance). What is _________?

Infection

What are the two arteries branching off the posterolateral abdominal aorta?

Inferior phrenic lumbar arteries

What is the MC type of breast cancer?

Infiltrating ductal carcinoma

Vasodilation is part of what phase of wound healing

Inflammatory phase

What are the three phases wound healing?

Inflammatory phase, Fibroplastic phase, remodeling phase

Identify the ligament that originates at one spinous process and terminates at an adjacent spinous process.

Interspinous ligament

What is formed by the inferior vertebral notch above and the superior vertebral notch from an adjacent vertebra below?

Intervertebral foramen

Which of the following fractures runs between the maxillary and orbital floors and may involve the medial and lateral walls of the maxillary sinuses and the pterygoid processes of the sphenoid?

Le Fort I

List the risk factors for hypothermia. Age extremes (elderly and pediatric patients), comorbidity, length of surgical procedure, cachexia, fluid shifts, cold irrigating fluids, and general and regional anesthesia are all considered as risk factors for hypothermia.

List the possible complications of spinal anesthesia. Complications of spinal anesthesia include: hypotension, total spinal anesthesia, positioning problems, and postdural puncture headache.

List the disadvantages of single-use laparoscopic instruments. Disadvantages of single-use instrumentation include the need for increased storage space, budgetary implications, and environmental concerns related to disposal of biohazardous waste.

List the three characteristics that differentiate laser light from ordinary light. Laser light differs from ordinary light because it is monochromatic, collimated, and coherent.

A breast biopsy is an example of what class of wounds? A breast biopsy is an example of a class I Clean Wound.

List the three types of wound healing. The three types of wound healing are: (1) primary intention, (2) secondary intention, and (3) delayed primary closure (tertiary intention).

Lesions in the left temporal lobe would present with

Loss of comprehension and aphasia (loss of speech)

What are the signs of breast cancer?

Mass (1 cm usually the smallest that can be palpated) dimple nipple rash edema axillary/supraclavicular LAD

This is an absorbable suture, monofilament, with polyglyconate, same as vicryl and PDS, with a complete absorbtion 180-210 days, more supple, degraded by hydrolysis, maybe a the absorbable suture of choice

Maxon

this is synthetic non-absorbale suture, composed of polyester fiber, multifilament, the first synthetic braided to last indefinetly in the body, it provides precise, consistent suture tension, with minimal breakage, uncoated

Mersilene- Last for ever

Study was conducted that compared laparoscopic and open-mesh methods of inguinal hernia repair and also compared TEP and TAPP techniques for cost and patient outcome effectiveness. Some of the factors measured included patients return to prior activities, persisting pain and numbness, infection, length of operation time, and complications. The researchers discovered several compelling findings and many differences between the two approaches. Select two statements that reflect true findings from the study.

Mesh infection rate was low with both techniques.

What incision is used for lumbar laminectomy?

Midline directly over the appropriate lumbar spines

What is relative contraindicated in Heparin Prophylaxis

Mild to moderate bleeding diathsesis or platelet count between 20-100,000,

What are the two chief goals of anesthetic management for a AAA? (broadly speaking)

Minimize Patient Morbidity Maximize Surgical Benefit

What is the role of axillary node dissection with DCIS?

No role in true DCIS (without microscopic invasion), some perform a sentinel node dissection for high-grade DCIS

What are possible Sxs for breast CA?

No symptoms, or... mass, pain (most are painless), nipple discharge, local edema, nipple retraction, dimple, nipple rash

Is the fluid from a breast cyst sent for cytology?

Not routinely, bloody fluid should be sent for cytology

The blood supply is introduced to the medullary canal through what structure?

Nutrient foramen

Dirty or infected wound

Old, physically induced wounds with retained devitalized tissue with existing infection like an abscess or delayed primary closure from a ruptured appendix.

What do you do with a mammatome biopsy that returns as atypical hyperplasia?

Open needle loc Bx as many will have DCIS or invasive CA

CHAPTER 6

POSITIONING THE PATIENT FOR SURGERY

What structures form the vertebral arch?

Pedicles and laminae

What is commonly used to retract the spermatic cord to repair defects in the floor and internal ring of the inguinal canal?

Penrose drain

The branches of the deep femoral artery that travel posterior to the femur and supply all three fascial compartments are the

Perforators

The soft outer layer of the bone provides blood supply to the cortex and is called the

Periosteum

Which of the following would affect normal wound healing? A. Penicillin allergy B. Peripheral vascular disease C. Clean wound D. Latex allergy

Peripheral vascular diseaser

Describe a strangulated hernia

Persistent incarceration with a compromised blood supply

Which of the following sutures should not be used in the placement of heart valves? A. Polypropylene B. Polyglactin C. Polydioxanone D. Polyglecaprone

Polydioxanone

10 Basic Concepts of Assisting

Pre-Practice All Techniques No Wasted Moves Anticipate Two Hands/Ten Fingers Assist, Don't Inhibit Think Team Know Your Limitations Practice to Improve Watch & Learn Take Notes

This needle is used in the dermal

Precision needle 5-O

This type of ulcer is also known as a decubitus ulcer

Pressure ulcer

This square shaped muscle is attached to the ischial tuberosity & the quadrate tubercle

Quadratus femoris

What is the attachment of the lesser omentum to the stomach

Right side of abdominal oesophagus down along the lesser curvature to the first 2cm of the duodenum

What nodes are located between the pectoralis major and minor muscles?

Rotter's nodes • not usually removed unless they are enlarged or feel suspicious intraoperatively

A type B aneurysm is also known as what?

Sacular or Berry

What are the common options for breast reconstruction after a mastectomy?

Saline implant TRAM flap

What is the name for the small pancreatic duct?

Santorini

What is the collagenase that we use as an enzymatic debriding agent?

Santyl

Describe a facet

Smooth articular surface of an articular process

what structures are located in the inguinal canal?

Spermatic cord/round ligament

The most dorsal portion of the vertebrae is the

Spinous process

What stage of breast cancer? Tumor < or equal to 2 cm in diameter with mobile axillary nodes or tumor 2-5 cm in diameter, no nodes

Stage II A

What stage of breast cancer? Tumor 2-5 cm in diabeter with mobile axillary nodes or tumor >5 cm with no nodes

Stage II B

This SUTURE has the greatest relative coefficient of friction of non absorbable sutures

Stainless steel

This suture has the greatest relative tensile strength of nonabsorbable sutures (TQ)

Stainless steel

Once DVT/PE is confirmed and initial heparin IV is done, how is coumadin given?

Start it as soon as long term anticoagulation is planned. takes 3-5 days to reach theraputic levels, monitor using PT, and Maintain at 1-1.5x baseline

How is the infracolic compartment divided?

The infracolic is below the root of the transverse mesocolon is divided by the root of the small bowel mesentery

What is the lesser sac?

The lesser sac is a diverticulum of the greater sac It is located posterior to the stomach and is entered through the Epiploic Foramen of Winslow.

A linear dimensional flap (square, rectangle) rotated about a pivot point

Transpositional flap

This is a lever type process that projects laterally from the junction of the pedicles and the laminae.

Transverse process

The node of Lund is located in the

Triangle of Calot

A cross section of the mid-shaft of the femur would appear

Triangular

What is the # 5 cranial nerve and what does it innervate?

Trigeminal, face

Which cranial nerves innervate the superior oblique muscle?

Trochlear (IV)

The cortical surface of the brain cannot be suctioned unless there is a cottonoid placed under the suction tip.

True

What type of endoleak is when blood flows into the aneurysm sac due to the porosity of the graft fabric, causing blood to pass through from the graft and into the aneurysm sac?

Type IV endoleak

The top half of the kidney is called:

Upper pole

What are the anatomical landmarks?

Ureter, godadal vessels

Which cranial nerve emerges from the medulla, passes through the skull, and descends through the neck region into the thorax and abdominal region?

Vagus (X)

What is the #10 cranial nerve and what does it innervate?

Vagus, organs of the thoracic and abdominal cavities

What muscle is located between the vastus lateralis and the lateral femoral shaft?

Vastus intermedius

This ulcer present most of the time on the medial side of the ankle, due to poor venous return, affects 3-5% of patients > 65 years, what will be the recommended treatment

Venous compression,

This type of ulcer is a result of incompetent venous return, most often near medial ankle, and in pts over 65

Venous insufficiency ulcer

This suture material is coated with CALCIUM STEARATE (coating lubricant),or with polyglactin 370

Vicryl (Victory you are done)- good for SubQ closure

Major factor leading to DVT

Virchow's triad

List two common interventions after the aorta cross-clamp is removed.

Volume loading (will happen almost w/o exception) Calcium 500mg - 1gm

CHAPTER 9

WOUND HEALING, DRESSINGS AND DRAINS

What grade would an ulcer (neurotrophic) which extends into the subQ tissue be graded as using wagner classification? Knighton?

Wagner-Grade 2; Knighton-II

What are the goals during emergence and extubation of a AAA repair?

Want smooth and hemodynamically stable emergence Temperature is critical Individualized: Preexisting medical condition

What are the types of mechanical debridement?

Wet to dry, Whirlpool, and wound irrigation

In this non selective form of mechanical debridement, healthy tissue is removed with dead tissue, is harmful, delays healing, is painful and labor intensive, and evaporation can occur to decrease the temp of the wound.

Wet-to-Dry

List agents identified by the Centers for Disease Control and Prevention (CDC) that could pose a threat to national and world security and safety through bioterrorism. The CDC has identified anthrax, smallpox, plague, botulism, and tularemia as bioterrorism threats

What is the human prion disease that is a fatal neurodegenerative disease of the central nervous system? Creutzfeldt-Jakob disease (CJD) is a fatal neurodegenerative disease of the central nervous system

In this multi-step process, removal of necrotic and fibrotic tissue takes place, then decreasing bacterail load, then managing exudate.

Wound bed preparation; continue debridement to slightly beyond the point of visible necrotic tissue

The most common cause of delayed would healing in the operative patient is: A. Would infection B. Tissue loss C. Dehiscence D. Adhesions

Wound infection

Contaminated wound

Wounds that include traumatic wounds of less than 4 hours duration and those in which operations with major breaks in aseptic technique and gross spillage from the gastrointestinal tract are also included.

Which lasers are used to destroy the lens casule during cataract surgery?

YAG

This consists of 2 triangular random pattern flaps. This plasty creates a lengthening effect in parallel axis and a shortening effect in the perpendicular axis, very useful for linear scar contractures

Z plasty

Immediately follow an arthroscopic assisted ACL reconstruction, a patient's knee is typically placed in

a hinged knee brace

A suture ligature is also referred to as

a stick tie

Indications for laminectomy include:

a. Compression fracture Incorrect b. Spinal cord stimulation c. Herniated nucleus pulposus

Personnel radiation monitoring is an important part of radiation exposure safety. Select the statement that best describes the proper placement of the personal radiation dosimeter.

a. On the trunk under the apron and at the collar level outside the apron

PMMA (bone cement) may affect local tissues by

a. causing tissue coagulation b. bone necrosisc c. cytoxic and lipotoxic effects

All of the following may be associated with a "heart block" except:

a. section of the superior part of the thoracic sympathetic trunk.

which of the following is not typically visable in a PA chest radiograph

a. the interventricular septum

Where is the abdominal aorta located?

after the aortic hiatus in the diaphragm, at the roof of the abdominal cavity slightly on the left

Traffic patterns in the surgery department are designed to

aid in preventing surgical site infections

The reactions ototoxity and nephrotoxity can be a result of administering

aminoglycosides

What may be requested during an emergency femoral embolectomy?

arteriograms

In what order should clamps be added?

artery first, then vein

Cricoid pressure is applied to prevent

aspiration

Which nerves are most frequently injured due to a shoulder dislocation?

axillary

Baby Boy Doe is a newborn found outside of the emergency department (ED). He is hypothermic and has several deep lacerations across his back, abdomen, and legs that will require surgical repair. He is transferred to the surgical bay of the ED and prepared for anesthesia induction. What risk reduction strategies are appropriate during the preanesthetic phase to achieve and maintain normothermia?

b. Provide a radiant heat lamp during placement of monitoring lines and induction

In the following illustration, the arrows points to a line that represents the:

b. separation between the superior mediastinum and the inferior mediastinum

Which ligament connects the thryoid to the trachea?

berry

The white line of Hilton is situated

between the external and internal anal sphincters

Positive assurance that sterilization conditions have been achieved can only be obtained through

biological indicators

Intraoperative fluid and electroyte imbalance are affected by

blood loss, fluid shifts and replacements

Serosanguineous drainage means

blood-tinged

What branches are in the visceral branch of the thoracic aorta?

broncho-esophagial artery

Select the true statement about antimetabolite drugs and their indications for use.

c. 5-Fluorouracil and mitomycin can both be administered topically to dissolve scars and pterygia.

An abdominal perineal resection, or APR, for a patient at high risk for colon cancer without anal/rectal involvement (e.g., familial adenopolyposis [FAP]) can be accomplished through an open laparotomy or laparoscopic-assisted ileoanal pull-through approach, per surgeon preference and appropriate patient selection. Which of these statements about approaches for APR is correct?

c. Both procedures require only an abdominal skin incision(s) as the rectal segment is removed and anastomosed intraluminally

A percutaneous method of instituting femoral vein-femoral artery CPB can be used for minimally invasive (or conventional open) procedures and in emergency situations where the environment is not conducive to traditional CPB methods. Select all the areas where percutaneous CPB would be indicated.

c. Cardiac catheterization unit and intensive care unit

Identify the dura fold that separates the right and left cerebral hemispheres.

c. Falx cerebelli

Select the alternative or complementary therapy that would be appropriate to integrate into the plan of care in the perioperative setting.

c. Guided imagery before and during anesthesia induction

The transverse colon meets the ascending colon at what point?

c. Hepatic flexure

Which of the following is incorrect for the cricoid cartilage?

c. It is an attachment of the vocal ligaments

Select the name of the procedure that the patient in the picture is positioned for

c. Stereotactic brain biopsy

recommended long-term pain management in the elderly includes

c. opoid analgesics (low potential for addiction)

Which of the following will the perioperative nurse be sure to have when setting up for a mediastinoscopy?

c. suture and needle holders

This tissue contains fat cells.

c. yellow marrow

In which procedures will the patient be placed in supine position with the right side slightly elevated by a wedge?

caesarean section

An emergency drug that increases myocardial contractility is

calcium chloride

An emergency drug used in cardiac surgery that increases myocardial contractility is

calcium chloride

Homan's sign is:

calf discomfort elicited by ankle dorsiflexion with knee extended

Identify the stage of bone healing that is bypassed when compression plate fixation is employed.

callous

A chemical reaction that provides energy by the breakdown of food molecules is called

catabolism

What parts of the body does the abdominal aorta supply with blood?

caudal parts of the body (diaphragm, abdominal wall and viscera, pelvic wall and viscera, hind-limbs)

What does the caudal mesenteric artery supply?

caudal portion of the large intestine. descending colon and rectum.

What accompanies the abdominal aorta in the thoracic cavity?

caudal vena cava

Jackson's membrane would be found near the

cecum

The condition of imperforate anus includes all of the following EXCEPT: *stenosis *agenesis *membrane atresia *chalasia

chalasia

What antibiotics should not be given as a preoperative prophylaxis to a pediatric patient?

ciprofloxacin

In patients who develop fever, abdominal pain, and profuse diarrhea, the MOST frequently seen antibiotic-associated enterocolitis is due to which organism?

clostridium difficile

Which of the following is the desired effect when applying a partially threaded cancellous screw to a bone fragment A. distraction B. rotation C. compression D. subluxation

compression

Heat loss from the body occurs by all the following methods except

condensation

Hirschsprung's disease is synonymous with

congenital aganglionosis

An adult male is injured in an automobile accident at 2 AM. An exploratory laparotomy for possible perforated bowel is initiated at 5AM. Which of the following is the best wound classification in this scenario?

contaminated

A major concern in draining a gangrenous gallbladder is

contamination

Which incisions allow for optimal exposure of frontal sinus fractures?

coronal

What is the biggest disadvantage to endovascular approach to AAA repair?

cost ($9,000+)

Why will the perioperative nurse ensure availability of hemodynamic monitoring equipment and supplies for the patient having thoracic surgery?

d. All of the above are true.

Neuropathologic conditions requiring surgical intervention can be found in any age group. What are the most common problems requiring neurosurgical procedures in infants and children?

d. All of the options are correct.

What special consideration should be made when assessing a geriatric trauma patient before surgery?

d. All of the options can negatively impact the geriatric patient's perioperative experience

What therapy is guided by the principle that dysfunction of one part of the body often affects secondarily the function of other discrete, possibly indirectly connected body parts?

d. Manual healing methods

Which of the following is incorrect pertaining to the hamstrings?

d. One of them is partially innervated by the common fibular division of the sciatic nerve.

A variety of implantable devices are used in aesthetic and reconstructive plastic surgery procedures. Tracking of these devices is critical to patient safety. Institutions must report any incident of death or serious injury relating to the use of a medical device. What regulatory agency, commission, or act mandates this process when medical devices fail or cause injury?

d. Safe Medical Device Act

while not as prevalent as arthoplasty of the shoulder, knee or hip, total elbow replacement is indicated for patients with traumatic lesions or excessive bone loss from rheumatic or degenerative arthritis. It may be used with or without pmma depending on the quality of the

d. Side plates, screws, and circumferential wires

What type of drainage is placed in the common bile duct after a CBDE is performed?

d. T-tube

A cross section of the mid-shaft of the femur would appear _____.

d. Triangular

What patient position is used for reduction mammoplasty?

d. both modified fowlers and supine

Following an argument with his wife in which a man backed through a glass storm door resulting in deep laceration of his buttocks, he is having great difficulty walking. To better understand the nature of his disability, you ask him to stand on one foot. By doing so, you are doing all of the following except:

d. evaluating the function of the gluteus maximus.

Removal of devitalized tissue is

debridement

A wound that is left open and closed secondarily when there is no longer evidence of infection is an example of

delayed closure

A wound that is known to be infected at the time of surgery and is left open to be closed at a later date is categorized as

delayed primary closure

The bladder is supported by the rectovesical fascia called

denonvilliers

The veins of sappey drain the

diaphragm

A microscopic blood count that estimates the percentages of each type of leukocyte within a sample is called a

differential

Which type of hernias occur in Hesselbach's triangle?

direct

What branches are in the parietal branches of the thoracic aorta?

dorsal intercostal arteries and dorsal costoabdominal artery

The three protective tissue layers called the meninges lie in the following order, starting with the most superficial:

duramater, arachnoid, pia mater (DAP)

Which of the following is incorrect pertaining to the umbilicus?

e. The umbilicus is in the L1 dermatome.

Which of the following is not correct pertaining to the cavernous sinus?

e. contains the mandibular division of the trigeminal nerve

which of the following associations is correct

e. pancreatic cancer-rarely metastasizes

The lithotomy position does not require

each leg raised slowly and gently as it is grasped by the toes

What is done in wound conversion of tx of neurotrophic ulcer?

eliminate necrotic tissue to get granular base by debridement, cleansing, and bacterial control.

In which procedure would a Fogarty catheter be utilized?

embolectomy

The doctrine of respondeat superior refers to

employer liability for employee's negligent conduct

Would a patient with a supra-renal AAA would be a better candidate for a endovascular approach or an open / traditional AAA repair?

endovascular

What are the terminal branches of the aorta?

external iliac arteries, internal iliac arteries, median sacral artery

Endoscopic knot tying performed externally, introduced, and secured intra-abdominally is

extracorporeal

The fold of dura mater that seperates the cerebral hemispheres

falx cerebri

What is teh MC cause of breast mass after breast trauma?

fat necrosis

Collagen is produced by which connective tissue cells?

fibroblasts

What is the MC cause of green, straw-colored, or brown nipple discharge?

fibrocystic disease

What is the DDx of breast cancer / mass?

fibrocystic dz of breast fibroadenoma intraductal papilloma duct ectasia fat necrosis absccess radial scar simple cyst

A common autologous donor site for a free flap graft for replacing the jaw bone is the

fibula

If the veres needle is not positioned inside the peritoneal cavity, what happens to the flow of the CO2 and ressure?

flow decreases and pressure increases

During a carotic endarterectomy, the arteries are unclamped and reclamped before arteriotomy closure is performed to

flush free debris from the carotid

Where do the mental nerves and vessels exit from the mandible?

foramen

Which site is used for autotransplanting one or more of the parathyroid glands?

forearm

A patient is being evaluated for undergoing a breast reconstruction procedure and indicates she still smokes. Which procedures would the surgical assistant recommend?

free TRAM flap

What is the flow of venous return?

from parts of the body to the heart

Fissure Sylvius separates the

frontal and temporal lobe

When an immediate confirmation of malignancy is needed, the specimen will be sent to pathology for

frozen section

What organism is a eucaryotic?

fungi

Where is Hartmann's pouch located?

gallbladder infundibulum

Dumping syndrome is a complication of

gastric bypass

In order to preserve the hepatic artery during liver and pancreas procurement, which branch artery must be identified and ligated?

gastroduodenal

The most appropriate devices to prevent skin pressure ulcers and deep tissue injury in the OR are

gel pads

During a laparoscopic hernia repair, staples should not be placed below the lateral iliopubic tract in order to avoid injuring the

genitofemoral and lateral femoral cutaneous nerves

The fascia surrounding the kidney is called

gerotas

The narrow rim of fibrocartilage around the edge of the glenoid cavity is

glenoid labrum

Which of the following is an example of a high level disinfection agent? A. glutaraldehyde B. isopropyl alcohol C. quaternary ammonium D. iodophor solution

glutaraldehyde

A vulsellum forceps is used in ___ surgery.

gynecologic

In cataract surgery, a viscous jelly sometimes used to occupy space is

healon

When the myocardium weakens and enlarges, losing the ability to pump blood into the systemic circulation, the resulting condition is called

heart failure

Valves of the gallbladder are called

heister

The volume of red blood cells in whole blood is determined by which of the following tests?

hematocrit

All of the following statements concerning hematocrit are true EXCEPT: *normal range for women is 37-43% *hematocrit value is the volume of rbc expressed as the number of cells per cubic mm *a decrease in hematocrit indicates a decreases in red blood cells *hematocrit readings are unreliable after blood transfusion adminstration

hematocrit value is the volume of rbc expressed as the number of cells per cubic mm

Mechanical, thermal, and chemical are appropriate mechanisms to achieve

hemostasis

A thoracoabdominal incision should be used for

hiatal hernia repair

Chvostek's sign can indicate

hypocalcemia

Vitamin D deficiency can lead to

hypocalcemia

Which cranial nerves are at risk for injury during acarotid endarterectomy?

hypoglassal

What is the #12 cranial nerve and what does it innervate?

hypoglossal, tongue

A hernia through which bowel or omentum extends and cannot be returned to its normal position through manipulation is referred to as a/an

incarcerated hernia

What is the MOST likely cause for varicose veins?

incompetent valves

Which artery divides into the anterior and middle cerebral arteries?

internal carotid

What is the MC cause of bloody nipple discharge in a young woman?

intraductal papilloma

Which organ is completely retorperitoneal?

kidney

Which surgical maneuvers refers to the dissection of the lateral peritoneal attachments of the duodenum to allow inspection of the duodenum, pancreas, and other retroperitoneal structures?

kocher

What nerve courses along the lateral chest wall in the midaxillary line, on the serratus anterior muscle? What does it innervate?

long thoracic nerve innervates serratus anterior muscle

A long-term complication of a cemented total hip arthroplasty is

loosening of the prosthesis

When performing a radical cystectomy with ileal conduit, which is an alternative patient position that a surgeon may prefer?

low lithotomy

Identify the soft osseous material filling the cavities of the cancellous bone.

marrow

A cholesteatoma is a benign tumor that develops in the

mastoid bone

Which of the following arteries is a direct branch of the internal carotid?

middle cerebral

Epidural hematoma is typically caused by trauma to what artery?

middle meningeal

The valve that prevents the retrograde flow of blood into the atrium from the left ventricle is the

mitral valve

Which breast procedure removes the entire breast and axillary contents but preserves the pectoral muscles?

modified radical mastectomy

when cathing a female patient the periop nurse must locate the urethra.

mons pubis, labia majora, labia minora, clitoris, urethra, vaginal opening.

A hemodynmaic complicatino in the patient could be caused by

myocardial ischemia

Chronic otitis media can lead to loss of hearing. In the early stages, the patient can be helped by

myringotomy

The term epistaxis means

nasal bleeding

Which nerves are dissected from the lesser curvature of the stomach when performing a highly sensitive selective vagotomy (parietal cell vagotomy)?

nerves of latarjet

Wound drains may be secured with

nonabsorbable suture on a cutting needle

The center structure of the intervertebral disc, which may herniate into the spinal nerve is the

nucleus pulposus

What can be a cause of locking of the knee joint?

osteochondral defects

The surgical assistant focuses on the ___________ circle

outer

What do the deep circumflex iliac arteries supply?

outer pelvis and abdominal wall

Which structures may be used for replacement of the ligament during an ACL repair?

patellar tendon

Comparatively, which of the following patients is at greatest risk for anesthesia complication during the operative phase? A. a patient who is undernourished B. a patient who is immunosuppressed C. a patient with COPD D. a patient with diabetes

patient with COPD

An absorbable suture that offers extended wound support

pds

What is the preferred procedure for recurrent or persistent carcinoma of the cervix after radiation therapy has been completed?

pelvic exenteration

It is recommended that the square knot is

performed as a two handed tie

The soft outer layer of the bone provided blood supply to the cortex and is called the

periosteum

When do most myocardial infarctions occur in the AAA repair population?

post-operative

The linea aspera is a ridge of bone located on the

posterior aspect of the femoral shaft

What is the best predictor of postop renal failure?

preop creatinine level

What is an effective way to estimate proper femoral nail diameter and length of an acute, severely comminuted femur fracture?

preoperative radiographs of the unaffected leg.

Platelet concentration is often transfused during the immediate preoperative period for the purpose of

preserving clotting ability

The purpose of hemostasis is to

prevent blood loss

Wound classification is assigned when

procedure is complete

Which hormone is mainly responsible for breast milk production?

prolactin

What must be ruled out with spontaneous galactorrhea (+/- amenorrhea)?

prolactinoma (check pregnancy test and prolactin level)

Deoxygenated blood is carried from the right ventricle to the lungs via which structure?

pulmonary arteries

Which is not a complication of a transurethral prostatectomy?

rectal inury

Another name for the folds on the inner wall of the stomach is

rugae

The force of integumentary injuries that results from the skin remaining stationary while the underlying tissues shift is known as

shearing

During a low anterior colon resection, the anvil of the EEA is inserted into the

sigmoid

The MOST common structure involved in a sliding left indirect hernia is the

sigmoid

The inner lining of the mucosa within the g.i. tract is composed of

simple columnar epithelium

At the base of each internal carotid artery is an enlargement called the

sinus

A left subcostal incision indicates surgery of the

spleen

What is used to reinforce a subcuticular closure?

steri-strips

Adipose tissue lies in which skin layer?

subcutaneous

Which tendone is located under the clavicle and is the site of most rotator cuff repairs?

subscapularis

What is the major risk with DCIS?

subsequent development of infiltrating ductal carcinoma in the same breast

When performing a high ligation and stripping of the great saphenous vein, which arteries are located at the saphenofemoral junction and should be kept from injury

superficial external pudendal

Which nerves can be injured during a lateral lower leg fasciotomy?

superficial peroneal

Which rectus muscle moves the eyeball upward and medially?

superior

The distal tibia articulates with which structures?

talus

When performing a tympanoplasty, the graft is secured to the oval window when

the footplate of the stapes cannot move

What do veins usually company?

the major arteries

Franklin hardy with mulitple moles skin lesions

the perioperative nurse recognized his signs and symptoms of bupivicaine and lidocaine toxicity and began immediate treatment with airway support.

Which nerve innervates the latissimus dorsi muscle?

thoracodorsal

What nerve courses lateral to the long thoracic nerve, on the latissimus dorsi muscle? What does it innervate?

thoracodorsal nerve innervates latissimus dorsi muscle

Which dressing is used when moderate to heavy fluid drainage is expected?

three layer

Why might a surgeon choose to put he end screws of the plate thorugh only one cortex?

to decrease stress concentration on the bone at the ends of the plate

What is the goal of debridement?

to get a 100% granular base

This test for DVT identifies thrombi by directly visualizing the venous lining and by demonstrating abnormal vein compressibility or, with doppler flow studies, impaired venous flow

ultrasonography

The donor left kidney is preferred in order to procure additional length of the

vein

Triangulation of focus is

when the intended object is not visible in midline position

The most common cause of delayed wound healing in the operative patient is

wound infection

This tissue contains fat cells.

yellow marrow

Describe the attachment of the small bowel mesentery

~15 cm long with the greatest depth at the central part at about 20 cm deep Begins on the left at the d-j flexure passes downward from left to right at 45d crosses the third part of the duodenum, where SMA vessels enter between its two layers continues downward across the aorta, ivc, right psoas muscle and ureter into the right iliac fossa

Ulnar Nerve: Injury Prevention

*Avoid pressure against OR table *Avoid pressure on ulnar groove *Position arm in pronated rather than supinated position (controversial) *Avoid flexing arm more than 110° *Education of care givers in the postop area?

Physiologic Changes in Lithotomy Position: Pulmonary

*Decreased Vital Capacity (Decrease in all lung volumes *Increased Peak Inspiratory Pressure especially in the obese

Femoral Nerve: Signs/Symptoms

*Decreased or absent knee jerk reflexes *Loss of flexion of the hip *Loss of extension of the knee *Sensation is decreased over the superior aspect of the thigh and medial and anteriomedial side of leg

Lateral Decubitus: LE Positioning *which leg stays straight?

*Dependent thigh flexed helps stabilize patient *Nondependent leg remains straight *Pillow between knees *Pressure points -Iliac crest -Greater trochanter

Sitting Position: Complications

*Stroke *VAE *Extubation

*? is the most common position for surgery? *Head is in the ? position *? nerve is the most commonly injured nerve here

*Supine, aka: dorsal recumbent position *Head should be maintained in neutral position *Ulnar nerve most commonly injured in this position

Positioning for Anesthesia and Surgery

*Take responsibility *It's in our Standard *Meticulous positioning, padding, ongoing monitoring *Thorough documentation

What are the history risk factors for breast cancer?

"NAACP" Nulliparity Age at menarche (younger than 13) Age at menopause (older than 55) Cancer of the breast (in self or family) Pregnancy with first child (older than 30)

What is the Tx for local reccurrence in breast after lumpectomy and radiation?

"salvage" mastectomy (axillary dissection already completed)

Physiologic Changes in Lateral Position: Cardiovascular

* Minimal changes, hypotension with extreme flexion which reduces venous return *Be aware that dependent (lower) arm will read higher BPs than non-dependent (upper) arm

Radial Nerve: Injury Prevention

*Avoid compression against the posterior and lateral aspect of humerus

The normal adult range of WBC is

5,000 - 10,000/mm3

Postop, skin staples are generally removed

5-7 days

In the diagram, identify the following structures of the biliary tract: Common bile duct; Ampulla of Vater; Intrahepatic ducts; Right hepatic duct; Left hepatic duct; Gallbladder; Cystic duct; Common hepatic duct

1. Intrahepatic ducts.... .......2. Left hepatic duct 3. Right hepatic duct...........4. Common hepatic duct 5. Gallbladder......................6. Cystic duct 7. Common bile duct......... 8. Ampulla of Vater

Which of the following are requirements for a sharps container? 1. self-closing lid to prevent overfilling 2. biohazard labels and color coding 3. durable, closable, leakproof, and puncture resistant 4. lids which allow sharps to enter by gravity

2, 3, and 4 only

A patient with BP of 120/76 what should the ankle pressure be?

250mmHg

What is the amount of time that heparin is systemically distributed in the adult?

3 minutes

Cleft lip repair is usually recommended at

3 months

An aneurysm with a diameter of ___ cm or greater dramatically increases the risk of rupture.

5 cm

The Anterior Spinal Artery provides what % of spinal cord blood flow and supplies what proportion of the spinal cord?

75% of blood flow 2/3 anterior portion of S.C. Supplies motor tracts Formed by two branches off of the vertebral arteries

What are the indications for modified radical postmastectomy radiation therapy?

> 4 lymph nodes, plus chest wall involvement by tumor

All of the following are true of the mediastinum except:

?b. it contains the lungs

Drilling through the femoral shaft correctly is evidenced by what kind of feedback?

A distinct tactile feeling that the drill bit is contacting hard lateral cortex, soft medullary canal, hard medial cortex, and finally the soft tissue of the medial thigh

Q. In terms of coagulation test results, which of the following is within normal range for partial thromboplastin time?

A. 25 sec Explanation: This is within normal range. The partial thromboplastin time has a range of 21-35 seconds.

An emergency drug that increases myocardial contractility is: A. Calcium chloride B. Levophed C. Lasix D. Isuprel

A. Calcium chloride

Q. Which of the following risk factors can impair the integrity of the patient's skin resulting in an adverse outcome?

A. Pooling of prepping solutions Explanation: Patients can experience irritation, an allergic reaction or the skin may breakdown from prepping agents that have pooled underneath them. It is important that during prepping towels be placed under the patient and that all solutions are cleaned promptly.

Q. When a patient is having a surgical procedure, which of the following would be the typical length of time that pressure injuries can become more common?

A. 1 hour Explanation: Pressure injuries most commonly occur in surgery after an hour or longer. If a surgery is a long one, the patient should be repositioned if at all possible to help prevent pressure injuries.

When drawing a blood sample, a safe time lapse between blood drawing and analysis is: A. 10 minutes B. 20 minutes C. 30 minutes D. 1 hour

A. 10 minutes

Q. The contemplation stage can last how long?

A. 2 Years. Explanation: The contemplation stage can sometimes last for 2 years. Patients may procrastinate or be ambivalent about the change.

Normal adult male range for RBC is: A. 4.2-6.1 mill/ul B. 2.8-4.2 mill/ul C. 1.4-2.8 mill/ul D. 0.4-1.4mill/ul

A. 4.2-6.1 mill/ul

The normal adult range for WBC is: A. 5,000-10,000/mm^3 B. 10,000-15,000/mm^3 C. 15,000-20,000/mm^3 D. 20,000-25,000/mm^3

A. 5,000-10,000/mm^3

Q. The maintenance stage is said to begin when?

A. 6 months after a person ceases a risky behavior Explanation: The maintenance stage is said to begin 6 months after a person ceases a risky behavior. Many people never get beyond maintenance - they end up in a cycle of maintenance and relapse for life.

Q. The largest body cavity which extends from the diaphragm to the lesser pelvic wall would be?

A. Abdominal Cavity Explanation: The abdominal cavity is the largest cavity in the body. This cavity holds the stomach, liver, pancreas, intestines, spleen and kidneys.

Q. When an organ is shaped or positioned incorrectly, the term used to describe this situation would be considered which of the following?

A. Abnormal Anatomy Explanation: This term would be considered abnormal anatomy. The fact that the patient may have an abnormal anatomy is of importance to the surgeon and could cause issues during the surgical procedure.

What is the mechanism of action of heparin: A. Breaks down the clot B. Prevents formation of new clots C. Inhibits Vitamin K synthesis D. Decreases platelet aggregation

A. Breaks down the clot

Q. Instrument equipment that can be used for visualization of the bronchus and trachea would be considered

A. Bronchoscope Explanation: A gastroscope would be used to view the stomach.A colonoscope would be used to view the entire large intestines. An arthroscope would be used to view the intra-articular surfaces such as the knee. A bronchoscope would be used to view the bronchus and trachea.

Q. Patient care should reflect ongoing management. An adverse outcome related to preparing the patient for a procedure would include:

A. Changes in the patient's health status are not identified and reflected in the patient care plan before procedures Explanation: Nurses failing to reflect changes in the patient care plan before a procedure is an adverse preparation or planning related outcome and nursing goal deficit . The other answer choices reflect adverse outcomes that may occur during the implementation/intervention nor planning phase of treatment.

Q. A family which has difficulty budgeting, providing proper nutrition, and otherwise meeting the basic needs of its members is known as which of the following?

A. Chaotic Explanation: A chaotic family may have financial difficulties, which can lead to difficulties in providing for basic needs. This may not be intentional, but the result of instability due to unemployment and other factors. The patient may see the nurse as a much-needed source of stability and thus want to depend on him or her. When working with a chaotic family, establish trust and work to help the family get basic needs met.

Which of the following types of patients is at risk for developing a latex sensitivity? A. Child with spina bifida B. Patient with multiple sclerosis C. Patient with a shellfish allergy D. Child with Hirshsprung's disease

A. Child with spina bifida

Q. A patient voices his concerns to the nurse about being able to maintain a recommend exercise program and diet. The nurse finds community resources to help the patient post-discharge. This nurse shows an example of which counseling skill?

A. Facilitation Explanation: Counseling skills are important to helping patients make behavioral changes. In this case the nurse best exemplifies facilitation because she directs the patient to resources necessary to make the recommended lifestyle changes.

The frontal lobe is bounded posteriorly by the central sulcus, also called the: A. Fissure of Rolando B. Fissure of Sylvius C. Tentorium cerebelli D. Transverse fissure

A. Fissure of Rolando

Which of the following separates the frontal and the temporal lobes? A. Fissure of Sylvius B. Fissure of Rolando C. Fissure of Bichat D. Longitudinal Fissure

A. Fissure of Sylvius

The fold of dura mater that separates the cerebral hemispheres: A. Flax cerebri B. Tentorium cerebelli C. Central sulcus D. Circular sulcus

A. Flax cerebri

Where do the mental nerves and vessels exit from the mandible? A. Foramen B. Fontanel C. Fossa D. Fissure

A. Foramen

Which of the following is a measurement of the amount of bicarbonate in arterial blood? A. HCO3 B. pH C. PO4 D. COHb

A. HCO3

Q. Actions undertaken by an individual to promote health and well being or to detect disease when it is early or asymptomatic are known as which of the following?

A. Health behaviors. Explanation: Health behaviors are behaviors undertaken by an individual before illness presents itself. Health behaviors are focused on promoting health and well being, as well as on catching disease in the early stages.

Q. Behaviors that are motivated by a desire to prevent disease, catch disease early, or return to optimum function even with disease are known as which of the following?

A. Health protection Explanation: Health protection is a set of behaviors designed to prevent disease, catch disease early, or return to optimum functioning even with disease. This is sometimes also known as disease prevention

Q. The hardest working organ in the human body even though it's only the size of a closed fist would be which of the following?

A. Heart Explanation: Even though it is fairly small, the heart is the hardest working organ in the human body, the heart is solely responsible for pumping the blood through the body's circulatory system.

Q. Use of gluteraldehyde would be an example of which level of disinfection?

A. High level Explanation: Housekeeping disinfectants and disinfectants used for surgical instruments are not interchangeable because of the level of disinfection. Gluteraldehyde is considered a high level disinfectant that is also capable of sterilizing items.

Significantly decreased volume of circulating blood or plasma in the body may cause the patient to go into: A. Hypovolemic shock B. Anaphylactic shock C. Metabolic crisis D. Malignant hyperthermia

A. Hypovolemic shock

The recurrent laryngeal nerve that supplies the vocal cords lies very close to the: A. Inferior thyroid artery B. Brachial artery C. External carotid D. Subclavian artery

A. Inferior thyroid artery

Q. Of the following, which would be considered types of hemostats?

A. Kelly and Lambott Explanation: A Lambott is considered an elevator. The common hemostats would be: Kelly Schnidt Mosquito Crile Mayo Right angle Mixter

The rectus sheaths that meet to form a tough fibrous white band that extends from the xiphoid process to the pubic symphysis is the: A. Linea alba B. Serratus anterior C. External oblique D. Rectus abdominis

A. Linea alba

Q. A recovering patient developed a rash on her wrist from the latex (gloves). Everytime a nurse touched the patient's wrist, it irritated the patient's skin sensitivity, and eventually a rash appeared. This patient had an adverse outcome for which care event category?

A. Monitoring and controlling the environment Explanation: A reaction to latex exposure is an adverse outcome associated with monitoring and controlling the environment. It is not due to drugs or solutions because the question specified the gloves. Additionally, positioning and handling of cultures and specimens are not proper answer choices.

Q. A patient appears disheveled and recently suffered the death of a spouse. Which assessment would you perform?

A. NGASR Explanation: NGASR The Nurses Global Assessment Risk for Suicide is used to determine a patient's suicide risk. A patient who recently lost a spouse and does not appear concerned with hygiene may have an increased risk for depression and suicide.

The ophthalmic artery divides into the: A. Supraorbital and supratrochlear B. Internal and external carotid C. Middle meningeal and dorsal nasal D. Episcleral and posterior ciliary

A. Supraorbital and supratrochlear

Studies show that the lowest infection rates have been obtained in surgical cases where patient body hair was: A. Not removed B. Removed using a depilatory C. Cut with surgical clippers D. Shaved

A. Not removed

Q. A family unit that consists of two parents and a child is known as which of the following?

A. Nuclear family. Explanation: Families consisting of two parents and children are nuclear families. Assessing family structure helps a nurse better understand the support and barriers present for the patient and his or her care. Careful attention to family dynamics and roles not only helps the nurse to better understand the patient, but also plays a role in planning effective care.

Q. Which of the following is a type of delegation that gives attention to scheduling?

A. Partnering Explanation: Working closely with one another leads to better commitment to one another. Partnering is when a RN and LPN work together on a regular basis. They are scheduled to work with each other on a constant basis. This type of delegation leads to attention to scheduling unlike pairing which does not.

Q. SSI stands for?

A. Surgical site infection Explanation: SSI stands for surgical site infection. SSI is often the result of contamination of the tissues by microorganisms during the intraoperative period.

Q. To provide effective postoperative care, the nurse must be aware of the factors influencing convalescence. Which of the listed factors is not a critical factor in the patient's convalescence?

A. Patient's gender. Explanation: The critical factors influencing convalescence include: preprocedure condition, amount of blood transfused, procedure time/length, and the organ system involved. The patient's gender is not a critical factor influencing convalescence.

A miotic drug is: A. Pilocarpine B. Homatropine C. Atropine D. Scopolamine

A. Pilocarpine

In which of the following circumstances could the patient sustain an injury to the pudendal nerves? A. Positioned on the fracture table B. Placed in lateral chest position C. Placed in lithotomy position D. Positioned on the urological table

A. Positioned on the fracture table

Q. A patient is being positioned for surgery and the genitalia and breast are padded. Which of the following positions would the patient more than likely be in?

A. Prone Explanation: If a patient is placed in a prone or any other position where they are face down, their genitalia and breast should be padded. Other parts that would need padding would include: Dorsum of the foot to protect the toes Anterior knees of kneeling patient Face/forehead/ears

Q. When a person finds him or herself occupying multiple roles and unable to meet all the expectations, he or she is experiencing which of the following?

A. Role overload Explanation: Role overload occurs when a person occupies multiple roles and is unable to meet all the demands. Role overload often occurs in families where there is illness because of the shift in responsibilities and the new responsibilities. Caregivers are especially at risk of role overload.

Q. A _____ is an abnormal heart sound when auscultating a patient?

A. Rub Explanation: Auscultation is an important part of an examination. An abnormal heart sound is a rub. This sound may indicate pericarditis. The other answers are normal sounds during an auscultation.

When a drug loses its effectiveness in a patient, the patient is said to heave developed: A. Tachyphylaxis B. Hypersensitivity C. Hyperactivity D. Antagonist action

A. Tachyphylaxis

Q. What is spinal anesthesia used to block

A. Somatic sensory nerves and motor fibers Explanation: Spinal anesthesia blocks somatic sensory and motor fibers. It often used for surgery on the legs or in the pelvic region.

Q. Which of these surgical dressings could be used in minor surgeries and for minor wounds to approximate small incisions and protect the wound?

A. Steri-Strips Explanation: When applying the Steri-Strips, a small amount of biological adhesive such as benzoin spray can be applied before the strips are put on to help them have extra adhesion. With some incisions, the Steri-Strips can protect the incision line and serves as a dressing.

Q. A desired patient outcome is the patient should be free from infection or injury from retained instruments. A common method used to confirm retention of instruments includes what?

A. Taking an X-ray Explanation: An adverse outcome related to the use of surgical instruments includes retained instruments. The retention of an instrument can be confirmed by an instrument count and taking x-rays.

Q. Nonabsorbable suture is used in what?

A. Tendons Explanation: Nonabsorbable sutures are used in places where additional wound support is required such as in tendons and fascia. It is also often used in cardiovascular and neurosurgery

Q. A health promotion stratey that is focused on returning a patient to optimum health and functioning is known as which of the following?

A. Tertiary prevention Explanation: Tertiary prevention strategies are focused on returning a patient to optimum health and functioning after illness or injury. Rehabilitation, physical therapy, and health counseling are all tertiary prevention strategies.

Q. Of the following, which would be considered correct for how medications would typically be obtained in the surgical suite?

A. The circulating nurse obtains the medications and then delivers them to the scrub technician. Explanation: The circulating nurse who is nonsterile typically obtains the medications, prepares them, and then delivers them to the scrub technician who remains sterile. The sterile scrub technician is the person responsible for passing the medications to the surgeon during the procedure. When the scrub technician passes the medication, they should clearly state the name of the medication and strength to the surgeon

Q. Of the following, which of the following procedures should be done first when arriving in the surgical department each day?

A. The first hand cleansing with a brush scrub Explanation: When a person arrives at the surgical department, the first thing that should be done would be the first hand cleansing with a brush scrub. When a person does a surgical scrubbing they should use a brush, antiseptic soap, and water.

Q. Of the following statements, which would not be correct when setting up for robotic surgery?

A. The robotic cart should be placed at the patient's head. Explanation: The robotic cart should not be placed at the patient's head. If the robotic cart is placed at the patient's head, it could interfere with anesthesia activities and be a safety problem

The avascular area located in the mesentery and to the left of the middle colic artery is: A. The space of Riolan B. Sudeck's point C. Jackson's veils D. Alcock's canal

A. The space of Riolan

Which of the following nerves is located lateral to the long thoracic nerve and innervates the latissimus dorsi muscle? A. Thoracodorsal B. Long thoracic C. Lateral pectoral D. Subscapular

A. Thoracodorsal

Which dressing is used when moderate to heavy fluid drainage is expected? A. Three layer B. Biologic C. Pressure D. Rigid

A. Three layer

Q. Nutritional therapy for a patient with chronic kidney disease includes all of the following except:

A. using protein supplements Explanation: All of the choices are part of the nutritional therapy for a patient with chronic kidney disease except this one. Patients should be taught to avoid high protein diets and protein supplements. They may overstress the diseased kidneys

When to use Homan's test, and when is it specific?

Absent in nearly half of cases when patient ha a DVT, more specific for DVT in presence of edema and local temperature increase

What is the Tx of a breast abscess?

Abx (e.g., dicloxacilln) needle or open drainage with cultures taken resection of involved ducts if recurrent breast pump if breastfeeding

Which of the following methods should the surgical assistant consider when cutting sutures? A. According to the surgeon's preference B. According to the assistant's preference C. Leave a one centimeter tail for nonabsorbable suture D. Cut the knot for absorbable suture

According to the surgeon's preference

The third ventricle of the brain empties into the fourth ventricle through the

Aqueduct of Sylvius

This type of ulcer is usually due to complete or partial blockage resultant in insufficient perfusion to the LE and resultant tissue necrosis. most often near the most distal part of foot

Arterial insufficiency ulcer.

Which of the following are the MOST important considerations when preventing wound infection in the surgical patient? 1. pathogen transmittal 2. portal of entry into the susceptible host 3. anesthetic options for the patient's operative process 4. invasion of the susceptible host

B. 1, 2, and 4 only

Water constitutes what average normal percentage of total body weight? A. 20% to 40% B. 50% to 70% C. 70% to 85% D. 15% to 25%

B. 50% to 70%

The normal pH of blood is: A. 3.5-4.5 B. 7.3-7.4 C. 8.5-9.0 D. 9.5-10.5

B. 7.3-7.4

The thoracic duct begins in the: A. Spinal column B. Abdomen at the Cisterna Chyli C. Base of the skull D. Thorax inferior to the mediastinum

B. Abdomen at the Cisterna Chyli

Excessive secretion of growth hormones in adults causes: A. Splenomegaly B. Acromegaly C. Cardiomegaly D. Hepatomegaly

B. Acromegaly

The reactions ototoxicity and nephrotoxicity can be a result of administering: A. Penicillins B. Aminoglycosides C. Cephalosporins D. Tetracyclines

B. Aminoglycosides

The most appropriate devices to prevent skin pressure ulcers and deep tissue injury in the O.R. are: A. Sand bags B. Gel pads C. Towels D. Sheets

B. Gel pads

The volume of red blood cells in whole blood is determined by which of the following tests? A. Differential count B. Hematocrit C. Hemoglobin D. Bleeding time

B. Hematocrit

All of the following statements concerning hematocrit are true EXCEPT: A. Normal range for women is 37-43% B. Hematocrit value is the volume of red blood cells expressed as the number of cells per cubic millimeter C. A decrease in hematocrit indicates a decrease in red blood cells D. Hematocrit readings are unreliable after blood transfusion administration

B. Hematocrit value is the volume of red blood cells

A lower oblique incision is called a/an: A: Pfannenstiel B. Inguinal C. Paramedian D. Midabdominal

B. Inguinal

Which of the following is not an inhalation anesthetic agent? A. Enflurane (Ethrane (R)) B. Propofol C. Sevoflurane D. Nitrous oxide

B. Propofol

A drug combined with heparin that causes a postoperative loss of anticoagulant activity is: A. Papaverine B. Protamine sulfate C. Tromethamine D. Angiovist

B. Protamine sulfate

Which of the following is the loose areolar tissue that separates the breast from the pectoralis major muscle? A. Subcutaneous tissue B. Retromammary space C. Cooper's ligament D. Superficial fascia

B. Retromammary space

The trigeminal arterial branches of the aorta are the: A. Renal and gonadal B. Right and left common iliacs C. Superior and inferior mesenteric D. Inferior phrenic and lumbar

B. Right and left common iliacs

The anastomosis, or linking, of arcades between the superior and inferior mesenteric vessels is known as the long anastomosis of: A. Roe B. Riolan C. Douglas D. Retzius

B. Riolan

Phagocytosis by the white blood cells is an example of the body's: A. First line of defense B. Second line of defense C. Third line of defense D. Fourth line of defense

B. Second line of defense

The force of integumentary injuries that results from the skin remaining stationary while the underlying tissues shift is known as: A. Pressure B. Shearing C. Friction D. Maceration

B. Shearing

The flexure between the transverse colon and the descending colon is called the: A. Hepatic B. Splenic C. Gastric D. Colic

B. Splenic

The left gastric, right gastric, right gastroepiploic, left gastroepiploic, and short gastric arteries supply the: A. Spleen B. Stomach C. Duodenum D. Kidney

B. Stomach

Which of the following arteries supplies the cecum, ascending colon, hepatic flexure, and proximal portion of the transverse colon? A. Pancreaticoduodenal B. Superior mesenteric C. Abdominal aorta D. Inferior mesenteric

B. Superior mesenteric

If a surgical assistant places the operative patient's well being above all other factors, the the surgical assistant is said to have a strong: A. Ethical sense B. Surgical conscience C. Moral judgment D. Set of values

B. Surgical conscience

Which of the following body systems evokes a physiologic response that is similar to the response evoked by sympathomimetic cardiovascular drugs? A. Endocrine systems B. Sympathetic nervous system C. Lymphatic system D. Parasympathetic nervous system

B. Sympathetic nervous system

The dura plane that separates the cerebrum from the cerebellum is the: A. Roof cerebelli, transverse B. Tentorium cerebelli, transverse C. Transverse cerebelli, sigmoid D. Triangle cerebelli, sagittal

B. Tentorium cerebelli, transverse

Which of the following local anesthetics is used most frequently for preoperative ocular instillation? A. Lidocaine B. Tetracaine C. Bupivacaine D. Mepivacaine

B. Tetracaine

The 2nd most commonly injured periperal nerve is the ?

Brachial Plexus -Possibly related to the long superficial course from the neck under the clavicle to the axilla past the humerus *Most frequently related to stretching *Postop, idiopathic brachial plexus neuritis may mimic intraoperative injury

Why is mammography more useful diagnostic tool in older women than in younger?

Breast tissue undergoes fatty replacement with age, making masses more visible. Younger women have more fibrous tissue, which makes mammograms harder to interpret.

What is the drug of choice when lidocaine fails to achieve response for treating ventricular tachycardia/ ventricular fibrillation during sudden cardiac arrest? A. Esmolol B. Adenosine C. Magnesium sulfate D. Bretylium

Bretylium

The isotonic solution that is closest to human physiology is: A. D5NS B. LR C. 0.9 NS D. 0.45 NS

C. 0.9 NS

The correct range fr the pneumatic tourniquet pressure on an average adult for an upper extremity is: A. 150-200 mmHg B. 200-250 mmHg C. 250-300 mmHg D. 300-350 mmhg

C. 250-300 mmHg

Which of the following cranial nerves innervates the lateral rectus muscle of the eye? A. Trochlear (IV) B. Trigeminal (V) C. Abducens (VI) D. Facial (VII)

C. Abducens (VI)

Drains are typically passed to the exterior through which of the following? A. Superior aspect of the incision B. Inferior aspect of the incision C. Adjacent stab wound incision D. Medial aspect of the incision

C. Adjacent stab wound incision

Some patients may show the most severe and life threatening type of reaction to certain drugs known as: A. Asthma B. Tachyphylaxis C. Anaphylaxis D. Autoimmune disease

C. Anaphylaxis

Which of the following veins drains the right side of the vertebral column? A. Thoracic B. Lumbar C. Azygos D. Posterior intercostal

C. Azygos

The white line of Hilton is situated: A. Between the superior hemorrhoidal artery and middle hemorrhoidal artery B. In the area between the proximal two-thirds and distal third of the transverse colon C. Between the external and internal anal sphincters D. Between the rectum and vagina

C. Between the external and internal anal sphincters

The inner lining of the mucosa of the gastrointestinal tract is composed of: A. Smooth muscle B. Areolar connective tissue C. Epithelium D. Elastic connective tissue

C. Epithelium

The inner lining of the mucosa within the gastrointestinal tract is composed of: A. Smooth striated muscle B. Areolar connective tissue C. Simple columnar epithelium D. Elastic connective tissue

C. Simple columnar epithelium

At the base of each internal carotid artery is an enlargement called the: A. Bulb B. Body C. Sinus D. Plexus

C. Sinus

Lidocaine with epinephrine is contraindicated in surgery of the: A. Shoulder B. Knee C. Hand D. Hip

C. Hand

Vitamin D deficiency can lead to: A. Hyperkalemia B. Hyponatremia C. Hypocalcemia D. Hypermagnesemia

C. Hypocalcemia

The arterial supply to the bladder is derived mainly from the: A. Obturator B. Internal pudendal C. Internal iliac D. Umbilical

C. Internal iliac

The proper placement of the legs for a patient in the lateral position is: A. Both legs straight B. Both legs flexed C. Lower leg flexed, upper leg straight D. Lower leg straight, upper leg flexed

C. Lower leg flexed, upper leg straight

Which of the following arteries is a direct branch of the internal carotid? A. Vertebral B. Aorta C. Middle cerebral D. Facial

C. Middle cerebral

The value that prevents the retrograde flow of blood into the atrium from the left ventricle is the: A. Tricuspid B. Pulmonary C. Mitral D. Aortic

C. Mitral

Which of the following types of blood components is the most common source of bacterial infection in a transfusion? A. WBCs B. RBCs C. Platelets D. Plasma

C. Platelets

Which of the following structures of the kidney does urine drain into from the major calyces? A. Nephron B. Bladder C. Renal Pelvis D. Ureter

C. Renal Pelvis

Another name for the folds on the inner wall of the stomach is: A. Microvilli B. Plicae C. Rugae D. Pyloris

C. Rugae

A barbiturate that may be used for sleep the night before surgery is: A. Diazepam B. Meperidine C. Secobarbital D. Scopolamine

C. Secobarbital

Two-thirds of the breast tissue rests on the pectoralis major muscle and the other one-third rests on the: A. Pectoralis minor B. Teres minor C. Serratus anterior D. External oblique

C. Serratus anterior

Which of the following statements describes second degree AV block? A. Conduction time is prolonged, but atrial beats are followed by ventricular beats. B. Ventricle contacts before the normal series of cardiac cycle C. Some, but not all, atrial beats are conducted to the ventricles D. Occurs in the AV node, bundle of His, or any of the bundle branches

C. Some, but not all, atrial beats are conducted to the ventricles

The mammary gland is fixed by the overlying skin and the underlying pectoral fascia by fibrous bands known as: A. Lobules B. Acini C. Suspensory ligaments of Cooper D. Deep pectoral fascia

C. Suspensory ligaments of Cooper

At which vertebral level is the ondontoid process located?

C2

What type of chemo is usually used for breast CA?

CMF (cyclophosphamide, methotrexate, 5-FU) CAF (cyclophosphamide, adriamycin, 5-FU)

Impaired venous return causing LE discomfort, edema and skin changes

Chronic vein isufficiency

During a laminectomy what is the preferred form of hemostasis?

Cottonoid strips or patties

The lithotomy position requires each of the following EXCEPT: A. Patient's buttocks rest along the break between the body and leg sections of the O.R. table B. Stirrups are at equal height on both sides of the O.R. table C. Stirrups are at the appropriate height for the length of the patient's leg to maintain symmetry D. Each leg is raised slowly and gently as it is grasped by the toes

D. Each leg is raised slowly and gently as it is grasped by the toes

The setting of the pneumatic tourniquet pressure on an average adult for a lower extremity should be: A. Lower than the systolic pressure by one-half of the value B. Higher than the diastolic pressure by one-half of the value C. Lower than the diastolic pressure by one-half of the value D. Higher than the systolic pressure by one-half of the value

D. Higher than the systolic pressure by one-half of the value

Which of the following is the most common change in vital signs that is associated with early hypovolemic shock? A. Bradycardia B. Decreased blood pressure C. Tachypnea D. Tachycardia

D. Tachycardia

The great sensory nerve of the face and head is the: A. Trochlear B. Oculomotor C. Hypoglossal D. Trigeminal

D. Trigeminal

Which of the following cranial nerves innervates the superior oblique muscle? A. Olfactory (I) B. Optic (II) C. Oculomotor (III) D. Trochlear (IV)

D. Trochlear (IV)

Which of the following cavities within the brain is filled with cerebrospinal fluid? A. Aperture B. Medulla C. Pyramids D. Ventricles

D. Ventricles

Define hypotension and the factors that contribute to it. Hypotension is a blood pressure reading that is 20% less than the baseline or preoperative blood pressure measurement. Hypotension may be caused by cardiac dysfunction or by medications (including anesthetic agents).

Define postoperative hypertension. Postoperative hypertension is defined by a 20% to 30% increase above baseline blood pressure measurement.

Define the term root cause analysis. A root cause analysis is a systematized process used to identify variations in performance that cause or could cause untoward patient events

Define the term extracellular fluid. Extracellular fluids (ECFs) are those fluids in compartments outside the cells of the body, including plasma, intravascular fluids, fluids in the gastrointestinal (GI) tract, and cerebrospinal fluid (CSF). ECFs comprise 30% of the body's fluid.

What is the capillary pressure threshold that, when exceeded, is associated with tissue trauma? The threshold for capillary pressure is 32 mm Hg.

Define the term friction. Friction is the force of two surfaces rubbing against one another.

Define the ASA P2 status. An ASA P2 status is defined as a patient with mild systemic disease.

Define the term local anesthesia. Local anesthesia is the administration of an anesthetic agent to one part of the body by local infiltration or topical application, usually administered by the surgeon.

What is the purpose of surgical drapes? Surgical drapes provide a barrier to microorganisms, particulate matter, and fluid while protecting the patient from both exogenous and endogenous sources of contamination.

Define the term surgical site (incisional) infection (SSI). A SSI is an infection involving body-wall layers that have been incised.

What is the fovea?

Depression in the head of the femur

Describe the energy path in a bipolar system. In a bipolar system, energy flows from one tine (or prong or blade) of the bipolar instrument to the other tine as it passes through the tissue located between these tines. Energy returns directly through the instrument to the generator to complete the circuit, eliminating flow of current through the patient.

Describe examples of flexible endoscopes. Flexible endoscopes include: angioscopes, bronchoscopes, choledochoscopes, colonoscopes, cystonephroscopes, hysteroscopes, mediastinoscopes, ureteroscopes, and ureteropyeloscopes.

Define the term pressure. Pressure is the force placed on underlying tissue.

Describe the prone position. In the prone position, the patient lies on his or her abdomen.

Describe the energy path in a monopolar system. In a monopolar system, electrical energy flows from the generator through an active electrode to the patient to a dispersive electrode (pad) or patient return electrode (PRE) to the generator, completing the circuit.

Differentiate between operative and diagnostic endoscopes. Diagnostic endoscopes are for observation only and have no operating channels. The system is sealed at both ends. Operative endoscopes are channeled to irrigate, suction, insert, and connect accessory instrumentation.

Adducting and externally rotating the femur will

Dislocate the hip anteriorly

The legal doctrine that mandages every professional to carry out his or her duties according to the national standards of care practiced throughout the country is the

Doctrine of reasonable man

The branches of the aortic arch are

brachiocephalic, left common carotid, left subclavian

Lacerations of the middle meningeal artery, often caused by a blow to the head, result in these hematomas.

Epidural

Focal conduction block: Patho response to injury *It's due to ? *Cessation occurs when?

Focal conduction block *Due to ischemia (modest degrees of tourniquet pressure) *Cessation of pressure can reverse the block, usually within minutes to hours

A vagal response would be

bradycardia

What is the purpose of a chemical indicator? A chemical indicator is used to detect failures in packaging, loading, or sterilizer function, such as the presence of cool air pockets inside the sterilizing chamber.

How does the Spaulding classification define critical items? Those items that enter sterile tissue or the vascular system are considered critical under the Spaulding system.

How does the Spaulding classification define noncritical items? Those items that come into contact only with intact skin are considered noncritical under the Spaulding system

How does the Spaulding classification define semi-critical items? Those items that contact nonintact skin and mucous membranes, but do not ordinarily penetrate the blood barrier are considered semi-critical under the Spaulding system

What is the mechanism of action in heparin?

breaks down the clot

What must be ruled out with a breast abscess in an nonlactating woman?

breast cancer!

Cold Ischemia

Kidney sits on ice

What is a radical nephrectomy?

Kidney, ureter, adrenal gland, gerota's fascia, lymph nodes

During a Whipple procedure, what surgical maneuvers are performed to dissect the lateral peritoneal attachments of the duodenum?

Kocher

What is the attachment of the lesser omentum to the liver?

L shaped to the fissure for the ligamentum venosum and to the porta hepatis

The spinal cord ends at what level?

L2

What prophylaxis regimen could be used for a pt with high risk (3-4 risk factors)?

LDUH q8h, LMWH, or SCD

This prophylaxis is given 1-2h pre-op, in 5000 units sq q8h-q12h, which binds to antithrombin III and inhibits platelet fxn

LDUH, Unfractionated heparin

This prophylaxis drug will be given 1-2h pre-op/ 8-12h post op, in inhibits factor Xa, and is given 30mg sq bid or 40mg sq qd

LMWH (Lovenox/enoxaparin)

Define the acronym LASER. LASER stands for: Light Amplification by Stimulated Emission of Radiation.

List the four components of a flexible endoscope. A flexible endoscope is composed of the following: control body, insertion tube, bending section at distal tip, light-guide connector unit.

What is obtained first, the mammogram or the biopsy?

Mammogram Tissue extraction may alter the mammographic findings FNA may be done prior because the fine needle usually will not affect the mammogram findings

What would be suspicious mammogram findings?

Mass Microcalcifications Stellate/spiculated mass

The major vessels are located on which aspect of the thigh at the mid-shaft level?

Medial

This type of ulcer is the result of microtrauma, is found over bony prominences, and in limbs with adequate vascular supply.

Neurotrophic ulcer

What type of ulcer do you treat by treating the underlying condition/etiology?

Neurotrophic; correct neuropathy, reduce repetitive microtrauma, remove deformity

Clean wound

Operative wound in which no re4spiratory, alimentary, or genitourinary tracts are entered and no break in sterile technique.

CHAPTER 2

PATIENT AND WORKER SAFETY

What is the median umbilical fold and what does it contain?

Passes from the bladder to the umbilicus contains the urachus which is the remnant of the fetal allantois

What are the indications for a biopsy?

Persistent mass after aspiration Solid mass Blood in cyst aspirate Suspicious lesion by mammography/US/MRI Bloody nipple discharge Ulcer or dermatitis of nipple Patient's concern of persistent breast abnormality

The linea aspera is a ridge of bone located on the

Posterior aspect of the femoral shaft

After retracting the nerve roots, this structure is incised before getting to the disc.

Posterior longitudinal ligament

This structure runs along the length of the vertebral column inside the vertebral canal. It helps keep the nucleus pulposus from protruding posteriorly and prevents the vertebral column from hyperflexing.

Posterior longitudinal ligament

Which radiographic option is best to evaluate a breast mass in a women under 30?

breast ultrasound

This tissue is concerned with the formation of red blood cells.

Red marrow

What is the etiology that needs addressing for a venous ulcer?

Reduce venous HTN, and improve venous return

The pituitary gland is found in the

Sella turcica

What antibiotics are used for bacterial colonization?

Silvadene 1% cream bid for 1 week, Silveron 30-40m w/ steril water under occlusion, acticoat/argleas- as a penetrative barrier not intended to reduce bacterial load, also orals and IV's.

This part of the vertebrae projects from the posterior vertebral body and is continuous with the laminae.

Spinous Process

What are the three pairs of arteries branching off the lateral abdominal aorta?

Suprarenal (L1) Renal (L1) Gonadal (L2)

What are the tx options for an arterial insufficiency ulcer?

Surgical revascularization, drugs, angioplasty, prevent trauma/infection, and lifestyle changes

In this needle the suture is attached at the factory

Swaged needle

What are the tx options for a venous ulcer?

Sx of damaged veins, elevation of legs, compression therapy

In the tx of pressure/decubitus ulcers, what is done to assess the pt's healing potential?

Systemic health, vascular eval, r/o OM; remember that sometime stx can be detrimental.

Which of the following is true regarding CSF in the brain?

The fluid does not play a significant role in supplying nutrition to the structures it bathes

What is the greater sac?

The space which contains all of the abdominal organs. Is really only a potential space and contains only a few ml of fluid for lubrication.

List several possible interventions done in preparation for removing the aorta cross-clamp.

Turn off SNP and consider lightening anesthetic Volume load judiciously Calcium chloride 1 gram Sodium bicarbonate, 50 to 100 meq If 15 of PEEP was used, remove it

What is the lesser omentum?

Two layers of peritoneum that extend down from liver to the upper border of stomach also known as gastrohepatic ligament form the anterior border of the lesser sac

What type of endoleak is when blood flows into the aneurysm sac due to opposing blood flow from collateral vessels?

Type II endoleak

Describe the sigmoid mesocolon

Upside down V shaped with the apex at the bifurcation of the left common iliac vessels over the Sacro-Iliac joint The lateral limb passes forward along the pelvic brim (over external iliac vessels) halfway to the inguinal ligament (about 5cm) the medial limb slopes down into the hollow of the sacrum where it reaches midline over S3

What are the relations of the ureter and IM vessels to the sigmoid mesocolon attachment

Ureter crosses pelvic brim just beneath peritoneum, lying over the bifurcation of the common iliac artery IMvessels lie medial to the ureter The IMV lies between the IMA and the ureter. L246 N

This part of virchow triad initiates clotting mechanism, and may be due to trauma/LE bone fracture. Endothelial lining is exposed to blood and is unable to inhibit platelet clotting.

Vein Endothelial damage; blood exposed to endo collagen

Define the term epidermis. The epidermis is the outermost layer of the skin composed of several layers consisting of keratin and lipids. It lines the ear canals, and is contiguous with the mucous membranes.

What are the defining characteristics of an organ/space SSI? Purulent drainage from a drain that is placed through a stab wound into the organ/space; organisms isolated from an aseptically obtained culture of fluid or tissue in the organ/space; an abscess or other evidence of infection involving the organ/space that is found on direct examination, during reoperation, or by histopathologic or radiologic examination; and diagnosis of an organ/space SSI by a surgeon or attending physician.

List the categories of microbial resistance. Microbial resistance can be divided into three categories: (1) presence of a naturally resistant strain of an organism before any drugs are administered, (2) acquisition of a drug-resistant strain from an external source, and (3) drug resistance from treatment-related causes

What is the retrovirus that attacks the immune system by destroying T-helper lymphocytes? Human immunodeficiency virus (HIV) is the retrovirus that attacks the immune system

List the risk factors for vancomycin-resistant enterococci (VRE) infections. Risk factors for VRE include: prolonged stays in the critical care unit, intra-abdominal or cardiothoracic surgical procedures, presence of indwelling central venous catheters, presence of urinary catheters, extended hospital stays, multiple antimicrobial therapies, and proximity to infected individuals

What is the standard for OR air exchanges per hour? The standard for OR air exchanges is a minimum of 15 total air exchanges per hour, with the equivalent of at least 3 replacements being of outside air to satisfy exhaust needs of the system.

Describe the supine position. In the supine position, the patient lies with the back flat on the operating room (OR) bed with their arms tucked at the side or placed on armboards.

What peripheral nerve is at risk during lithotomy positioning? The common peroneal nerve is at increased risk for injury during lithotomy positioning.

A familiarity with basic neurologic assessment tools gives the nurse the ability to perform a standardized neurologic exam that can be used preoperatively to establish a baseline assessment and postoperatively to establish a return to baseline or assess postoperative neurologic stability. The Glasgow Coma Scale (GCS) is commonly used to assess patients with brain injury using three indicators of cerebral function: eye opening, verbal communication, and motor response to verbal and noxious stimuli. Within the scoring range for the GCS, the best possible score is _____ and the worst possible score is ______.

a. 15, 3

Can tamoxifen prevent breast cancer? What are the SEs of tamoxifen?

Yes In high risk women, tamoxifen for 5 yrs will lower risk by up to 50% (but with an increased risk of endometrial cancer and clots) Endometrial cancer**, DVT, pulmonary embolus, cataracts, hot flashes, mood swings

While the pancreas' function is carbohydrate metabolism with the production of insulin and digestive enzymes, the spleen's function is primarily ___________ with the production of _______________.

a. immunologic; leukocytes

The thoracic duct begins in the

abdomen at the cisterna chyli

The P-wave displays__, and the PR segment is caused by the delay in the AV node.

atrial depolarization

A common pathogen associated with decubitus ulcers is Staphyloccocus

aureus

The most frequent location for a cerebral aneurysm is the ______.

b. Cerebral arterial circle

The liver is divided into two principal lobes by which ligament?

falciform

The surgeon focuses on the ______ circle

inner

Studies show that the lowest infection rates have been obtained in surgical cases where patient body hair was

not removed

The gelantinous substance within an intervertebral disc is called the

nucleus pulposus

The blood supply is introduced to the medullary canal through what structure?

nutrient foramen

Which types of fractures is at the greatest risk for infection?

open

Hernia means

rupture

What is the classic picture of breast CA on mammogram?

spiculated mass (stellate)

Between what postop days can dehiscence typically occur?

fifth and tenth

Which of the following is not true of a hydrocele?

It is typically associated with a direct inguinal hernia.

Lateral Position

*Head -Head in neutral alignment, tube secured, dependent eye free from pressure *Trunk -Axillary Roll (not in the axilla), Bean Bag or posts *Arms -Arm board, supinated, padded and secured *Legs -Pillows between legs, dependent leg flexed, non-dependent leg slightly flexed

Supine: UE pronation vs. supination ..pronation is palms facing inward *Supination puts pressure on the ? nerve *Pronation puts press on the ? nerve Thus, ?

*The supinated position puts pressure on the olecrannon process, which avoids pressure on the condylar groove, thus reducing pressure on the ulnar nerve *However, there is a greater stretch of the brachial plexus than with the pronated arm *Recommend a neutral position

Prone: Head Positioning

*To avoid turning the head, a head rest or foam pillow can be used -Mayfield head rest

Lateral Surgeries: Examples

*Total Hip Replacement *Partial Nephrectomy *Thoracotomy or lung resection surgeries

Neurotmesis is ? Recovery is ?

*Total severance or disruption of nerve *No spontaneous recovery, surgical intervention is necessary

Most common peripheral neuropathy is of the ? nerve *it sits along the ?

*Ulnar -Due to its superficial path along the medial epicondyle of the humerus

Physiologic Changes in Lateral Position: Pulmonary

*V/Q mismatching (most of any position) especially with PPV and one lung ventilation

Sciatic Nerve: Signs/Symptoms

*Weakness in all skeletal muscle below knee *Diminished sensation over the lateral half of the leg and almost all of the foot, except the inner border of the arch *Can be mistaken for injury of the common peroneal nerve (foot drop) -Branch of the sciatic nerve which is more vulnerable to stretch

Axonotmesis is what to the axon? Recovery is dependent on ? Recovery takes about ? - ? months

*When the axon is physically damaged with preservation of the endoneurium and other supporting connective tissue structures *Recovery depends on the rate of re-growth of axon *Can take up to 12 to 18 months

Demyelination is b/c ? is damaged Degree of damage proportional to ?

*With acute or chronic compression, Schwann cells are damaged leading to loss of myelin segments *Degree of damage is proportional to the degree and duration of pressure applied

Supine: Trendelenburg *This puts the head up/down? *Shoulder braces put pressure on the ?

*down *Use of shoulder braces can lead to brachial plexus injury

What does the thoracic lymph duct drain into?

the cranial vena cava or left jugular vein

How much Heparin is given during aortic reconstruction / AAA repair?

5,000 - 10,000 units Heparin

What do the renal arteries do?

functional as well as nutritional support to the kidneys

Q. Which of the following would be considered the study of occurrence and distribution of disease?

A. Epidemiology Explanation: Epidemiology would be considered the study of a diseases' occurrence and distribution. Epidemiology could also be considered the sum of all of the factors which controls the presence or absence of a disease.

Which section of the stomach is wrapped behind the esophagus during a Nissen procedure?

fundus

What characteristics of sutures is described as the amount o force necessary to break suture?

tensile strength

The dura plan that seperates the cerebrum from the cerebellum is the

tentorium cerebelli, transverse

Q. Which nursing intervention would you prioritize as first for a diagnosis of ineffective thermoregulation with febrile signs?

A. Administer antipyretics Explanation: The patient has a fever and requires antipyretics. This should be prioritized before the other nursing interventions. The temperature should be monitored after the antipyretics are administered.

Q. The endocrine system is controlled by stress, which can contribute to adverse outcomes. The two major endocrine glands affected by stress are which of the following?

A. Adrenal and pituitary Explanation: The adrenal and pituitary glands are the two glands most affected by stress. The adrenals produce the stress "hormones" which cause the "flight or fight" response. The stress hormones are cortisol, norepinephrine, and growth hormone.

The area where the common bile duct joins the pancreatic duct and enters the duodenum is called the: A. Ampulla of Vater B. Duct or Santorini C. Duct of Wirsung D. Islets of Langerhans

A. Ampulla of Vater

An enzyme active in the digestive of starches is. A. Amylase B. Trypsin C. Lipase D. Lactase

A. Amylase

A chemical reaction that provides energy by the breakdown of food molecules is called: A. Catabolism B. Metabolism C. Glycogenesis D. Anabolism

A. Catabolism

Q. The electronic information on a computer is referred to as ______?

A. Data Explanation: Inputting would be the process of entering information into the computer. Images would be pictures on the computer screen. The process is used in order to make the equipment work. The electronic information is referred to as data.

Which of the following should be confirmed before administration of any blood products? A. Date of expiration B. Clots are present C. Registration number on blood bag matches patients ID number D. Blood product is homologous

A. Date of expiration

Q. Which of the following is not factor in determining nursing priorities?

A. Determining the easiest tasks Expanation: A nursing priority mistake is performing the easier tasks first. The ease of a task does not determine priorities. The other answers need to be considered when determining nursing priorities.

Q. What is the last part of a research study?

A. Discussion section Explanation: The last part of a research study is the discussion section. In this part of the study, the researches put together all of the information from the study and look at it in a single picture.

Acid based imbalances reflect: A. Hypoxia, hypothermia and hypovolemia B. A blood pH of 7.35 to 7.45 C. The oxygen supply being equal to the oxygen demand D. Oxygen saturation of 80-100 mmHg at sea level

A. Hypoxia, hypothermia and hypovolemia

Q. Which of the following statements about compensation (the return of an abnormal pH to a normal pH) is least accurate?

A. In metabolic acidosis with a base excess, the lungs compensate through hypoventilation. Explanation: This is the least accurate statement. In metabolic acidosis with a base excess, the lungs compensate through hyperventilation. This lowers PaCO2 and returns the pH to normal.

Q. What is a common complication with drainage systems?

A. Infection Explanation: Infection is a common complication of drainage tubing. The site of the tube insertion should be constantly monitored to ensure there are no changes in the skin.

Q. Which of the following would be considered correct for the interaction between the pathogenic microorganism, the environment, and the host?

A. Infectious disease process Explanation: The infectious disease process would be the interaction between the pathogenic microorganism, the environment, and the host. The infectious disease process consists of six stages.

Q. Agreement by a patient to accept a course of treatment is known as which of the following?

A. Informed consent. Explanation: The agreement by a patient to a course of treatment, based on the information provided, is informed consent. Children under 18 cannot provide informed consent, but only assent; a parent's informed consent is needed for most procedures. Be familiar with local laws regarding exceptions, as some minors can seek some care without parental consent. Informed consent must involve a signature.

Q. Which of the following is true of the action stage?

A. It is the most risky stage of change, as it has the most potential for relapse Explanation: The action stage is considered the most risky stage of change. It is during the action stage that most people relapse into prior risky or unhealthy behaviors.

What local anesthetic is used most frequently for preoperative ocular-instillation?

tetracaine

Q. Nursing supervision generally involves all of the following except for:

A. Providing licensure or certification for nurses Explanation: The nursing supervision role is broad and varies from facility to facility. In general, nursing supervisors provision orientations, in-services, and continuing education. They discuss policies and procedures with nursing team members and conduct performance appraisals. They do NOT certify, license, or provide credentialing for nurses.

Q. What is a disadvantage of using a standardized care planning form in a multidisciplinary setting?

A. Reduce individual intervention. Explanation: The need for a nurses individual intervention may diminish with standardized care planning. It may also limit the focus to problems that are predictable. The other answers are pros of standardized care planning.

Q. Debridement is what?

A. Removal of necrotic tissue. Explanation: Correct answer: Removal of necrotic tissue Debridement is the process of surgically removing dead or damaged tissue from the patient. It is similar to the term amputate but amputation usually involves cutting off a limb.

Q. Erik Erikson described development that occurs throughout the lifespan. In which of Erikson's stages of psychosocial development does a child need to begin asserting control and power over his environment?

A. initiative vs. guilt Explanation: In the stage of initiative vs. guilt children need to begin asserting control and power over the environment. Success in this stage leads to a sense of purpose. Children who try to exert too much power experience disapproval, resulting in a sense of guilt.

What could occur when a posterior fossa craniotomy is performed with the patient in a sitting position?

venous air embolism

The T-wave indicates

ventricular repolarization

What are the three arteries that branch off the anterior abdominal aorta?

Celiac ( T12) Superior mesenteric (L1) Inferior mesenteric (L3)

Name 8 major arteries that branch off the abdominal aorta

Celiac, Superior mesenteric, Inferior mesenteric, Suprarenal, Renal, Gonadal, Inferior phrenic, Lumbar

The most common congenital defect of the heart is a

ventricular septal defect

Define class IV Dirty or Infected Wounds. Class IV Dirty or Infected Wounds are old, physically induced wounds with retained devitalized tissue and wounds that involve an existing clinical infection or perforated viscera.

Define class III Contaminated Wounds. Class III Contaminated Wounds are described as open, fresh, accidental wounds, such as penetrating trauma, open fracture, or one that occurs during an operation with major breaks in aseptic technique. Incisions with signs of infection or gross spillage from the gastrointestinal tract also are included.

List examples of objective factors that can be used to evaluate pain. Objective factors used to evaluate pain include protective guarding behavior, moaning, crying, whimpering, restlessness, irritability, diaphoresis, dilated pupils, facial expression of pain, and changes in vital signs (blood pressure, respiratory rate, or pulse).

Describe the components of typical discharge instructions. Discharge instructions usually include information relating to wound or incision care, signs and symptoms of a wound infection, pain management, return to activity, physician contact numbers, medication management, and physician follow-up.

Define the term ligature A ligature is a strand of suture material used to tie off (seal) blood vessels for the prevention of minor bleeding, or to isolate a mass of tissue for excision (cut out)

Describe the ideal suture. An ideal suture causes minimal inflammation and tissue reaction while providing maximal strength during the lag phase of wound healing.

What term is used to describe knot tying outside of the body during laparoscopic procedures? Knot tying outside of the body is known as extracorporeal suturing.

Describe the key elements of laser safety. Laser safety includes the use of eye protection, controlled access, fire safety, smoke (plume) evacuation, skin/tissue protection, and electrical safety.

Which of the following is incorrect pertaining to the inguinal ligament

It is often perforated by a direct inguinal hernia

Describe the techniques used to ligate in deep tissues. The following techniques can be used to ligate in deep tissue: (1) A hemostat is placed on the end of the structure; the ligature is then placed around the vessel. The knot is tied and tightened with the surgeon's fingers or with the aid of forceps. (2) A slipknot is made, and its loop is placed over the involved structure by means of a forceps or clamp. (3) In deeper cavities, ties are often placed on clamps with the long end extending from the tip. These are sometimes called ties on a pass or bow ties. The extending long end is held tightly against the rings by the surgeon (creating the bow), who then passes the tip of the clamp under the vessel or duct to be ligated. The first assistant grasps the extending tie with a forceps, the surgeon releases it, and the tie is pulled under and up to the wound surface and tied. (4) A forceps or a clamp is applied to the structure, and transfixion sutures are applied and tied. A suture ligature, stick tie, or transfixion ligature is a strand of suture material threaded or swaged on a needle. This is usually placed through the vessel and around it to prevent the ligature from slipping off the end.

List examples of absorbable sutures. Plain or chromic surgical gut, collagen, and glycolic acid polymers are all examples of absorbable suture

List the advantages of single-use laparoscopic instruments. Single-use instruments are associated with sharpness, reliability, guaranteed sterility, safety, no reprocessing, and no repair costs.

List the risks of overpressurization during pneumoperitoneum. Overpressurization can cause hypercarbia, gastric regurgitation and aspiration, decreased respiratory effort, decreased cardiac output, and phrenic nerve irritation or damage.

Define the term traumatic wound. A traumatic wound is one caused by mechanical, thermal, or chemical destruction.

List the three phases of wound healing. The three phases of wound healing are: (1) inflammatory, (2) proliferative, and (3) remodeling.

Describe the acceptable range of intra-abdominal pressure to maintain during procedures requiring pneumoperitoneum. The acceptable range for pneumoperitoneum is 14 to 16 mm Hg.

List the types of lasers commonly used in the perioperative setting. Lasers used in the perioperative setting include the CO2, erbium:YAG, holmium:YAG, Nd:YAG, KTP, argon, tunable dye, diode, and excimer lasers.

Describe laryngospasm. During laryngospasm, the muscles of the larynx contract and partially or completely obstruct the airway.

List two drug classes often used in combination to treat postoperative pain. Medications commonly used to treat postoperative pain include nonsteroidal antiinflammatory drugs (NSAIDs) and opiates.

It is a reverse-cutting needle specially polished and carefully honed for plastic surgery

Precision point needle

What stage of breast cancer? Tumor >5 cm with mobile axillary nodes or tumor > 5 cm with fixed axillary nodes, no mets

Stage III A

What stage of breast cancer? Peau d'orange or chest wall invasion/fixation or inflammatory CA or breast skin ulceration or breast skin satelite mets or any tumor and + ipsilateral internal mammary lymph nodes

Stage III B

What stage of breast cancer? Distant mets including ipsilateral supraclavicular nodes

Stage IV

What constitutes an open or compound fracture?

The broken bone punctures through the skin

The frontal lobe is bounded posteriorly by the central sulcus, also called the

fissure of rolando

What seperates the frontal and temporal lobes?

fissure of sylvius

Carlos Gracia is a 12-year-old boy scheduled for a laparoscopic hernia repair. During the preoperative assessment and interview, the perioperative nurse notes that Carlos has mild myotonia congenita. Carlos' mother states that she is worried about the anesthesia because her husband's brother died many years ago when he became very feverish during surgery and they could not cool him. The perioperative nurse's appropriate risk reduction strategy in this situation would be to:

a. perform a careful assessment of family history to identify risk for developing malignant hyperthermia (MH).

What are the screening recommendations for breast cancer? a. breast exam recommendations? b. mammograms?

a. self-exams monthly; exam every 2-3 years for 20-40 yo; exam annually for those over 40 b. baseline betwen 35-40 years; mammogram annually or every other yr for 40-50 yo; annual after 50

Cirrhosis of the liver is associated with all of the following except:

a. softening of the liver.

Mastitis... a. What is it? b. In what circumstance does it most often occur? c. What bacteria are MC cause? d. Tx? e. Why closely follow up this pt?

a. superficial infection of breast (cellulitis) b. breastfeeding c. staph aureus d. stop breastfeeding; use breast pump instead; apply heat; Abx e. to be sure she doesn't have inflammatory breast CA

which of the following is incorrect pertaining to the pancreas

a. the superior mesenteric artery lies anterior to its neck.

what statements regarding vascular clamps are true? select all that apply

a. vascular clamps are designed to partiall occlude blood flow. b. vascular clamps are designed to totally occlude blood flow. c. all vascular clamps are designed to hold vessels securely without trauma.

What do the phrenicoabdominal arteries supply?

abdominal wall and diaphragm

Which cranial nerves innervate the lateral rectus muscle of the eye?

abducens (VI)

During arterial cannulation in cardiopulmonary bypass (CPB), the cannula tip must be directed into which of the following arteries?

aorta

This orthopedic diagnosis is a complex deformity that is diagnosed prenatally through ultrasound or at birth. It may occur unilaterally or bilaterally and may be idiopathic or one in a combination of other syndromes with associated anomalies. The general characteristic for all cases includes inversion of the foot such that the anterior foot is located in the typical position of the posterior foot; often there is a deep crease in the midfoot. Select the orthopedic diagnosis.

b. Clubfoot

Select the true statement about application and operation of the pneumatic tourniquet.

b. Cuffs should overlap a minimum of 3 inches and a maximum of 6 inches; excess overlap can pinch skinfolds. A tourniquet cuff that is too short can loosen after inflation.

The brainstem is located in the posterior fossa and forms the floor of the fourth ventricle. It is the site of many ascending and descending fiber tracts that allow for communication among the structures of the brain and between the brain and spinal cord. The brainstem consists of what three structures?

b. Midbrain, pons, and medulla oblongata

Examples of triangulation of focus are procedures in which an object is _____________ or __________________

deep in the abdomen or underneath overlying structures that should not be disrupted

Question text Morris Bettelman, an 86-year-old retired plumber, states that he has suffered from a groin hernia all of his adult life; he states, "as far back as I can remember." Sometimes he has to push the bulge back into his abdominal muscle and then he is fine until he has a coughing spell. This morning the bulge was large and tender when he woke up and he could not reduce it. As the pain increased, he felt weak and nauseous. His daughter took him to the emergency department. Morris was diagnosed with a strangulated incarcerated hernia and scheduled for emergency surgery. If the contents of Morris' hernia sac become compromised, with strangulation of the bowel, the probable label on his surgical specimen will be:

c. necrotic bowel.

What is the greater omentum?

double sheet of peritoneum that passes down from stomach over transverse colon, to cover bowel, turns around and comes back up to loosely blend with the peritoneum overlying transverse colon and mesocolon Essentially is four layers below the transverse colon - that fuse with each other to form an apron.

What type of breast cancer do men develop?

ductal carcinoma (men don't usually have breast lobules)

The head of the pancreas is surrounded by the

duodenum

What is the fornix in the kidney organ?

edge of the minor calyces

What is the inherent ability to regain the original form and length after being restructured?

elasticity

The primary purpose of a robotic device designed to hold and maneuver a laparoscope is to

eliminate tremor associated with operating an endoscope.

Which of the following veins drains the intracranial venous sinuses to veins on the outside of the skull?

emissary

All of the following complications can result from a total thyroidectomy EXCEPT: *hypothyroidism *tetany *hoarseness *esophageal rupture

esophageal rupture

Zenker's diverticulum is an outpouching of the

esophagus

What hormone receptors must be checked for in the biopsy specimen of suspected breast CA?

estrogen and progesterone receptors -- this is the key for determining adjuvant treatment ***

The term for a wound that completely separates and the internal organs extrude is

evisceration

Which wound complication would require immediate surgery?

evisceration

What party of the femur articulates with the tibia and patella?

femoral condyles

Which retractors are typically used to retract the strap muscles during a thyroidectomy?

green

Pathologic enlargement of the male breast is called

gynecomastia

Reasons for skin graft failure include

hematoma

What is the purpose for the surgical creation of an arteriovenous fistula?

hemodialysis

A potential complication that can occur when the patient is undergoing an occipital craniotomy while in Fowler's position is

hypotension

Body temperature is regulated by the

hypothalamus

A common complication during emergence from general anesthesia is

hypoxia

What is the anastomosis of choice when perfoming a total abdominal colectomy for ulcerative colitis?

ileoanal

List the risk factors for male breast cancer?

increased estrogen radiation gynecomastia from increased estrogen estrogen therapy Klinefelter's syndrome (XXY) BRCA2 carriers

Which arteries are identified, ligated, and divided during the perineal dissection of an abdominoperineal resection?

inferior rectal artery

The recurrent laryngeal nerve that supplies the vocal cords lies very close to the

inferior thyroid artery

What is the name of the cutaneous nerve that crosses the axilla in a transverse fashion?

intercostobrachial nerve • Many surgeons try to preserve this nerve.

Which branch of the trigeminal nerve must be identified and prserved when performing a submandibular gland removal?

lignual

The proper placement of th legs for a patient in the lateral position is

lower leg flexed, upper leg straight

When opening the aneurysmal sac during resection of an abdominal aortic aneurysm (AAA), which of the following arteries is ligated?

lumbar

An organelle that contains digestive juices is the

lysosome

What is the anesthetist's chief goal in monitoring and treating hemodynamics during AAA repair?

maintaining cardiovascular stability

AAA occurs most often in what group of people?

males > 60 yrs

Which of these local anesthetics has a longer duration? *xylocaine *marcaine *novocaine *cocaine

marcaine

Which structures that travel thoughthe temporal bone must be identified and preserved when performing a mastoidectomy?

mastoid sinus, facial nerve

Which surgical maneuvers mobilizes the descending colon to the midline to expose the abdominal aorta?

mattox

In this phase of wound healing, collagen fibers rearrange, and increase in strength

maturation/remodeling phase

The arterial blood supply to the nose is from branches of the internal and external

maxillary

The most frequent site of cartilage tears in the knee joint is the

medial meniscus

What are the 5 causes of male gynecomastia? What is the major differential diagnosis in the older patient? What is the treatment?

medications ** illicit drugs (marijuana) liver failure increased estrogen decreased testosterone Male breast cancer Stop medications, correct underlying cause, perform biopsy or SQ mastectomy if refractory to conservative treatment

The structure that bind the small intestine to the posterior abdominal wall is the

mesentery

The ovary is suspended from the posterior leaf of the broad ligament by the

mesovarium

Injury to which of the following arteries is commonly seen in the temporal lobes, where the skull is the thinnest and meningeal blood vessels are numerous, and is the usual cause of epidural hematomas?

middle meningeal

When assisting the surgeon with the draping, where should the surgical assistant stand in relation to the surgeon?

on the opposite side of the OR table from the surgeon

According to the Mallampati classification of preoperatively assessing airway adequacy for intubation purposes what is Class III?

only the base of the uvula can be visualized

Primum non nocere

or first do no harm

What would affect normal wound healing?

peripheral vascular disease

In the event that a child needs an emergency procedure and the parents cannot be located to sign the permission

permission is signed by two consulting physicians

When possible during a plastic surgery procedre, incision should be placed

perpendicular to the long axis of the underlying muscle

When removing a tumor, isolation technique is used to

prevent mechanical metastasis

Which muscle is an important anatomical landmark in fractures of the forearm?

pronator teres

What is not an inhalation anesthetic agent?

propofol

A drug combined with heparin that causes a postop loss of anticoagulant activity is

protamine sulfate

Plication of the inferior vena cava or insertion of an imbrella-type filter is performed to treat

pulmonary embolus

Which of the following suturing techniques is used for aortic and atrial cannulation for cardiopulmonary bypass

purse-string

Surgical enlargement of the passage between the prepylorus of the stomach and the duodenum is a

pyloromyotomy

A vagotomy is typically performed in conjection with which other procedure?

pyloroplasty

The frozen section examination provides the surgen with a

quick preliminary diagnosis

The mechanical strength of a weak eye muscle due to strabismus in a pediatric patient can be corrected by all of the following EXCEPT: *tucking *advancement *recession *resection

recession

The section of the gastrointestinal tract that is least affected by Crohn's disease is the

rectum

A sigmoid colostomy is brought through the

rectus muscle

What nerves must be identified and protected from injury during a thyroidectomy?

recurrent laryngeal

Which nerve is at the greatest risk for injury during a thyroidectomy?

recurrent laryngeal

Which paired nerves must be carefully identified during a thyroidectomy?

recurrent laryngeal

The primary factor when selecting the surgical needle is

reducing as much as possible the trauma to the tissue

Which artery is susceptible to injury during a cholesycstecomy?

right hepatic

Know the arterial branches of the celiac trunk

right hepatic artery left hepatic artery proper hepatic artery celiac trunk splenic trunk left gastric artery common hepatic artery gastroduodenal artery

What chamber of the heart ejects deoxygenated blood into the pulmonary artery for oxygenation by the lungs?

right ventricle

Which carpal bones are fractured most frequently?

scaphoid

During a posterolateral approach to the hip, which of the following structures usually found superficially to the obturator and gemelli should be identified and protected?

sciatic nerve

A procedure to treat retinal detachment is

scleral buckling

Incomplete closure of paired verterbral arches that can be treated surgically is know as

spina bifida

The flexure between the transverse colon and the descending colon is called the

splenic

another name for the left colic flexure is

splenic flexure

Which two arteries must be preserved during pancrease retrieval?

splenic, superior mesenteric

What is the most common physically identifiable pathology of portal hypertension?

splenomegaly

The left gastric, right gastric, right gastroepiploic, left gastroepiploic, and short gastric arteries supply the

stomach

Two hands/ten fingers

strive to be equally proficient with both of his hands, should be able to use all instruments with both hands, Ability to hold an instrument in part of the hand, while using the rest of the hand to accomplish another task

The large artery that arises from the left side of the aortic arch and descends into the arm is the

subclavian

Continuous lateral stitches placed beneath the epithelial layer of the skin are

subcuticular

The arterial supply to the intestinal tract, with the exception of the stomach, duodenum, and distal rectum, is derived from the

superior and inferior mesenteric arteries

Which arteries supplies the cecum, ascending colon, hepatic flexure, and proximal portion of the transverse colon?

superior mesenteric

What does the celiac artery supply?

supplies major part of the abdominal viscera through its branches (liver, spleen and stomach)

The opthalmic artery divides into the

supraorbital and supratrochlear

What is a correct statement concerning the principles of suture placment?

sutures should be placed in deep tissue to prevent hematoma

What body system evokes a physiologic response that is similar to the response evoked by sympathomimetic cardiovascular drugs?

sympathetic nervous system

The amount of tension or pull expressed in pounds that a suture can withstand before it breaks is

tensile strength

Which of the following conditions is MOST common when insulation failure occurs in laparoscoopic surgery? A. hypothermia B. wound infection C. tissue injury D. adhesions

tissue injury

What referes to a civil wrong committed against a person or property, not including a breach of contract?

tort

The trendelengurg position would be most appropriate for which procedure?

total abdominal hysterectomy

Retraction of eyelid tissues requiring intermittent retraction and release over a period of time is accomplished by utilizing

traction sutures

Pulling on the wrist to reduce a radius fracture incorporates which principle of surgical assisting?

traction-countertraction

The surgeon applies ________ and the surgical assistant applies ________

traction; countertraction

Both direct and indirect hernias are the result of tears in the

transversalis fascia

The border of the acetabular fossa of the hip joint that is deficient forming a C-shaped fossa that creates a gap is bridged by the

transverse ligament

Non-union can be caused when bone graft isn't used to fill a bony deficit at the fracture site

true

Why does skin retraction occur with breast CA?

tumor involvement of Cooper's ligaments and subsequent traction on ligaments pull skin inward

What of the following is NOT a layer of an artery? *tunica media *tunica intima *tunica adventitia *tunica muscularis

tunica musclaris

Langer lines generally lie perpendicular to the direction of

underlying muscle tension

What is the surgical treatment of vesicoureteral reflux?

ureteral reimplantation

Which arteries are contained within the cardinal ligament?

uterine

When preforming compression hip screw fixation, the lag screw is inserted

within 1 cm of the femoral articular surface

A patient scheduled for surgery has a hematocrit reading of 45% of total blood volume, which is

within normal range

The only depolarizing muscle relaxant in the clinical use is: A. Rocuronium (Zemuron(R)) B. Vecuronium (Norcuron(R)) C. Cistracurium (Nimbex(R)) D. Succinylcholine (Anectine(R))

D. Succinylcholine (Anectine(R))

Describe the verification steps for transfusion of blood or blood products. The circulating nurse and anesthesia provider (or a second licensed individual) confirm the following during blood and blood product verification: (1) The unit number on the blood product corresponds with the unit number on the blood requisition. (2) The name, birth date, and number on the patient's identification band agrees with the name, birth date, and number on the slip with the blood product. (3) The patient's name on the blood product corresponds with the name on the requisition. (4) The blood group indicated on the blood product corresponds with that of the patient. (5) The date and time of expiration have not been reached. (6) The blood product bag is free of leaks, damage, or signs of possible bacterial contamination (e.g., presence of fine gas bubbles, discoloration, clots or excessive air in the bag)

List examples of initiatives developed by the World Health Organization (WHO) to promote patient safety. The WHO has endorsed the Hand Hygiene Initiative (Clean Care is Safer Care), the Surgical Hand Preparation guideline, and the Surgical Safety Checklist (Safe Surgery Saves Lives)

Describe Stage IV anesthesia. Stage IV spans from cessation of respiration to circulatory failure that leads to death.

List the populations at increased risk for intraoperative awareness (IOA). Obstetrics, major trauma, and cardiac surgery are all at increased risk for IOA.

Local anesthesia, or monitored anesthesia care (MAC), is used for most ophthalmic surgery. The local anesthetic regimen, administered by the operative surgeon, is typically a combination of anesthesia delivery methods, or routes. Select the anesthetic plan that is administered by the surgeon before the start of surgery.

b. Topical anesthetic eye drops, subconjunctival infiltration, and retrobulbar block

A solution used for eye irrigation is

balanced salt solution (BSS)

Which one of the folowing statements about the heart is INCORRECT? *Aortic valve sound can best be hear over the right second intercostal space *Most cardiac veins drain into the coronary sinus *Blood passes though the bicuspid (mitral) valve when it flows fromt he right atrium to the right ventricle *The fossa ovale is the site of the former, embryonic shunt from the right atrium to the left atrium

blood passes through the bicuspid (mitral) valve when it flows from the right atrium to the right ventricle

What is considered to be a biological hazard in the operating room?

body fluids and laser plume

Identify the marginal boundaries of the breasts

c. Second to sixth rib and horizontally from the lateral edge of sternum to anterior axillary line

Of the three salivary glands, which one is the largest?

d. Parotid

A radical lymphadenectomy is performed to treat cancer of the

testicles

What constitutes an open or compound fracture?

the broken bone punctures through the skin

What is the mammary "milk line"?

the embryological line from shoulder to thigh where "supernumerary" breast areolar and/or nipples can be found

Q. Which of the following would not be included on a surgeon's preference card?

A. Patient's insurance plan Explanation: Some of the things that could be included on the surgeon's preference card includes: Glove size Drapes Sutures Patient's name Surgeon's name Positioning Instruments Procedure Medications Disposable supplies Patient data Sponges Notes Any special request Prep

Platelet concentration is often transfused during the immediate preoperative period for the purpose of: A. Preserving clotting ability B. Preventing hypovolemia C. Treating hypoproteinemia D. Correcting Rh mismatch in prenatal women

A. Preserving clotting ability

The P-wave displays _______, and the PR segment is caused by the delay in the AV node. A. Atrial depolarization B. Electrical activity C. Ventricular repolarization D. Activation of right and left ventricles

A. Atrial depolarization

Q. A situation where two or more moral principles apply but point to diverging and contradicting courses of action is known as which of the following?

A. Ethical dilemma Explanation: An ethical dilemma is said to occur when two or more moral principles are at work but the courses of action are contradictory. This may occur when two equally undesirable choices are posed, or when and individual's ethics or values conflict with the law.

Q. What is one role of the risk manager in a healthcare facility providing perioperative services?

A. Evaluate outcomes of patient care Explanation: Risk management in healthcare facilities oversee any concerns related to hazards for patients and staff. They evaluate outcomes of patient care and often work closely with performance improvement committees to provide quality care and minimize facility costs.

Q. A dynamic state of well being that includes the physical, social, spiritual and cultural factors of a family Is known as the following?

A. Family health Explanation: Family health focuses on the many factors that can impact the functioning of both members of a family and the family system. When working with pediatric, elderly, disabled, or very ill patients, assessing family health is especially key, as these patients are likely to be highly dependent on family for daily needs and for caregiving.

Q. Which of the following nursing interventions is least likely to be used for a patient with unstable angina?

A. monitoring for bleeding Explanation: Nursing considerations for a patient with unstable angina would include all of the choices except this one. Other considerations include: recording the amount of medication required to relieve pain, obtaining cardiac enzyme levels, administering oxygen, and administering medications as ordered.

Q. In terms of nursing research, when people in the research study are arbitrarily assigned to different groups, this is known as which of the following?

A. randomization Explanation: Using randomization helps ensure research quality. Randomization requires arbitrarily assigning people to different groups. This makes it a hallmark of high-quality research.

To establish a tracheostomy, a transverse or midilne incision is made in the neck between the

cricoid cartilage and suprasternal notch

To perform cholangiography, which duct is identified and incised for placement of the cholangiocatheter?

cystic

The three structures that form the anatomical triangle of Calot are

cystic duct, common hepatic duct, lower edge of the liver

Specific malignant conditions that may require splenectomy include all EXCEPT _____.

d. Carcinoma of left kidney

Which rectus muscle moves the eyeball upward and medially? A. Lateral B. Median C. Inferior D. Superior

D. Superior

List examples of nonabsorbable sutures. Silk, nylon, polyester fiber, polypropylene, and stainless steel wire are examples of nonabsorbable sutures.

List the indications for using a surgical adhesive. Surgical adhesives are used in place of subcuticular sutures 4-0 in diameter or less in areas that are not under high tension or subject to increasing tension with movement

List the possible complications of epidural or caudal anesthesia. Complications of epidural or caudal anesthesia include: inadvertent dural puncture, vascular injection, and subarachnoid injection.

List examples of long-acting nondepolarizing muscle relaxants. Tubocurarine, metocurine, pancuronium, pipecuronium, and doxacurium are long-acting nondepolarizing medications.

Define the term purse-string suture. A purse-string is a continuous circular suture placed to surround an opening in a structure and cause it to close. This type of suture is often used around the appendix or in an organ such as the cecum, gallbladder, or urinary bladder before it is opened so that a drainage tube can be inserted.

List examples of types of surgical clamps. Clamps include hemostats, occluding clamps, graspers and holders, forceps, pickups, and needle holders

Describe the semi-Fowler position. In semi-Fowler, the patient is supine with the upper body flexed at approximately 45 degrees and the leg section is lowered.

List the characteristics of Stage III pressure ulcer. Stage III is characterized by full-thickness tissue loss. Subcutaneous fat may be visible but bone, tendon, or muscle is not exposed.

during a lap chole, the scrub person inspects the integrity of the insulation along the shaft. what nursing outcome is expected. the patient will be free from:

c. impaired skin integrity

What is fast-tracking? Fast-tracking is a process in which the patient bypasses the PACU phase I and is admitted directly to phase II.

What factors contribute to postoperative bradycardia? Contributing factors for postoperative bradycardia include hypoxemia, hypothermia, high spinal anesthesia, vagal stimulation, and some medications.

Which congential anomalies limit the skull's volume and increases the intracranial pressure?

craniosynostosis

What are the human body's three lines of defense against infection? The three lines of defense against infection are: (1) external barriers, such as the skin and mucous membranes; (2) the inflammatory response, which prevents an invading pathogen from reproducing and possibly involving other tissue; and (3) the immune response, which is triggered after the inflammatory response

What gram-positive anaerobe proliferates and causes symptoms of diarrhea, colitis, toxic megacolon, dehydration, and colonic perforation Clostridium difficile causes symptoms of diarrhea, colitis, toxic megacolon, dehydration, and colonic perforation.

List the minimum information needed for a surgical specimen identification. Information needed for a surgical specimen includes: correct patient name and identification number, specific origin of the specimen, and laterality (e.g., Jane Doe, 100001, right breast biopsy)

What are the key elements of informed consent? During informed consent, the patient must be informed about the proposed operation or other invasive procedure and its inherent risks, benefits, alternatives, and complications in terms that he or she can understand.

Describe the difference between a primary and secondary dressing. Primary dressings are placed directly over or in the wound. Secondary dressings are placed directly over the primary dressing.

What are the major categories of wound scaffolding materials? Allogeneic, biosynthetic, synthetic, and xenographic.

Define the term general anesthesia. General anesthesia is a reversible, unconscious state characterized by amnesia (sleep, hypnosis, or basal narcosis), analgesia (freedom from pain), depression of reflexes, muscle relaxation, and homeostasis or specific manipulation of physiologic systems and functions.

What devices are used to monitor and display the patient's CO2 concentration? A capnometer and capnograph are used to monitor and display the patient's CO2 concentration.

List the risk factors for hypothermia. Risk factors for hypothermia include: Female gender, age (very young or very old), length and type of surgical procedure, cachexia, significant fluid shift, use of cold irrigants, ambient room temperature, burns, and neuraxial anesthesia.

What factors are assessed during the PACU respiratory assessment? Respiratory assessment comprises rate, rhythm, auscultation of breath sounds for ventilatory adequacy, and the oxygen saturation level.

What are the time and temperature parameters for pre-vacuum steam sterilizers? Recommended time and temperature settings for pre-vacuum sterilizers are: 132-135° C (270-275° F) for 3-4 minutes, or follow manufacturer's instructions

What gram-positive organism is typically responsible for surgical site infections, bacterial endocarditis, septicemia, and urinary tract infections (UTIs)? Enterococcus is the gram-positive organism responsible for many healthcare-associated infections such as surgical site infections, bacterial endocarditis, septicemia, and UTIs.

Define the ASA P6 status. An ASA P6 status is defined as a patient declared brain dead whose organs are being removed for donor purposes.

What hydantoin skeletal muscle relaxant is used to treat malignant hyperthermia? Dantrolene (Dantrium) is used to treat malignant hyperthermia.

Define the term healthcare-associated infections. Healthcare-associated infections (HAIs) are those infections acquired by patients during hospitalization, with confirmation of diagnosis by clinical or laboratory evidence.

What is the causative agent for what is sometimes referred to as serum hepatitis? Hepatitis B virus causes serum hepatitis

What is the recommended temperature in the operating room (OR) to control bacterial growth? The recommended temperature to control bacterial growth is 68° F (20° C) to 73° F (23° C)

What is the causative agent of what is referred to as infectious hepatitis? Hepatitis A virus causes infectious hepatitis.

Define the term debridement. Debridement is the act of removing dead and devitalized tissue from a wound.

What type of wound healing occurs when the wound is allowed to heal from the inside toward the outer surface? Secondary intention occurs when the wound is allowed to heal from the inside toward the outer surface.

DCIS... a. What does it stand for? b. What is it also known as? c. Describe it. d. SSx? e. Mammographic findings? f. How is diagnosis made? g. What is the most aggressive histiologic type?

a. ductal carcinoma in situ b. intraductal carcinoma c. cancer cells in the duct w/o invasion (not penetrating basement membrane) d. usually none (usually nonpalpable) e. microcalcifications f. core or open bx g. comedo

Matthew Ryan, a 9-year-old boy, is admitted to the emergency department for a sledding accident, when he lost control of his sled and crashed into a tree. He is pale and in pain; his abdomen is tender and firm to palpation. The CT scan suggests rupture of the spleen with internal bleeding. Matthew's scheduled surgery will most probably be a/an:

a. laparoscopic splenic resection with sutured mesh overlay

Name a disadvantage of ORIF with plating technique.

additional soft tissue trauma due to open surgical intervention

Which vein, in addition to the gonadal and lumbar, should be clamped and ligated when performing a left kidney procurement?

adrenal

Which of the following hemodynamic concepts describes the resistance the heart must overcome to pump blood into the systemic circulation?

afterload

What are some hemodynamic effects of aortic cross-clamping?

afterload INCREASE hypertension Catecholamine and metabolite release Absence of blood flow below clamp

Colles' (wrist) fracture is a dorsally angulated fracture of the distal end of the radius. Most of these fractures can be managed successfully with closed reduction and immobilization, but external fixation is especially useful in the case of a comminuted intra-articular fracture. Internal fixation with Kirschner wires (K wires) is indicated when the distal end of the radius is severely comminuted and displaced. Select the nursing considerations for a patient with a Colles' fracture fixation.

b. Provide soft tissue instruments in OR and casting supplies available outside OR.

Marissa Walton is a 6-year-old girl with full-thickness burns involving both lower legs, circumferentially, excluding her feet, over less than 2% of her body surface area (BSA) and partial-thickness burns over less than 15% of her BSA, after her clothes caught fire during a camping trip when she stepped into the campfire. Based on the American Burn Association and the "Rule of Nines" classifications, Marissa's burns would be classified as ______________ with a burn surface area percentage of approximately ______________.

c. minor 9%

select the statement that most correctly matches a risk factor for adhesions with an appropriate preventitive strategy

c. patients with endometriosis may be best served with a laparoscopic approach

Greatest advantage for Lovenox

can be given outpatient

What are the branches of the left coronary artery?

circumflex and anterior descending arteries

What is Tx of LCIS?

close follow up (or bilateral simple mastectomy in hi risk patients)

Which of the following bones make up the pelvic girdle? 1. coccyx 2. acetabulum 3. sacrum 4. os coxae

coccyx, sacrum, os coxae

Identify the type of fracture that has more than two fragments.

comminuted

Know the structures of the biliary tract

common bile duct ampulla of vater intrahepatic ducts right hepatic duct left hepatic duct gallbladder cystic duct common hepatic duct

During a carotid endarterectomy, which arteries are initially incised?

common carotid

Calot's Triangle is formed by the

common hepatic duct, cystic duct, and cystic artery

Wilber Johnson, an 84-year-old retired musician of Jamaican descent, was seen by his family practice physician for urinary problems. His physician performed a digital rectal exam (DRE), which was abnormal, ordered a PSA, and referred him to a urologist. Wilber's PSA was 12 ng/ml. His transrectal prostatic biopsy was positive. The biopsy specimen determined a Gleason score of 10 and his AUA score was D2. Based on this clinical diagnostic information, what is the best surgical treatment plan for Wilber?

d. No surgery. Wilber is not a candidate for surgery

When preparing an adult for flexible bronchoscopy, what patient considerations must the nurse keep in mind?

d. all of the above

a number of screening tools have been developed for comprehensive geriatric assessment (CGA)

d. indentifying at-risk individuals

upon palpating the anterior abdominal wall of a patient you notice an intense reflexive muscular contraction in the abdominal muscles which is not correct>

d. it is best observed when the patient is perfectly supine.

The part of the femur that is most susceptible to fracture in conjunction with osteoporosis is:

d. neck

Which procedure is done for chronic dacryocystitis?

dacryocystorhinostomy

The primary drug utilized to treat MH is

dantrolene

What should be confirmed before administraion of any blood product?

date of expiration

The part of the femur that articulates with the acetabulum is the

femoral head

What is the most common type of hernia that occurs in females?

femoral hernia

During a triple arthrodesis, the capsule of the talonavicular, calcaneoocuboid and subtalar joint are circumferentially incised to

gain as much ankle joint mobility as possible

Woulds in which there is significant bacterial contamination, foreign bodies, or extensive tissue trauma are generally closed by which intention?

granulation

Which of the following is the primary node associated with metastatic disease?

sentinel

Two-thirds of the breast tissue rests on the pectoralis major muscle and the other one-third rests on the

serratus anterior

Which of the following are rotator cuff muscles? 1. teres major and teres minor 2. supraspinatus and teres minor 3. levator scapulae and infraspinatus 4. infraspinatus and subscapularis

2 and 4 only

Marvin Townsend is a 29-year-old man with paraplegia caused by spinal cord injury from a diving accident 4 years ago. He is transported into the trauma ED after being hit by a car as he was crossing the street in his wheelchair. He has a possible rupture of his liver and is scheduled for surgery. The perioperative nurse, on response to the trauma call, meets him in the ED to conduct a preoperative assessment and prepare for his procedure. Marvin tells the nurse that he is latex-sensitive because he was told that as a spinal cord injured person he should always be considered latex-allergic. An appropriate risk reduction strategy for Marvin's latex sensitivity/allergy would be to:

b. alert the OR and postoperative recovery areas of his latex allergy, check the setup for latex-containing items, and replace those items with non-latex alternatives

When preparing for pars plana vitrectomy in the posterior segment, the perioperative nurse must be aware that a combined scleral buckling procedure may be necessary. Other important information the perioperative nurse should know before preparing the OR for the procedure includes the location of the ocular problem, the surgeon's plan to address the problem, and the instrumentation and biomedical equipment and devices to be used. Technologic advances in ophthalmic surgery require that perioperative nurses be familiar with complex biomedical equipment. A best practice, stated below, relevant to the safe use of complex equipment would be:

b. demonstrate competence with equipment and check each piece carefully before the patient arrives in the OR.

Marla Moriarity, a 32-year-old recently unemployed and uninsured college professor, is diagnosed with an early ectopic pregnancy. Her obstetrician has given her the option of surgical or medical therapy. She is well-informed about the details of each medical and surgical option, but her primary concern about her condition is that her decision will be based upon her strong cultural belief system that will not sanction the destruction of the fetus. Marla's treatment decision will most likely be:

b. laparoscopic injection of methotrexate to the extrauterine gestational sac.

A plain radiograph of a patient following a knife wound to the left side of his neck showed elevation of the left hemidiaphragm, narrowing of the left intercostals spaces, and displacement of the trachea to the left. You suspect:

b. pneumothorax due to knife penetration of the cervical pleura.

Question text When setting up for a gastrectomy, the scrub person will ensure that appropriate instruments are available to clamp and ligate the:

b. splenic vessels.

Osteomalacia is a metabolic bone disease characterized by inadequate mineralization of bone, which leads to a reduced absorption of calcium and phosphorus. This malabsorptive condition is due to a deficiency of ______________ that can be treated by __________________.

b. vitamin D; dietary supplements and exposure to sunlight

What structure traverses the opening indicated by the arrow in the following figure?

c. Aorta

Acoustic neuromas arise from the Schwann cells of the vestibular portion of the eighth cranial nerve and are benign tumors. What is the postauricular incisional approach that offers the best chance of saving the integrity of the facial nerve?

c. Translabyrinthine

Many gynecologic conditions and diseases cause chronic pelvic pain. Both surgical and nonsurgical therapies include invasive manipulations. Select the least invasive treatment option for chronic pelvic pain that is recommended by the Agency for Healthcare Research and Quality (AHRQ).

c. Trigger point injections and acupuncture

When using the posterolateral incisional approach, which muscles are retracted anteriorly to expose the femoral fracture site

c. Vastus lateralis and vastus intermedius

what mechanisms do chest catheters and drainage systems use to remove fluid and air from the pleural cavity

c. expiratory pressure and gravity

 The renal artery and vein enter and exit the kidney on the medial side of the organ through a concave area known as the:

c. hilum.

Prone Positioning Complications

*Eye injury *VAE *ETT- mainstem, extubation *Brachial plexus injury *Facial edema

Supine Surgeries: examples

*Cataracts *Tonsillectomy *Mastectomy *Laparoscopic Cholecystectomy *Exploratory Laparotomy *Total Knee Replacement

? is the most frequently damaged nerve of the lower extremity

*Common Peroneal Nerve. aka: fibular nerve *Mechanism of injury -Compression between the head of the fibula and the metal frame of the stirrup *May be mistaken for sciatic nerve injury

Brachial Plexus: Mechanisms of Injury- Compression

*Compression -Shoulder braces Utilized during steep trendelenburg position -Brachial plexus is compressed between the clavicle and 1st rib, especially when the shoulder is moved dorsally -Correct placement: Place over acromioclavicular joint -Even with correct placement injury can occur

Femoral Nerve: Mechanism of Injury

*Compression at the pelvic brim by the blade of a retractor *Stretched by excessive angulation of the thigh when patient is in lithotomy *Most patients with femoral nerve injury have DM

Ulnar Nerve: Two major sites of damage *2 most common sites of damage are

*Condylar groove -External compression against the posterior aspect of the medial epicondyle of the elbow -Less adipose padding, M>F *Cubital tunnel -Compression of the nerve distal to the condylar groove -More likely when roof of the cubital tunnel is more developed, M>F -Worse with elbow flexed > 110 °

Lithotomy Position: Examples

*Cystoscopy *Ureteroscopy *Transurethral Resection of Prostate *Robotic Assisted Laparoscopic *Prostatectomy *Vaginal Hysterectomy *Hysteroscopy *Dilation and Curettage

Physiologic Changes in Reverse Trendelenberg Position: Cardiovascular

*Decreased Preload, Cardiac Output and Arterial Pressure *Baroreceptors increase HR and PVR (remember this response can be obtunded by GA)

Physiologic Changes in Supine Position: Pulmonary

*Decreased Tidal Volume (inefficient lung mechanics) *Decreased Functional Residual Capacity (cephalad displacement of the diaphragm)

Lateral Decubitus: UE *Dependant arm is on the ? *Non dependant is on the ? *Elbows are ?

*Dependent arm placed on arm board with shoulder less than 90° *Nondependent arm supported on 1-2 pillows with should less than 90° *Elbows flexed and supported

Prone Position

*Head -Arms Neutral alignment and supported (head pillows, horse shoe), or turned on the side -Eyes, nose, mouth, ears free from pressure *Trunk -Chest bolster (clavicle and iliac crest), Abdomen hang free, protect breasts and genitalia *Arms -Tucked or abducted (superman), 90 degrees, padded and secured *Legs -Padded, compression devices to prevent pooling

Lithotomy Position

*Head -Same as supine *Arms -Tucked or Abducted *Legs -Raised and lowered simultaneously -Pressure points meticulously padded

Sitting Position

*Head -A variety of prongs or head rest devices exist. The key is to ensure neutral positioning of the neck. *Arms - Arms are elevated to rest in a neutral position at the sides of the patient *Legs - Legs are padded as necessary

Lithotomy: Nerve injuries *Most common never injury is ?

*The common peroneal nerve is the most vulnerable nerve in this position -Caused by compression at the fibula head from the stirrup **Other nerve injuries *Sciatic nerve -External rotation of thigh and leg can cause traction *Saphenous nerve -Compression by the leg holder and tibia *Obturator nerve -Calf pressure from the leg holder *Femoral nerve -Nerve is pressured by pubic ramus with excessive thigh flexion

Obturator Nerve Injury: Signs/Symptoms

*Inability to adduct the leg *Diminished sensation over the medial aspect of the thigh

Median Nerve Injury: Signs/symptoms

*Inability to oppose the 1st and 5th fingers *Decreased sensation to the palmar surface of the thumb and lateral 2½ fingers

Physiologic Changes in Reverse Trendelenberg Position: Pulmonary

*Increased FRC (and all lung volumes relative to other positions) *Decreased Peak Inspiratory Pressure

Physiologic Changes in Trendelenberg Position: Cardiovascular

*Initially an increase in Central Blood Volume *Activates baroreceptor response decreasing CO, PVR, HR, and Blood Pressure

Common Peroneal Nerve: Injury Prevention *does the head of the fibular touch the stirrup?

*The head of the fibula should not touch the stirrup, or it should be padded

Supine: LE Positioning *Legs remain ? *Putting the table in the ? position helps flexion

*Legs must remain uncrossed *Pressure points to be monitored/padded Occiput, heels, lumbar lordosis *May place table in "lounge chair" position with hips/knees flexed and trunk slightly elevated

Sciatic Nerve: Injury Prevention

*Limit time in lithotomy *When putting the patient in stirrups, bring legs midline and raise them to place in the stirrups at the same time *When taking the patient out of lithotomy the procedure is reversed *IM injections only in upper outer quadrant of the buttocks

Lithotomy *Nerve injury will increase with this position at ? hour, BMI <?, and....

*Nerve injuries increase with this position if the duration of surgery is longer than 4 hours, the patient's BMI is <20, or the patient is a smoker or a diabetic

Position related injury

*Peripheral Nerve Injury -18% of cases in ASA Closed Claims- 2nd only to death *Blindness/Corneal Abrasion *Compartment Syndrome *Decubitus Ulcer *Amputation *Loss of airway

Risk Factors for Nerve Injuries

*Positioning devices *Length of procedure -Exceeding 2 hours duration (some sources say 4 hours) *Body habitus -Extremely underweight e.g. BMI <18.5 kg/m² -Obesity e.g. BMI >30 kg/m² *Preeixisting Pathology -Diabetes Mellitus -Low-flow blood states *Alcohol or tobacco use *Impaired oxygen-carrying capacity *Malnutrition *Impaired sensorium *Limited joint mobility *Anesthesia technique -Regional anesthesia -Hypotensive technique May cause nerve ischemia -General anesthesia

Physiologic Changes in Prone Position: Pulmonary

*Proper positioning allows abdomen to hang free and improve diaphragmatic movement *Improved V/Q matching when properly positioned

Supine: UE Adducted *Palms should be ? the thigh *Humeral head can compress the ?

*Secure close to patient's side, palms touching the thigh, on a padded surface *Humeral head can compress the brachial plexus when arms are tucked *Avoid metal surfaces of the OR table

Positioning and Documentation

*Standards for Nurse Anesthesia Practice -Standard V, f *Responsibility of padding and positioning, thorough documentation of how the patient is carefully positioned, padded and constantly monitored

Physiologic Changes in Lithotomy Position: Cardiovascular

*Temporary increase in preload when legs are raised. *Opposite when legs are lowered

Radial Nerve Injury: Signs/Symptoms *#1 sign is ?

*Wrist drop *Inability to extend the metacarpophalangeal joints *Weakness of abduction of the thumb *Decreased sensation over the dorsal surface of the lateral 3 ½ fingers of the hand

Poor results from plating usually occur because of

*poor selection of plate size *screws don't go through both cortices *inadequate compression

Name an indication for treating a mid-shaft fracture of the femur with compression plating.

*third degree open fracture with a compromised blood supply *multiple trauma *no access to rodding system or the correct sized rod

In the diagram, identify the following structures: Inferior mesenteric vein; Splenic vein; Right gastric artery; Portal Vein

1. Portal Vein......................2. Right gastric artery 3. Splenic vein....................4. Inferior mesenteric vein

Using the UTHS classification: This ulcer involves the subcutaneous tissue and is ischemic

1C

The 2 posterior spinal arteries provides what % of spinal cord blood flow and supply what proportion of the spinal cord?

25% of S.C. blood flow 1/3 posterior portion of S.C. Supply the sensory tracts Receive flow from posterior and inferior cerebellar arteries, vertebral arteries, and the Radicular arteries

Which of the following is the normal value of the BUN for adults? A. 7-18 mg/dl B. 18-29 mg/dl C. 29-40 mg/dl D. 40-51 mg/dl

A. 7-18 mg/dl

Q. The intentional mistreatment or harm of another person is known as which of the following?

A. Abuse Explanation: Abuse is intentional harm to another human. This may or may not be physical. Nurses have an obligation to report abuse by other nurses or family/significant others if they observe signs or questionable behavior. They also have an obligation to refrain from abusing patients and family members in any way

The point of separation between the posterior rectus sheath superiorly and absence of the sheath inferiorly is called the: A. Arcuate line B. Linea alba C. Semilunaris D. Aspera

A. Arcuate line

Q. The errors with specimens could be caused by which of the following?

A. The specimen requisition and the specimen container. Explanation: For years there have been errors made with patient specimens. The specimen requisitions and the specimen containers have been identified as the cause of some of these errors. Causes such as unlabeled specimens, empty containers, incorrect side designation, no identified tissue site or incorrect or no patient name has the potential to cause these errors also.

Q. Which of the following weight loss surgeries is solely a restrictive type of surgery?

A. vertical banded gastroplasty Explanation: Restrictive and malabsorptive-restrictive are the two types of surgeries primarily used to promote weight loss. In gastric restriction, also known as vertical banded gastroplasty and adjustable gastric banding, the size of the stomach is surgically decreased so that a patient feels full after eating a small amount of food. The other choices are malabsorptive-restrictive procedures.

Q. Which of the following statements about rheumatoid arthritis (RA) is least accurate?

A.Nursing interventions should begin with teaching the patient how to prevent RA. Explanation: This is the least accurate statement. Prevention of RA is not possible at this time. However, education should include information on symptom recognition to promote early diagnosis and treatment.

In addition to the zygoma, which other bone compromises the zygomatic arch? A. Maxilla B. Temporal C. Mandible D. Sphenoid

B. Temporal

The cartilage distal to the thyroid cartilage and proximal to the thyroid gland is the

cricoid

Which metacarpals is most commonly fractured when associated with a "boxer's fracture?

fifth

What statement describes second-degree AV block?

some, but not all, atrial beats are conducted to the ventricles

Radial Nerve: May be injured if... *Don't continue BP cuff going off at q?

*Arm slips off OR table *External compression by ether screen or other device *External compression by NIBP cuff -Greater than Q2 minutes for a prolonged time *Placement of an IV in the dorsolateral aspect of wrist can cause injury to a superficial branch of the radial nerve associated with a neuroma formation

Obturator Nerve Injury: Mechanism of Injury

*Excessive flexion of the thigh to the groin *During use of forceps in a difficult forceps delivery

Physiologic Changes in Supine Position: Cardiovascular

*minimal alteration, initial increase in preload, SV, CO- attenuated compensatory mechanisms with Anesthesia

Identify the type of fracture that has more than two fragments.

Comminuted

Describe the induction phase of anesthesia. Induction begins with administration of anesthetic agents and continues until the patient is ready for positioning or surgical prepping (surgical prep), surgical manipulation, or incision.

Describe the maintenance phase of anesthesia. Maintenance continues from the incision (or earlier) until near completion of the procedure.

What is the optimal time to administer antibiotic prophylaxis? Within 1 hour of the incision.

Describe the mechanism of action responsible for healing via negative pressure wound therapy. Negative pressure results in mechanical tension on the tissues, causing the development of new blood vessels, mitosis, and dilation of the arterioles; and produces a centripetal force that helps to pull the wound edges together. It promotes evacuation of excess wound fluid and reduces bacterial colonization in the wound bed.

Describe measures taken to prevent deep vein thromboembolism (DVT) or pulmonary embolus. Measures to prevent DVT or pulmonary embolus include administration of prophylactic heparin, aspirin, dextran, or warfarin; and application of intermittent pneumatic compression device (IPCD) or graduated compression stockings.

Define postoperative hypothermia. Postoperative hypothermia is defined as a temperature less than 36° C (96.8° F).

Define the term subcuticular suture. Subcuticular sutures are placed completely under the epidermal layer of the skin. This technique is often used for cosmetic closure.

Define tensile strength. Tensile strength is the amount of weight (breaking load) necessary to break a suture (breaking strength); it varies according to the type of suture material.

Define the ASA P4 status. An ASA P4 status is defined as a patient with a severe systemic disease that is a constant threat to life.

Define the ASA P3 status. An ASA P3 status is defined as a patient with a severe systemic disease that limits activity, but is not incapacitating.

Describe the parts of a surgical needle. The eye, the body, and the point or tip comprise the parts of the surgical needle.

Define the primary suture line. A primary suture line is composed of those sutures that obliterate dead space, prevent serum from accumulating in the wound, and hold the wound edges in approximation until healing takes place.

Describe the "no touch" sharps technique Sharps are placed in a predesignated basin, tray collection device, or safe ''neutral'' zone on the field, from which the surgeon can retrieve the sharps. After use, the item is placed back in the neutral zone, and the scrub person retrieves it

Define the secondary suture line. The secondary suture line is composed of those sutures that supplement the primary suture line.

Define the term sentinel event. Sentinel events are unexpected occurrences involving death or risk of serious physical or psychologic injury to a patient.

Define the term "never events." Never events are events resulting in unsafe patient outcomes that are no longer considered reimbursable under the Centers for Medicare & Medicaid Services (e.g., wrong-site surgery

Define the acronym SBAR. SBAR stands for: S=Situation, B=Background, A=Assessment, R=Recommendation.

Define the term active transport. Active transport is the process by which molecules are moved across a cell membrane against a concentration gradient, with the use of external energy

Define the purpose for Gram staining. Gram staining is a procedure for staining bacteria that is the first step in classifying and differentiating them into two large groups (gram-positive and gram-negative) based on the chemical and physical properties of their cell walls.

Define the term airborne transmission. Airborne transmission occurs by dissemination of an infectious agent by evaporated droplets or dust particles suspended in the air that can be inhaled or deposited on a host.

Define class I Clean Wounds. Class I Clean Wounds are uninfected operative wounds in which no inflammation is encountered, and the respiratory, alimentary, and genitourinary tracts are not entered. The wound is primarily closed and can be drained with a closed wound drainage system.

Define the term deep incisional surgical site infection (SSI). A deep incisional SSI is one involving deep soft tissues (e.g., fascial and muscle layers) of the incision and at least one of the following: purulent drainage from the deep tissue, incision spontaneously dehisces or is deliberately opened by a surgeon, abscess or other evidence of infection involving the deep incision, and diagnosis of a deep incisional SSI by a surgeon or attending physician.

Describe the time periods associated with each phase of wound healing. The proliferative phase begins within hours of the injury. The remodeling phase begins after approximately 2 to 4 weeks, depending on the size and nature of the wound and may last 1 year or longer. Contraction begins approximately 5 days after the wound onset and peaks at 2 weeks, gradually shrinking the entire wound.

Define the term dermis. The dermis is the layer beneath the skin that provides strength and structure and is composed of collagen.

What is the steam saturation point that is necessary to kill microorganisms? The recommended steam saturation temperature is 250° F (121° C).

Define the term disinfection. Disinfection is the process of eliminating many or all pathogenic organisms, except bacterial spores, from inanimate objects.

List examples of physical risks to patients in the perioperative environment. Physical risks include any damaging or noxious element that comes into contact with the patient to cause harm, such as: electrosurgical/laser beam, pooled prep solution, glutaraldehyde retained in an endoscope, or a retained foreign object.

Define the term electrolytes. Electrolytes are substances in the blood essential to transmission of nerve impulses, regulation of water distribution, contraction of muscles, generation of adenosine triphosphate (ATP, needed for cellular energy), regulation of acid-base balance, and hemostasis.

When are liquid peracetic acid sterilant systems used? Liquid peracetic acid systems are used for devices that are heat sensitive, can be immersed, and have been validated by the device manufacturer for use in these systems. They are suitable for devices that cannot be sterilized with terminal sterilization methods.

Describe Standard Precautions. Standard Precautions are applied to all patients receiving care, regardless of presumed infection status. They are the primary strategy for successful infection control and prevention and include barrier protection, such as gowns and gloves to protect healthcare workers.

A hysterectomy is an example of what class of wounds? A hysterectomy is an example of a class II Clean Contaminated Wound.

Describe the purpose of a surgical drain. Drains are important for the control of ecchymosis and provide egress of air and fluids to be evacuated from the operative site. They also eliminate dead space.

Define the term surgical wound. A surgical wound is one caused by an incision or excision.

Describe the purpose of a surgical dressing. A wound dressing offers cushioning and protection of the wound from trauma and gross contamination, and absorption of drainage.

List examples of accessory and ancillary instruments. Suction tips and tubing; irrigators-aspirators; electrosurgical devices; and special-use devices, such as probes, dilators, mallets, and screwdrivers.

Describe the purpose of surgical retractors. Retractors are used for holding back the wound edges, structures, or tissues to provide exposure of the operative site.

What is the function of the barbs in barbed suture? The barbs help the tissue to stay approximated with equal wound tension and eliminate the need to tie knots.

Describe the purpose of surgical soft goods. Surgical sponges are used to affect hemostasis via direct pressure, absorb intraoperative blood loss and drainage, aid in blunt dissection, pack viscera from the field, and keep areas of the wound moist

List the advantages of high-speed ultrasonic device surgery. Advantages of high-speed ultrasonic surgery include: no surgical plume or odor, minimal aerosolization, reduced damage to adjacent tissue, retention of tactile feedback, no nerve or muscle stimulation, no stray electrical or laser energy, and precise cutting and control.

Describe the purpose of white-balancing. White-balancing adjusts the camera to all other optical components (endocoupler, light cable, laparoscope) to enable the camera to reference white so that it can identify all primary colors properly.

Describe the Fowler position. In the Fowler position, the patient is supine with the upper body flexed at approximately 90 degrees and the leg section is lowered.

List the forces that contribute to injury of the skin and underlying tissue during positioning. Pressure, shear, friction, moisture, heat, cold, and negativity contribute to positioning injury.

Malnutrition, drug therapy/radiation therapy, infection, arterail/venous insufficiency, and DM are all factors that what?

Factors that impact wound healing

Differentiate between fiberoptic endoscopes and videoscopes. Fiberoptic endoscopes have an eyepiece with a lens for visualization; the image is carried through the endoscope via a bundle of tiny glass fibers. Videoscopes have, at their distal end, a video chip that provides an image that is directly viewed on a monitor; a videoscope does not have an eyepiece for direct viewing, the eyepiece is replaced with an endoscopic video camera.

List the indications for cryosurgery. Indications for cryosurgery include: skin tumors, liver tumors, prostatic cancer, and cervical dysplasia.

Where does the falciform ligament pass? What does it contain?

Liver to supraumbilical abdominal wall down to umbilicus and also to diaphragm in its free edge contains the ligamentum teres - obliterated umbilical vein The two edges of the falciform ligament split on the anterosuperior surface of the liver to form the coronary ligament on each side

What condition can be a postoperative complication of compression plating?

Localized osteoporosis

These bones are broad plates that flair inward from the pedicles to form part of the vertebral arch.

Laminae

Define the term suture Suture is a generic term for all materials used to sew severed body tissues together and to hold these tissues in their normal position until healing takes place; to suture is to stitch together cut or torn edges of tissue

List examples of instruments or other devices used for blunt dissection. Blunt dissectors include gauze dissectors (e.g., peanuts, pushers, kitners), a sponge on a stick, the back of a knife handle, and the surgeon's finger or hand.

Define the term endoscope. An endoscope is a tube inserted into a natural body orifice or through a small incision to access internal organs or structures.

List the advantages of laser surgery. The advantages of laser surgery are: seals small blood vessels, lymphatics, and nerve endings; sterilizes tissue; decreases postoperative stenosis; reduces tissue damage; reduces operative and anesthesia time; allows a shift to more ambulatory surgery procedures; allows more use of local anesthetics instead of general anesthetics; and provides quicker recovery and return to daily activities.

Describe the benefits of laser surgery. Benefits of laser surgery include: reduced operative and anesthesia time, a shift to more ambulatory surgery procedures, more use of local anesthetics instead of general anesthetics, quicker recovery and return to daily activities.

List the advantages of robotic surgery. Robotic surgery typically minimizes fatigue associated with holding and moving the scope, returns scope control to the physician, accommodates quick scope movements through a computerized memory, shortens the procedure time by minimizing scope positioning time, provides a steady image that enhances video quality, ensures consistent scope positioning and movement, and frees the surgeon's hands to perform other tasks.

Define direct coupling. Direct coupling occurs when the active electrode accidentally touches a noninsulated metal instrument, allowing the electrical energy to flow from one to the other (metal-to-metal sparking).

List the benefits of argon-enhanced electrosurgery. Benefits of argon-enhanced electrosurgery include: rapid coagulation, reduced blood loss, reduced risk of rebleeding, noncontact tissue coagulation, reduced depth of penetration by electrical energy, and less adjacent tissue damage.

What factors contribute to postoperative tachycardia? Postoperative tachycardia is often caused by pain, hypoxemia, hypovolemia, increased temperature, and anxiety.

List the categories included in the Aldrete score. The Aldrete score evaluates Activity, Respiration, Circulation, Consciousness, and O2 Saturation. Numerical scores are assigned to each category based on the patient's response.

Describe the correct procedure for tucking a patient's arms during supine positioning. To tuck the patient's arms: wrap the draw sheet smoothly around the arm, extending to above the elbow, and then tuck the draw sheet under the patient's body instead of under the mattress.

List the characteristics of Stage IV pressure ulcer. Stage IV is characterized by full-thickness tissue loss with exposed bone, tendon, or muscle. Slough or eschar may be present on some parts of the wound bed.

Define the term retention suture Retention sutures are placed at a distance from the primary suture line to provide a secondary suture line, relieve undue strain, and help obliterate dead space

List the different types of endomechanical staplers used for internal stapling. Ligating and dividing staplers (LDSs), gastrointestinal anastomosis (GIA) staplers, thoracoabdominal (TA) staplers, end-to-end anastomosis (EEA) staplers, laparoscopic hernia mesh staplers, open hernia mesh staplers, and endo-GIA devices are used internally

How does the edema of the dermis in inflammatory carconima of the breast appear?

Peau d'orange (orange peel)

Describe the difference between phase I and phase II of postanesthesia care. Phase I activities focus on primary assessment, breathing and circulation, and facilitating the patient to a level of physiologic stability. Phase II PACU prepares the patient for discharge.

What factors are considered for the PACU neurologic assessment? The nurse will consider the following: Has the patient reacted (awakened from anesthesia)? Can the patient follow commands? Is the patient oriented, at least to name and hospital? Can the patient move all extremities?

List the sequence of steps to remove a soiled surgical gown.To remove a soiled gown: (1) Wipe gloves. (2) Bring the neck and sleeve of the gown forward and off the gloved hand, turning the gown inside out and everting the cuff of the glove.

What gram-positive organisms are responsible for necrotizing fasciitis? Group A streptococci are responsible for necrotizing fasciitis.

What is the process that is characterized by performing a preoperative verification process, marking the operative site, and conducting a ''time-out'' immediately before starting the procedure? The Universal Protocol is characterized by performing a preoperative verification process, marking the operative site, and conducting a ''time-out'' immediately before starting the procedure

What is the difference between blood typing and blood crossmatching? Blood typing refers to the test to determine the ABO and Rh blood type. Crossmatching refers to testing the compatibility of the recipient's serum and the donor's red blood cells

List the sequence for surgical skin preps. Surgical skin preps are conducted in the following manner: Prepare (''prep'') the cleanest area first and then move to the less clean areas (clean to dirty).

What is the difference between high-, intermediate-, and low-level disinfectants? High-level disinfectants kill all microorganisms except high numbers of bacterial spores. Intermediate-level disinfectants may kill tubercle bacilli, vegetative bacteria, and most viruses and fungi but not bacterial spores. Low-level disinfectants kill most vegetative bacteria and some viruses and fungi.

Jordan Gribbins, a 69-year-old pipe smoker with a history of cardiac disease and atrial rhythm disturbance, is scheduled for a supratentorial craniotomy and is securely positioned in a semisitting position. The perioperative nurse and anesthesia provider have discussed the anesthesia plan and Mr. Gribbin's surgical and anesthesia risks, and agree that he is at risk for venous air embolism (VAE). Sequential compression devices have been placed on both legs. Pulmonary lines have been placed and the nurse has prepared an area for preparation of TEE (transesophageal electrocardiography), if needed. During the procedure, Mr. Gribbins reacted with decreases in arterial pressure and end-tidal CO2. Precordial "washing machine" sounds emanated from the Doppler. What should the perioperative nurse anticipate as a possible next response?

c. Reposition the patient in left lateral decubitus position to entrap air in the right atrium.

A herniation of the cul-de-sac of Douglas is a/an

enterocele


Related study sets

Anatomy and Physiology (heart and blood vessels), Teas 6

View Set

Intro to business chapter 4 quiz

View Set

F.A Davis Alterations in Women's Health

View Set

River Valley Civilizations in the Ancient Near East

View Set

Marketing Research, Troy University, Exam 2

View Set

Healthcare Chapter 17:1-2 Providing First Aid and vocab

View Set

Phlebotomy: Chapter 13 Self Study

View Set

Sociology of Sex and Reproduction

View Set

Leadership and Independence at the Federal Reserve

View Set